145
TEMAS DE EXAMEN Internado de Pediatría - 2013 - Int@ Valentina Mansilla Vera Calotitos

Temas de examen Pediatria

Embed Size (px)

DESCRIPTION

Temas de examen internado pediatría UFRO-HHHA

Citation preview

Page 1: Temas de examen Pediatria

Oph ytg

2013

TEMAS DE EXAMEN

Internado de Pediatría - 2013 -

Int@ Valentina Mansilla Vera

Calotitos

Page 2: Temas de examen Pediatria

CONTENIDOS

1. Paro Cardiorespiratorio en Pediatría ....................................................... 3

2. Asfixia Perinatal ..................................................................................... 10

3. Apnea en el Lactante ............................................................................. 15

4. Síndrome Distrés Respiratorio en el RN.................................................. 20

5. Ictericia Neonatal ................................................................................... 23

6. ICC .......................................................................................................... 28

7. Cardiopatías congénitas ........................................................................ 29

8. Síndrome Convulsivo .............................................................................. 33

9. Síndrome Purpúrico ................................................................................ 40

10. Leucemia en el Niño ............................................................................... 44

11. Síndrome febril sin foco del lactante ........................................................ 4

12. Síndrome febril prolongado ..................................................................... 7

13. Infección Tracto Urinario ....................................................................... 11

14. Meningitis Bacteriana ............................................................................ 15

15. Exantemas Virales .................................................................................. 20

16. Infección Respiratoria Alta ..................................................................... 25

OMA................................................................................................. 25

Sinusitis ............................................................................................ 27

FaringoAmigdalitis .......................................................................... 28

Laringitis aguda ................................................................................. 2

17. Estridor en el lactante .............................................................................. 4

18. Sibilancias Recurrentes en el niño ............................................................ 8

19. Asma en menores de 15 años ................................................................ 14

20. Influenza ................................................................................................ 23

Page 3: Temas de examen Pediatria

21. Neumonía .............................................................................................. 26

22. Constipación .......................................................................................... 31

23. Diarrea Aguda ........................................................................................ 35

24. Diarrea Crónica ...................................................................................... 43

25. Síndrome Hemolítico Urémico ............................................................... 47

26. Hepatitis Aguda ..................................................................................... 50

27. Talla Baja ............................................................................................... 54

28. Vacunas de uso habitual ........................................................................ 60

Consideraciones Generales .............................................................. 60

Tipos de Vacunas ............................................................................. 62

Vacunas Vivas Atenuadas ................................................................ 63

Anexo: Campaña de vacunación contra W-135 .............................. 71

Elaborado por: Int@ Valentina Mansilla Vera;

Page 4: Temas de examen Pediatria

1. PARO CARDIORESPIRATORIO EN PEDIATRÍA

Aspectos destacados de las Guías de la American Heart Association de 2010 para para reanimación cardiopulmonar y atención cardiovascular de emergencia

Part 13: Pediatric Basic Life Support: 2010 American Heart Association Guidelines for Cardiopulmonary Resuscitation and Emergency Cardiovascular Care. Circulation.

2010;122:S862-S875,

Part 14: Pediatric Advanced Life Support: 2010 American Heart Association Guidelines for Cardiopulmonary Resuscitation and Emergency Cardiovascular Care.Circulation.

2010;122:S876-S908

Apunte de clase “RCP pediátrico” realizada por Dr. Sergio Puebla.UFRO 2010.

CASTILLO A., PARIS E., Apunte Reanimación cardiopulmonar pediátrica, PUC.

DEFINICIÓN

Toda situación clínica que comprende un cese inesperado, brusco y potencialmente reversible de las funciones respiratorias y/o cardiocirculatoria espontáneas.

ETIOLOGÍA

El PCR en el niño es un evento en general secundario a un evento hipóxico, a una hipoxemia, de modo tal que el pronóstico es muy malo, porque bastan 4 minutos para que una neurona sin oxígeno se muera. A diferencia del adulto, en que el paro cardiorrespiratorio es un evento que en un 90% es por un trastorno de arritmia A partir del año, las causas circulatorias comienzan a aumentar. De hecho, hoy día se dice que a partir de los 8 años, cuando el paciente tiene 30 kilos, la prioridad es desfibrilar porque lo más probable es que sea secundario a un trastorno del ritmo severo.

CUADRO CLÍNICO

Secundario a evento hióxico

• Si hay falla respiratoria va a estar hipóxico, también va a estar alterado de conciencia, taquicárdico, cianótico. Va a tener una falla respiratoria evidente, con uso de musculatura accesoria

Secundario a falla circulatoria

• Inicialmente habrá un síndrome adrenérgico, estará taquicárdico, los pulsos van a ser firmes. Si esto se perpetúa va a caer el llene capilar, va a haber una redistribución de flujos, enviando sangre a los territorios nobles en desmedro de los otros. Por lo tanto el paciente va a orinar menos, la piel va a estar pálida.

Page 5: Temas de examen Pediatria

TRATAMIENTO DE EMERGENCIA

SOPORTE VITAL BÁSICOPEDIÁTRICO

Las Guías de la AHA de 2010 para RCP y ACE recomiendan cambiar la secuencia de los pasos de SVB/BLS de A-B-C [Airway, Breathing, Chest compressions] a C-A-B en adultos, niños y lactantes (excepto RN).

Al cambiar la secuencia a C-A-B, las compresiones torácicas se inician antes y el retraso de la ventilación es mínimo (sólo el tiempo necesario para aplicar el primer ciclo de 30 compresiones, alrededor de 18 seg.; cuando participen 2 personas en la reanimación de un lactante o un niño, el retraso será incluso menor).

La mayoría de las víctimas de paro cardíaco extrahospitalario no reciben RCP por parte de un testigo presencial. Esto puede deberse a múltiples razones, pero una de ellas puede ser el hecho de que la secuencia de pasos A-B-C comienza con el procedimiento que le resulta más difícil al reanimador, es decir, abrir la vía aérea y dar ventilaciones. Empezar por las compresiones torácicas puede animar a otros testigos a iniciar la RCP

Page 6: Temas de examen Pediatria

La mayoría de las víctimas de paro cardíaco extrahospitalario no reciben RCP por parte de un testigo presencial. Esto puede deberse a múltiples razones, pero una de ellas puede ser el hecho de que la secuencia de pasos A-B-C comienza con el procedimiento que le resulta más difícil al reanimador, es decir, abrir la vía aérea y dar ventilaciones. Empezar por las compresiones torácicas puede animar a otros testigos a iniciar la RCP

Se ha eliminado de la secuencia la indicación de “Observar, escuchar y sentir la respiración” para valorar la respiración después de abrir la vía aérea. Tras aplicar 30 compresiones, el reanimador único abre la vía aérea de la víctima y aplica 2 ventilaciones.

A

Vía aérea permeable, traccionando la cabeza hacia atrás y el mentón hacia arriba para lograr despejar la vía aérea. Si se sospecha lesión de trauma cervical se debe subluxar la mandíbula hacia adelante para despejar la vía aérea y mantenerla permeable. • Cuando ocurre la obstrucción de la vía aérea por una cuerpo extraño, se utiliza la

Maniobra De Heimlich • En los lactantes se toma en el antebrazo boca abajo

con inclinación de 45º y se dan 3-4 golpes en la región interescapular, se da vuelta y se comprime el tercio medio del esternón.

• En los niños más grandes es igual que en adulto. Se empuña la mano y se comprime en el epigastrio hacia arriba

• Se evalúa si respira para ventilar boca-boca o boca-nariz.

B

Se debe ventilar boca-nariz o boca-boca. • Se debe dar 2 a 5 respiraciones lentas de 1 a 1,5 segundos, para

asegurarse que al menos dos sean efectivas. • Con una frecuencia en el niño menor de 20 por minuto y de 15 por

minuto en el adolescente y preescolar. La bolsa de ventilación (ambú es un nombre comercial) es el gold standard para lograr una adecuada oxigenación y ventilación, es decir, niveles óptimos de oxígeno y de CO2. Lo ideal es que tenga reservorio porque si no tiene lo que va salir no es más allá que un 40 % en cambio con reservorio va a llegar prácticamente a un 98% de fio2 y los más importante es que hay estudios en que cuando el paro ocurre fuera del hospital, el uso de la bolsa de ventilación es exactamente igual que intubar al paciente. Una cosa importante es que a veces se debe hacer presión sobre el cricoides (maniobra de Sellick), para colapsar el esófago y evitar que el esófago se llene de aire porque eso a veces permite la regurgitación (aunque no sale en la guía, lo que hace el doctor el instalar una sonda nasogástrica y aspira antes de ventilar para evitar que el contenido gástrico llegue a la vía aérea ante un eventual reflujo) Es importante sellar bien con la máscara, que debe incluir nariz y boca (hay de distintos tamaños según la edad). Con el pulgar y el índice se hace una “C”

Page 7: Temas de examen Pediatria

para sellar la máscara y con el meñique, anular y medio se hace un tridente para traccionar la mandíbula hacia arriba (todo con una mano) para que con la mano hábil se ventile. Si son 2 reanimadores, uno puede sellar con la técnica de la “C” utilizando las 2 manos mientras el otro ventila.

C

La palpación de los pulsos para evaluar si el corazón partió, está obsoleta.

Técnica de compresión

• Objetivo: lograr una frecuencia de compresión de al menos 100/min • Profundidad: al menos un tercio del diámetro torácico AP en lactantes y niños • Permitir una expansión torácica completa después de cada compresión. • Reducir al mínimo las interrupciones de las compresiones torácicas. • Relación compresión-ventilación de 30:2 para un solo reanimador en niños y

lactantes

Page 8: Temas de examen Pediatria

SOPORTE VITAL AVANZADO PEDIÁTRICO

• Tratar la causa del paro, estabilizar al paciente, manejar drogas y desfibrilación. • Lo primero es intubar al paciente. Para los niños existen 2 tipos de hojas, las

rectas que se prefieren para los niños (RN y lactantes) ya en preescolares de comienza a utilizar la de Macintosh o curva

Page 9: Temas de examen Pediatria
Page 10: Temas de examen Pediatria

Para intentar desfibrilar a niños de entre 1 y 8 años de edad usando un DEA, el reanimador debe emplear un sistema de atenuación de la descarga para dosis pediátricas, si dispone de uno. Si el reanimador practica la RCP a un niño que ha sufrido un paro cardíaco y no dispone de un DEA con un sistema de atenuación de la descarga para dosis pediátricas, debe emplear un DEA estándar.

En lactantes (menores de 1 año) es preferible utilizar un desfibrilador manual. Si no se dispone de un desfibrilador manual, sería conveniente utilizar un DEA con un sistema de atenuación pediátrico. Si ninguno de ellos está disponible, puede utilizarse un DEA sin un sistema de atenuación de dosis.

Para la desfibrilación inicial se puede utilizar una dosis de 2 a 4 J/kg, pero para facilitar el entrenamiento, se puede probar con una dosis inicial de 2 J/kg. Para descargas posteriores, los niveles de energía deben ser de al menos 4 J/kg, e incluso se pueden contemplar niveles de energía más altos, pero sin exceder los 10 J/kg o la dosis máxima para un adulto (200J bifásico, 360J monofásico)

Como han aumentado los indicios de que la exposición a altos niveles de oxígeno puede ser peligrosa, se ha añadido la nueva recomendación de ajustar la dosis de oxígeno inspirado (cuando se cuente con el equipo apropiado), una vez recuperada la circulación espontánea, para mantener una saturación de oxihemoglobina arterial igual o superior al 94% pero inferior al 100%, y limitar así el riesgo de hiperoxemia.

DERIVACIÓN

Page 11: Temas de examen Pediatria

2. ASFIXIA PERINATAL

Aspectos destacados de las Guías de la American Heart Association de 2010 para para reanimación cardiopulmonar y atención cardiovascular de emergencia

Part 15: Neonatal Resuscitation: 2010 American Heart Association Guidelines for Cardiopulmonary Resuscitation and Emergency Cardiovascular Care. Circulation.

2010;122:S909-S919.

TEJERINA H., Asfixia neonatal. Rev Soc Bol Ped 2007; 46 (2): 145 - 50

La asfixia perinatal es la causa principal que condiciona la necesidad de reanimación cardiopulmonar del recién nacido (RN) en el momento del parto.

DEFINICIÓN

Asfixia se define como la falta de oxígeno (hipoxia) o la falta de perfusión (isquemia) en diversos órganos. Fisiopatológicamente se caracteriza por hipoxemia, retención de CO2 y acidosis metabólica. Según la Academia Americana de Pediatría (AAP) y la Academia Americana de Obstetricia y Ginecología (ACOG) existen criterios clínicos estrictos que incluyen sólo a los casos más graves, que son los que desarrollan secuelas invalidantes. Definiendo asfixia a los recién nacidos con:

1- Apgar inferior a 3 a los 5 minutos. 2- pH de cordón bajo 7,00. 3- Manifestaciones sistémicas de asfixia, incluyendo una Encefalopatía

Hipóxica Isquémica (EHI) moderada a severa.

PREVENCIÓN

El daño cerebral que produce la asfixia perinatal es un proceso evolutivo que puede ser tratado si se interviene precozmente, evitando la injuria cerebral secundaria. Preparación para la reanimación: 1. Anticiparse a la necesidad de reanimación. Procurar que los RN que

tengan mayor riesgo nazcan en centros especializados, ya que transferir al binomio madre-hijo al nivel terciario es la mejor estrategia para el traslado perinatal.

2. Preparados en caso de cesárea programada. Los lactantes sin factores de riesgo prenatal, nacidos por cesárea programada realizada con anestesia regional entre las semanas 37 y 39 de gestación, tienen una necesidad menor de intubación, pero una necesidad ligeramente mayor de ventilación con mascarilla, en comparación con los lactantes nacidos por parto vaginal normal.

3. Preparación adecuada de equipo y personal: personal entrenado en reanimación en cada parto, fuente de calor radiante encendida, equipo de reanimación disponible y en buenas condiciones.

FACTORES DE RIESGO

La mayoría de las causas de hipoxia perinatal se originan intrauterinamente, el 10% comienza antes del inicio del trabajo de parto y el 90% durante el parto y expulsivo.

Page 12: Temas de examen Pediatria

MANIFESTACIÓN CLÍNICA

Su principal expresión clínica usada como sinónimo es la encefalopatía hipóxico isquemica. Después de una fase inicial de 12 horas de la acción causal, se observan signos de disfunción neurológica, un neonato obnubilado o comatoso, con respiración periódica e irregular (reflejo de disfunción bihemisférica), hipotónico, con ausencia de reflejos, movimientos oculares erráticos y generalmente reflejos pupilares conservados.

MANEJO INICIAL

El paro cardíaco neonatal suele producirse por asfixia, de modo que se ha mantenido la secuencia de reanimación A-B-C con una relación compresión-ventilación de 3:1, excepto cuando la etiología es claramente cardíaca.

Una vez comenzada la administración de ventilación con presión positiva o de oxígeno adicional, deben evaluarse simultáneamente tres características clínicas: la frecuencia cardíaca, la frecuencia respiratoria y el estado de oxigenación. Para el estado de oxigenación, lo ideal es determinarlo con un pulsioxímetro, y no simplemente evaluando el color.

La aspiración inmediatamente posterior al nacimiento (incluida la realizada con un aspirador nasal) debe reservarse para los neonatos con obstrucción obvia de la respiración espontánea o que requieran una ventilación con presión positiva. No hay suficientes datos para recomendar un cambio de la actual práctica de hacer una aspiración endotraqueal a los neonatos faltos de vigor con líquido amniótico teñido de meconio.

Page 13: Temas de examen Pediatria

La relación compresión-ventilación recomendada sigue siendo 3:1. Si se sabe que el paro tiene una etiología cardíaca, debe considerarse utilizar una relación más alta (15:2).

Se recomienda proporcionar hipotermia terapéutica a los lactantes nacidos con 36 semanas o más de gestación con una encefalopatía hipóxico-isquémica de moderada a grave. La hipotermia terapéutica debe administrarse con protocolos claramente definidos similares a los utilizados en los ensayos clínicos publicados y en centros con capacidad para proporcionar un cuidado multidisciplinario y un seguimiento longitudinal.

En un recién nacido sin una frecuencia cardíaca detectable, que continúa siendo indetectable durante 10 minutos, es adecuado considerar la conveniencia de detener la reanimación.

Page 14: Temas de examen Pediatria

DERIVACIÓN

El transporte Neonatal (TN) es un eslabón de vital importancia, dado que forma parte integral de las acciones de salud orientadas a disminuir la morbimortalidad neonatal.

Comunicación

Elemento fundamental del TN. El centro referente debe realizar la solicitud de traslado (generalmente vía telefónica), en el centro de referencia se evalúa la factibilidad operativa, la disponibilidad de cupos, etc.

Equipamiento

Idealmente el equipo de TN debe mantener su equipamiento, medicamentos e insumos siempre listos para ser utilizados y deben ser de uso exclusivo.

Medio de transporte

Ambulancia: El transporte terrestre es el más utilizado para traslados cortos, es de menor costo, cuenta con buen espacio interior, generalmente no tiene inconvenientes de operar frente a malas condiciones climáticas, aunque tiene la desventaja de ser un medio lento, sujeto a las condiciones de tránsito. Debe estar acondicionada para trasladar adecuadamente con el máximo de seguridad una incubadora de traslado y el equipo humano de TN (2 a 3 miembros), con el respectivo equipamiento. Debe proveer además de una fuente de poder para los equipos eléctricos, oxígeno y aire comprimido en cantidad suficiente para un traslado de al menos 200 km.

Preparación para el traslado

• Recopilación de información • Recomendaciones para estabilización • Información a los padres y trámites administrativos • Confirmación de traslado y llamada al equipo de turno de traslado • Preparación del equipo y medicamentos para el traslado

Comunicación con centro receptor:

Debe mantenerse una comunicación frecuente con el médico de la UCI que recibirá al paciente, informando la condición y requerimientos del paciente, momento de salida y tiempo estimado de llegada, lo cual permite coordinar lo necesario para la recepción del RN en la UCI.

Durante el traslado

El RN se mantiene en constante evaluación clínica con adecuado monitoreo de su condición cardiorrespiratoria, hemodinámica y signos vitales, con el objetivo de procurar las mejores condiciones que la salud del paciente permita; muchas veces es necesario implementar o realizar ajuste de terapias, como aporte de volumen, aumento de drogas vasoactivas, administración de sedación, paralizantes, etc. Mientras se realiza el TN pueden presentarse emergencias, en ocasiones es aconsejable detener la ambulancia si es necesario intubar o reintubar al RN.

Page 15: Temas de examen Pediatria

Llegada a centro receptor

Una vez en el centro de destino, se entrega el paciente, se colabora en su estabilización, se completan los formularios de TN y se deja nota en la ficha del RN. Finalmente se procede a la revisión, reposición y reubicación del equipamiento utilizado.

REANIMACIÓN NEONATAL

Page 16: Temas de examen Pediatria

3. APNEA EN EL LACTANTE

Exposición “ALTE y muerte súbita”, realizada Dra Fierro y Dra Payahuala Díaz en

reunión clínica del Servicio de Pediatría, HHHA, abril 2013.

Apparent life – Threatening events: An UPTODATE, Pediatrics in Review 2012; 33;361.

MARIN M, ORDOÑEZ O, PALACIOS A. Manual de Urgencias de Pediatría, Hospital 12 de octubre, capitulo “Episodio aparentemente letal”, 2011.

J. López López 1 , V. Alonso Morales. Episodio aparentemente letal en el lactante: diagnóstico diferencial e indicaciones de monitorización domiciliaria, 2009

DEFINICIÓN

• Apnea: cese del flujo aéreo en las vías respiratorias. • Apnea patológica: pausa respiratoria prolongada de más de 20

segundos o de duración menor si se acompaña de cianosis, palidez, bradicardia y/o hipotonía (hipertonía a veces).

• Apnea central: cese del flujo aéreo y de los esfuerzos respiratorios

con ausencia de movimientos toraco-abdominales. Se da por disfunción del centro respiratorio o neuromuscular (ej: apnea del prematuro, trauma , drogas, enfermedades congénitas). Las enfermedades sistémicas suelen producir apnea central

• Apnea obstructiva: cese del flujo aéreo en presencia de movimientos

toraco-abdominales y por tanto con incremento de los esfuerzos respiratorios. Se da por obstrucción de vía aérea (ej: hipertrofia adenoamigdalina, cuerpo extraño, laringomalacia)

• Apnea mixta: apnea central seguida de movimientos respiratorios, sin

flujo aéreo. O apnea obstructiva con esfuerzo respiratorio en que se intercalan pausas centrales. Prácticamente siempre hay componente obstructivo. Ej: prematuro con congestión nasal

• Hipopnea: obstrucción parcial de las vías aéreas superiores durante el sueño,

con disminución del flujo aéreo, asociada en ocasiones a despertar transitorio e hipoxemia.

• Pausa: cese del flujo de aire por más de 3 segundos en menores de 6 meses y más de 5 segundos en mayores de un año

• ALTE: episodio agudo, consistente en combinación de apnea (central u obstructiva), cambios de coloración (palidez sobre todo, otras veces cianosis y a veces eritema) y cambios en el tono muscular (hipotonía casi siempre), que requieren intervención significativa por parte del observador, en un intento de restaurar la respiración normal del paciente, desde una sacudida vigorosa hasta la reanimación cardiopulmonar. El ALTE no es una enfermedad específica en sí misma, sino una forma de presentación clínica de diversos problemas o patologías.

Los episodios de apnea y de pausa respiratoria son eventos comunes que se presentan en recién nacidos y lactantes. Las pausas respiratorias pueden ser

Page 17: Temas de examen Pediatria

normales a diferentes edades, sin embargo, deben ser consideradas anormales si son mayores a 20 segundos de duración o si producen hipoxemia o cianosis asociada. Una parte significativa de la susceptibilidad de los lactantes menores a presentar apnea y pausas respiratorias, se relaciona al desarrollo del control de la respiración que ocurre dentro de los primeros meses de la vida. La mayoría de los pacientes con inmadurez del centro respiratorio normalizan sus parámetros al año de vida No se ha podido establecer la relación entre un episodio de ALTE y la probabilidad de repetir el episodio y presentar una muerte súbita

ALTE: Apparent Life-Threatening Event Episodio atemorizante, que ante los ojos del observador pone en riesgo la

vida de un lactante y que se caracterizan por:

1. Apnea • Cese del flujo respiratorio por más de 20 segundos ó • Pausa respiratoria acompañada de bradicardia, cianosis,

palidez y/o hipotonía.

2. Cambio de color • Palidez, Cianosis, Rubicundez. • La cianosis puede ser central (Sat.O2<85%) o periférica

3. Alteraciones del tono muscular

• Resistencia al movimiento pasivo de una parte del cuerpo

• Se presentan como hipotonía, hipertonía, que puede deberse a defectos centrales o reacciones vasovagales.

4. “Atoro” o sofocación.

• Mecanismo de defensa, frente a estímulos en faringe, laringe o vía aérea baja.

• Puede ser una manifestación normal o producto de patologías neurológicas, digestivas o respiratorias.

Generalidades

• La mayoría de los pacientes son sexo masculino (58%)

• En un 30% los pacientes son prematuros • La mayoría de los pacientes se encuentran entre los 0 y 5 meses con alta

incidencia a los 2 meses. • La principal etiología es respiratoria. En un primera instancia, el RGE

correspondia al 50%, ahora ha disminuido hasta menos 5%. Y en la actualidad está descrita como una de las tres primeras causas a estudiar.

Factores de riesgo:

• Recurrencia • Durante sueño • Prematuros

• Familares con SMSL o ALTE • Factores riesgo social • Drogadicción materna, madre VIH, Tabaco.

Clasificación

• ALTE MENOR: Episodio que revierte espontáneamente o a través de una estimulación leve.

• ALTE MAYOR Episodio que revierte a través de una estimulación vigorosa o reanimación cardiopulmonar

Page 18: Temas de examen Pediatria

ETIOLOGÍA

ESTUDIO CHECKLIST

BUENA HISTORIA

• Lugar dónde ocurrió: casa, auto, cuna, sofá, cama de los padres. • ¿Quiénes estaban presentes? • En vigilia o sueño, posición, hora en que ocurrió • Cambio de color en cara y/o cuerpo, postura (hiper o hipotonía),

movimientos anormales, desviación de mirada. • Tiempo de duración el episodio. • ¿Qué medidas se realizaron para abortar el evento? • Tuvo relación con la comida, llanto, posición, etc. • Esfuerzo respiratorio, tos, vómito, leche o restos alimentarios durante o

después. • Como era el estado de conciencia luego del evento. • Precedido de: fiebre, alguna enfermedad, inmunizaciones,

medicamentos. Antecedentes.

RGE y ALTE

• La Asociación causa efecto es controvertida. Si bien el diagnóstico de RGE es clínico, debieran realizarse estudios que demuestren relación temporal (polisomnografía y phmetría).

Gastrointestinales RGE / Infecciones / Invaginación intestinal /

Fístulas TE / Patologías estructurales.

Neurológicas Trastornos convulsivos:

HIC.Hidrocefalia.RDS.Hipoxia. /Infecciones /Hipertensión Endocranena/Malformaciones

Respiratorias

Infecciones: VRS, Bordetella, Micoplasma/ Anomalías Congenitas: Quiste de la vía aerea,

Angiomas./Obstrucción: Vómitos, Cuerpo extraños.

Cardiovasculares Cardiopatías congénitas /Arritmias

/Miocardiopatías /Anormalidades coronarias.

Metabólicas, endocrino Hipoglicemia, Desordenes electrolíticos

Abuso infantil Síndrome de malos tratos: Munchausen por

poderes. Niño sacudido. / Intoxicaciones: intencionales: Drogas de abuso. Medicamentos.

Otros Errores en alimentación, Efectos adversos de

medicamentos, Sofocación accidental, Asfixia en la cuna.

Idiopático hasta 50%

Page 19: Temas de examen Pediatria

DIAGNÓSTICO DIFERENCIAL

La Anamnesis debe ser recogida de la persona que presenció el episodio. Con una correcta anamnesis podremos diferenciar si el episodio ha sido realmente un ALTE o se ha confundido con otros tipos de comportamientos considerados normales, como son: • Respiración periódica: patrón respiratorio con 3 o más pausas de más de 3

segundos de duración con menos de 20 segundos de respiración normal entre las pausas.

• Respiración irregular en la fase REM del sueño. • Apneas de 5-15 segundos de duración, o de mayor duración si la apnea es tras

suspiro. • Atragantamiento, tos o náuseas transitorias durante la toma.

TRATAMIENTO

Inicialmente se procederá a descartar la existencia de situaciones de compromiso vital, proporcionando el apoyo cardiorrespiratorio necesario.

Monitoreo domiciliario

• Ningún tipo de monitor ha demostrado lograr disminuir la frecuencia de ALTE ni SMSL.

• Los monitores CP NO deben ser indicados para PREVENIR un SMSL. • Pueden ayudar a detectar eventos posiblemente peligrosos.

DERIVACIÓN

A centro de mayor complejidad: Todo ALTE verdadero

Se recomienda hospitalizar a la mayoría de los pacientes que consulten por ALTE, por un mínimo de 23 horas para observación y monitorización continua.

Si es primer episodio de ALTE, sin antecedentes de RNPT, sin antecedentes mórbidos, estable, episodio breve y autolimitado, podría darse de alta antes de 23 horas.

Estudio básico • Hemograma con

frotis • GSA • ELP • GLICEMIA • EX. ORINA • EKG • VRS-BORDETELLA • RX TX

Page 20: Temas de examen Pediatria

ENFRENTAMIENTO ALTE

Consideraciones sobre Muerte Súbita del Lactante

• Definición: Muerte inesperada de un lactante < de 1 año, con historia, examen físico y evaluación post-mortem, que no revele una causa conocida.

• Prevención • Promover la posición supina al dormir (“back to sleep”) • Sugerir que los lactantes duerman en su cuna y no con los padres en

la misma cama. • Evitar que los lactantes duerman sobre superficies blandas. • Evitar elementos blandos como peluches, cobertores y similares en

la cuna del lactante. • Suspender el tabaquismo. • Considerar el uso de chupete al dormir, pero no forzar su uso una vez

que el niño lo suelte.

Estudio ALTE - Hospitalizar

Historia y examen físico

detallado ¿sugerente de causa?

Cardiológico (soplos, cianosis)

EKG, ECO cardio

Neurólogo Evaluar por neurólogo,

EEG, TAC.

Sospecha de IRA IFD virales, IFD Bordetela

¿Ronquido?

¿sospecha apnea obstructiva?

Evaluar por neumologo, PSG

¿al alimentarse?

Sospecha RGE PHmetria

¿reflujo eyecto-lácteo?

Revisar historia

Sospecha trastorno deglución

video-deglución

Monitorización por 48 hrs

¿repite episodio?

No ¿causa identificable

reversible?'

No ALTA con monitor, PSG, EKG, estudio

metabólico.

Sí ALTA sin monitor,

Tratar la causa pesquisada.

Sí ALTA con monitor, PSG, EKG, estudio

metabólico • Educar sobre las condiciones

ambientales y cuidado del lactante.

• Educar sobre SMSL. • Curso de RCP a

padres/cuidadores. • Control seriado.

Page 21: Temas de examen Pediatria

4. SÍNDROME DISTRÉS RESPIRATORIO EN EL RN

Apunte de clase “Síndrome de dificultad respiratoria neonatal”, realizada por Dra. Velásquez, UFRO 2010.

Manual CTO Pediatría, 8va edición

DEFINICIÓN

• Conjunto de síntomas y signos que traducen un incremento en el trabajo respiratorio del neonato y que obedece a múltiples etiologías.

Manifestaciones Clínicas

• TAQUIPNEA (aumento de la FR , pero superficial) • POLIPNEA (aumento de la FR, en el RN debe ser mayor a 60m para hablar de

polipnea) • RETRACCIÓN DE PARTES BLANDAS (supraesternal, supraclavicular, retracción

esternal, intercostal, subcostal) • ALETEO NASAL (apertura de las narinas) • QUEJIDO ESPIRATORIO (se cierra la glotis y se quejan) • APNEA (no olvidar q los RN tienen una respiración irregular, pueden dejar de

respirar hasta 19 seg y es normal, pero si es sobre esto, va a ser apnea) • CIANOSIS (pueden o no tenerla, pueden o no requerir O2) • El niño puede tener uno o varios de estos síntomas e igual clasifica para SDR. • RUIDOS RESPIRATORIOS (generalmente no hay sibilancias en el RN, si crepitos,

estertores finos) • ALTERACIÓN MURMULLO VESICULAR (generalmente es solo disminución del

murmullo, no broncofonía como en el niño mayor) • SOPLOS CARDIACOS • ALTERACIONES DEL TONO CARDÍACO

DIAGNÓSTICO DIFERENCIAL

• Obstrucción de la vía aérea • Taquipnea transitoria del RN. • Enfermedad de membrana hialina. • Síndrome de aspiración meconial. • Extravasación extrapulmonar de aire. • Persistencia de la circulación fetal. • Infecciones: neumonía, sepsis, etc. • Enfermedad pulmonar crónica: displasia bronco pulmona r . • Malformación diafragmática, laringomalacia, síndrome de Pierre-Robin, etc.

MANEJO INICIAL

(no olvidar que es independiente de la causa!!) • Régimen 0 • SOG (sonda oro-gástrica porque los niños no respiran por la boca, si le tapo la

nariz con una SNG, aumento su apremio respiratorio. Se usa como vía de aspiración para desocupar el estomago tanto de aire, como de liquido que tenga, para con esto disminuir el esfuerzo respiratorio. La sonda queda abierta)

Page 22: Temas de examen Pediatria

• Posición, ATN (ambiente térmico neutro), ya que los RN no son capaces de regular la T° por si mismos. Esto dependerá si son prematuros o de término. Podemos usar por ejemplo una cuna de calor radiante. Los RN son muy sensibles, especialmente a la posición del cuello, si quedan doblados, hacen apnea, por lo q la posición debe ser de olfateo.

• Oxígeno, primero si se puede hacer una saturometria de pulso, para poder aportar el O2 necesario.

• Fleboclisis con glucosa 10% (no electrolitos, que se incorporan al 3° día de vida, solo administrar glucosa a un volumen de 60ml/kg)

• Exámenes de laboratorio: • Hemograma (buscar policitemia, anemia, desviación izquierda, o

puede no decirme nada), • PCR (si esta alta: infección), • Rx. Tórax, • Gases (para evaluar necesidad de VM), • Dextro (por la hipoglicemia), • También podríamos necesitar hemocultivos (pero estos no nos

corresponden, todos se derivan, pudieran tratarse en un hospital terciario cuando requieren menos de 40% de O2, siempre y cuando no requiera hacer estudios que no se dispongan.)

DBP: Displasia broncopulmonar

Page 23: Temas de examen Pediatria

• SAM: Síndrome de Aspiración Meconial • PCF: Persistencia de la circulación fetal (hipertensión pulmonar persistente del

RN) • Enfermedad de la membrana hialina (EMH)

DERIVACIÓN

Page 24: Temas de examen Pediatria

5. ICTERICIA NEONATAL

AAP. Management of hyperbilirrubinemia in the newborn infant 35 or more weeks of gestation. Pediatrics 2004; 114:297-316.

MINSAL. Hiperbilirrubinemia, Guias Nacionales de Neonatología, 2005.

“Ictericia Neonatal”, Dra. María Isabel Toledo y Dr. César Gutiérrez, Manual de Apuntes Pediatría y Cirugía Infantil, Universidad de la Frontera.

DEFINICIÓN

Entidad clínica caracterizada por aparición de ictericia en piel y mucosas provocada por aumento de bilirrubina sérica sobre los niveles considerados normales para la edad y peso del neonato. En los neonatos se requieren valores>5mg/dl de bilirrubina para ser detectada la hiperbilirrubinemia como ictericia.

FACTORES DE RIESGOS

Alto riesgo Riesgo medio Bajo riesgo

* Bili T en zona de alto riesgo * Ictericia en <24 hrs vida * Enf. Hemolítica * EG: 35 – 36 semanas * Hermano anterior que

requirió fototerapia * Cefalohematoma * LM inadecuada o pérdida

de peso excesiva * Raza asiática materna

* Bili T en zona riesgo intermedio alto

* Ictericia post alta * EG: 37–38 sem. * Hermano previo

con ictericia * RN macrosómico o

HMD * Madre >25 años * Sexo masculino

* Bili T en zona de bajo riesgo

* EG: >39 sem * Lactancia artificial

exclusiva * Raza negra materna * Alta del hospital

después de las 72 hrs.

CLASIFICACIÓN

SEGÚN EDAD DE PRESENTACIÓN

Precoz (primeras 24 hrs)

Normal (2° al 7° día) Tardía (después de la primera

semana)

Enfermedad hemolítica (Rh, ABO, grupos menores)

Microesferocitosis familiar congénita

Muy común

Ictericia fisiológica Frecuentes:

Enf.hemolítica (Rh, ABO, grupos menores)

Poliglobulia

Reabsorción de hematomas y cefalohematomas

Prematuridad

Sepsis * Menos frecuente:

Obstrucción intestinal

Enfermedad metabólica del metabolismo de la bilirrubina: Gilbert, Criggler-Najjar tipo I yII

Frecuentes

Ictericia por leche materna

Ictericia por hipoalimentación

Menos frecuente

Alteraciones hepatobiliares

Atresia de vías biliares*

Hepatitis neonatañ*

Ictericia por alimentación parenteral

TORCH *

Hipotiroidismo

Galactosemia*

• *Ictericia de predominio Directo

Page 25: Temas de examen Pediatria

SEGÚN FISIOPATOLOGÍA

ICTERICIA POR LECHE MATERNA

PRECOZ (O ICTERICIA POR HIPOALIMENTACIÓN)

Los RN alimentados exclusivamente por leche materna experimentan la máxima pérdida de peso al 3° día de vida, aproximadamente un 6,1% (DE 2,5%) de su peso de nacimiento. A los RN que tienen un descenso de peso >10% se le debe hacer una evaluación dirigida a evaluar la técnica de lactancia. Posibles causas: • Insuficiente leche materna (hipogalactia). • RN con problemas neurológicos. • RN con nariz obstruida o sin fuerza. • Deformidades. • Falta de estímulo a la lactancia materna. Mecanismo: Probablemente se deba a un aumento del círculo enterohepático de la bilirrubina por disminución de la motilidad intestinal, a lo que se agrega un cierto grado de deshidratación.

DE INICIO TARDIO (O ICTERICIA POR LECHE MATERNA)

Ictericia asintomática de inicio tardío entre el 4º-7º día, que puede prolongarse hasta la 4ª-12ª semana de vida.

•Enfermedad Hemolitica

•Anomalias Morfológicas del GR.

•Policitemia

•Extravasación sanguinea (Cefalohematoma)

•Circulación entero hepática exagerada (obstrucción o ileo gastrointestinal; sangre deglutida durante el parto; ayuno.).

Aumento de la producción de

bilirrubina

•Ausencia congénita de Glucoronil transferasa (S. Crigler – Najjar I).

•Inhibición de la Glucoronil transferasa (fármacos, S. De Lucey – Driscoll).

Defectos de la conjugación

•Hipoalimentación

•Presencia de betaglucoronidasa en la leche materna, que desconjuga.

•Otros.

Ictericia por leche materna

•Galactosemia,

•Hipotiroidismo,

•Diabetes materna,

•Secundaria a sepsis.

Trastornos metabólicos

•Atresia vías biliares,

•quiste del colédoco,

•enf. hepatocelular (TORCH, enf.mtb).

Ictericia Colestásica

•No cumple los requisitos de ictericia fisiológica y no se determina con seguridad otra etiología

Idiopática

Page 26: Temas de examen Pediatria

El diagnóstico es clínico tras la exclusión de otras causas.

Se postulan varias causas: • se postula el rol de la betaglucuronidasa en la leche materna, que actúa sobre la

bil. conjugada en el intestino liberando bilirrubina no conjugada y aumentando la circulación enterohepática.

ICTERICIAS HEMOLÍTICAS

POR INCOMPATIBLIDAD SANGUÍNEA (REACCIÓN ISOINMUNE)

Incompatibilidad grupo clásico Incompatibilidad RH

Se produce en madre OIV que sea capaz de producir anticuerpos del tipo IgG que atraviese la placenta.

Test de Coombs tiene baja sensibilidad

De los 35 antígenos Rh sólo seis tienen importancia clínica. El más importante es el gen D, que confiere el ser Rh (+), y es responsable del 97 % de la Enfermedad hemolitica por Rh. Una persona Rh (+) puede ser homocigota DD o heterocigota Dd,

Hallazgos

Anemia: que puede ser precoz o tardía

Hepatoesplenomegalia

Disminución de la hemoglobina (< 14 mg/dL)

Reticulocitosis: > 6 %,

Aumento de los glóbulos rojos nucleados. (Eritroblastosis)

Hipoglicemia producida por hiperplasia de los islotes pancreáticos debido a aminoácidos liberados por la hemólisis.

Test de Coombs positivo, pero tiene baja sensibilidad. Factores que sugieren la posibilidad de una enfermedad hemolítica: • Historia familiar de enfermedad hemolítica importante. • Comienzo de la ictericia antes de las 24 horas de edad. • Aumento de la bilirrubina sérica > 0,5 mg/dl/hora • Palidez, hepatoesplenomegalia • Rápido aumento de la bilirrubina sérica total después de 24 – 28 hrs. • Sugerencia étnica de enf. hereditaria (por ej: deficiencia de G-6-P D, etc.) • Fracaso de la fototerapia para disminuir bilirrubina sérica total.

DIAGNÓSTICO

CRITERIOS PARA DESCARTAR ICTERICIA FISIOLÓGICA:

Ictericia clínica evidente en las primeras 24 horas de vida.

Aumento de la bilirrubina total > 5 mg/dl por día.

Bilirrubina total sérica > 14 mg/dl en un RNT o de 10 mg/dl en un prematuro.

Bilirrubina directa sérica > 2 mg/dl.

Ictericia clínica persistente >1 sem en RNT o > 2 semanas en RNPT

Page 27: Temas de examen Pediatria

TRATAMIENTO

Precoz (o ictericia Por Hipoalimentación)

• Reforzar lactancia y suplementar en los casos de hipogalactia.

De inicio tardio (o ictericia por leche materna)

• APP aconseja no interrumpir la LME en niños de término sanos y a su vez, recomienda aumentar la frecuencia (10 a 12 veces en 24 hrs)

• Dr. Rudolph aconseja suspender lactancia materna por 24 horas y luego reiniciarla.

Fototerapia

Indicaciones:

Valor de BilT sobre la curva de indicación de fototerapia que le corresponde al RN según los factores de riesgo que presente

• Factores de riesgo: Edad gestacional al nacer, Enfermedad hemolítica, deficiencia G6OD, asfixia, letargia significativa, inestabilidad térmica, sepsis, acidosis, albumina sérica <3,0g/dl.

Si los niveles de BilT no descienden o continúan aumentando a pesar de la fototerapia, sugiere que está ocurriendo hemolisis.

Exsanguineo-transfusión

Debe ser realizada sólo por personal especializado, en unidades de cuidados intensivos, con monitorización continua y equipos de reanimación con personal entrenado. Indicaciones Guideliness

Valor de BilT sobre la curva de indicación de exsanguineotranfusión que le corresponde al RN según los factores de riesgo que presente

• Factores de riesgo: Edad gestacional al nacer, Enfermedad hemolítica, deficiencia G6OD, asfixia, letargia significativa, inestabilidad térmica, sepsis, acidosis.

Se hace en general un recambio con el doble del volumen sanguíneo, esto es ‡ 2 x 80 ml//kg = 160 ml/kg.

El recambio debiera de tomar como mínimo 1 hora y como máximo un y media a 2 horas.

Page 28: Temas de examen Pediatria

Gamaglobulina

Indicaciones

Enfermedad Hemolítica isoinmune • Si los niveles de BilT no descienden o continúan aumentando a pesar

de la fototerapia, sugiere que está ocurriendo hemolisis.

El uso precoz de gammaglobulina permite reducir la necesidad de fototerapia y exsanguineotransfusión por hiperbilirrubinemia.

LOS 10 PUNTOS CLAVES

1. Promover y apoyar la lactancia materna exitosa. 2. Establecer protocolos para identificar y evaluar la hiperbilirrubinemia 3. Medir BilT, o nivel de Bilirrubina transcutanea en RN que presentan ictericia

durante las primeras 24 horas. 4. La estimación visual del grado de ictericia puede llevar a errores en RN con

pieles oscuras. 5. Interpretar los valores de Bilirrubina acorde a la edad del RN en horas. 6. RN <38 sem. EG, especialmente los alimentados por pecho materno, tienen

un riesgo alto de desarrollar hiperbilirrubinemia, por lo que requieren una vigilancia estricta y monitoreo.

7. Previo al alta, educar a todos las madres de RN, sobre el riesgo de que desarrollen una hiperbilirrubinemia severa.

8. Entregar a los padres información tanto escrita como verbal sobre los RN ictéricos.

9. Realizar un seguimiento apropiado acorde al momento del alta y la evaluación de los riesgos.

Momento del alta Control médico

<24 horas de vida 72 horas de vida

24-48 horas de vida 96 horas de vida

48-72 horas de vida 120 horas de vida

10. Tratar a los RN cuando este indicado, ya sea con fototerapia o ex sangineo transfusión.

Page 29: Temas de examen Pediatria

6. ICC

CAUSAS

DIAGNÓSTICO

MANEJO INICIAL

DERIVACIÓN

Page 30: Temas de examen Pediatria

7. CARDIOPATÍAS CONGÉNITAS

CLASIFICACIÓN

RESEÑA DE LAS MÁS FRECUENTES

CIV

• Es la CC más frecuente. • Defecto en el septo interventricular que permita la comunicación entre ambos

ventrículos. • Las manifestaciones clínicas dependerán de la localización y el tamaño de la CIV.

Si el defecto es grande, provoca un hiperflujo pulmonar, con la consiguiente ICC, y Rx de tórax, habrá cardiomegalia con hiperflujo pulmonar.

• CIV pequeñas o restrictivas: se suelen diagnosticar por la presencia de un soplo sistólico los primeros días de vida, generalmente de 2-3/6 en el BPEI, sin otros síntomas. Aproximadamente un 35% presenta un cierre espontáneo en los primeros dos años de vida.

• Existe una estenosis relativa de la arteria pulmonar. • Según su localización pueden ser: membranosa, del septo de salida, muscular o

del septo de entrada.

Page 31: Temas de examen Pediatria

• La CIV membranosa (o paramembranosa o subaórtica) es la más frecuente de las CIV (75%) y se localiza debajo de la válvula aórtica y detrás de la valva septal de la tricúspide.

• Deben quedar con indicación de profilxaxis para endocarditis en casos necesarios (intervención dental por ejemplo)

Soplo:

• Holosistólico, Foco Mitral. Entre más intenso el soplo, menos grave es.

CIA

Defecto en el tabique interauricular

Las consecuencias hemodinámicas dependerán de la localización y tamaño del defecto y de la asociación con otros defectos.

Se produce una sobrecarga de cavidades derechas.

CIA ostium secundum: corresponde al 70% de todas las CIA, es más frecuente en mujeres y hay una recurrencia familiar del 7-10%. Se localiza en la región de la fosa oval.

La CIA aislada no suele dar síntomas en la infancia y el diagnóstico suele hacerse por la presencia de un soplo o de un desdoblamiento fijo del 2R cardiaco.

Puede haber un cierre espontáneo de las CIA si son inferiores a 8 mm, con una tasa de cierre que puede llegar al 70-80% antes de los 18 meses.

Si se deja una CIA a su libre evolución podría dar lugar a HTP y taquiarritmias (fibrilación auricular) a partir de la 3ª década.

Soplo:

• Mesosistólico con desdoblamiento fijo del 2º R. Foco pulmonar

EKG

• Suele haber un patrón de rsR’ en V1 (bloqueo incompleto de rama derecha), que suele ser más bien la expresión de una hipertrofia del VD, más que de un trastorno de conducción

DAP

• El ductus conecta el tronco de la arteria pulmonar con la aorta descendente por debajo de la arteria subclavia izquierda.

• Se produce una sobrecarga de cavidades izquierda. • En el período postnatal, ocurre el cierre del ductus, primero de forma funcional,

con oclusión de la luz por protrusión de la íntima y, posteriormente, el cierre definitivo, que suele ocurrir entre los 20-30 días de vida, que da lugar al ligamento arterioso.

• Entre los factores que determinan el cierre postnatal, están el aumento de la PO 2 y la disminución de las PGE2.

• En un niño normal, el ductus se cierra funcionalmente a los 3 – 4 días. • Ductus grande (> 6 mm) presentan clínica de IC izquierda consecuencia de la

sobrecarga de cavidades izquierdas y del edema pulmonar, que condiciona: taquipnea, tiraje, dificultades de alimentación e infecciones respiratorias frecuentes.

Page 32: Temas de examen Pediatria

Soplo

• Continuo, en maquinaria. Foco Pulmonar, además en región infraclavicular izquierda

Pulsos Saltones

Coartación Aórtica

• Es un estrechamiento de la Aorta torácica distal a la arteria subclavia izquierda, aunque en algunos casos también puede ser proximal a ella. También en ocasiones, se acompaña de un segmento hipoplásico.

• Es importante la palpación del pulso femoral al mismo tiempo que el axilar, para poder detectar precozmente una Coartación Aórtica neonatal.

No da soplo

Tetralogía De Fallot:

• Está compuesta por: 1. Estenosis pulmonar y subpulmonar 2. CIV 3. Cabalgamiento de Aorta sobre CIV 4. HVD

• Es Pentalogía de Fallot cuando hay: 5. CIA

• Puede ser rosada o cianótica, según el grado de estenosis pulmonar. Esto además determina la forma del corazón.

• Son frecuentes las anomalías coronarias.

Soplo

• Todo Fallot tiene un soplo, ya sea pulmonar o de CIV.

ECG:

• Se detecta una HVD, con R prominentes y T+ en V1.

RX de tórax:

• La forma clásica del corazón es la de forma de “zueco” por una zona cóncava a nivel del tronco de la arteria pulmonar. No hay cardiomegalia y el flujo pulmonar es normal o disminuido.

SOSPECHA DIAGNÓSTICA

Síntomas Signos

Disnea

Sudoración

Crisis Anoxémica

Mareos

Síncopes

Precordalgia

Palpitaciones

Cianosis

Soplo

Arritmias

Alteración pulsos

Cardiomegalia

Manifestaciones ICC

Retardo pondoestatural

Page 33: Temas de examen Pediatria

MANEJO INICIAL

Si satura<80% se le administra Prostaglandina y no oxígeno.

En todo RN con CCC está autorizado el uso de PGE1 (Prostaglandina E), ante la eventualidad de presentar Cardiopatía Ductus Dependiente.

• Presentación: ampollas de 1 ml., que contiene 500 g. • Dilución: suero glucosado al 5% de preferencia. • Vía de administración: EV por vía periférica o central (BIC).

• Dosis: se recomienda iniciar con 0.02 g/kg/minuto; si no hay respuesta al cabo de 30 a 40 minutos se puede aumentar al 0.03; de haber resultado, se recomienda disminuir a 0.015 y luego a 0.01, dejando esta última de mantención. Cálculo: 30 mg/kg en 50 cc SG 5% a pasar 1

cc/hrs = 0.02 g/kg/minuto. • Almacenamiento: debe almacenarse en frío entre 4-8 grados.

TRATAMIENTO DE LA CRISIS Portadores de Tetralogía de Fallot

Acuclillamiento ( RV y RVP).

O2 : tendría escaso valor, ya que existe disminución del flujo sanguíneo pulmonar y no de la oxigenación.

Morfina: 0,1 - 0,2 mg/kg, SC - IM (anula Centro Respiratorio e hipernea)

Corregir acidosis: HCO3 Na 1 meq-kg-EV ( acidosis, lo cual inhibe CR).

Fenilefrina: EV 0.01-0.04 mg/kg/dosis (vasoconstricción RVS y FP).

Propanolol: 0.1 mg/kg EV.

Corregir anemia. • Superada la crisis el paciente debe mantenerse con tratamiento profiláctico con

Propanolol 2-4 mg/kg/día oral (estabilización de reactividad vascular periférica y

riesgo espasmo TSVD). • Niño en crisis hipoxémica requiere tratamiento médico urgente en el hospital

más cercano. Derivar a centro de mayor complejidad una vez estabilizado. • Crisis hipoxémica es indicador de tratamiento quirúrgico paliativo o corrector

urgente. Se debe derivar a centro de referencia cardioquirúrgico a la brevedad.

TRATAMIENTO DE LA CRISIS Portadores de CCC Ductus Dependiente

No dar oxígeno, o hacerlo con precaución.

PGE1 : Dosis 0,02 - 0,1 g/kg/min.EV en goteo continuo con bomba de infusión. Regular dosis a la mínima necesaria para lograr adecuada saturación O2. Estar atento a la necesidad de usar V.M.

Se presenta la crisis de cianosis ante el cierre progresivo del DAP (Ductus), del cual depende la circulación pulmonar (Estenosis Pulmonar crítica, Atresia Pulmonar, TGA, Atresia Tricúspidea).

DERIVACIÓN

CONDICIONES DE TRASLADO

• Termorregulación: Incubadora de transporte. • Oxigenoterapia: Ventilación mecánica, mascarilla. • Hidratación: Usar bombas de infusión. • Medicamentos: PEG1, drogas vasoactivas. • Considerar: posibilidad de reanimación.

Page 34: Temas de examen Pediatria

8. SÍNDROME CONVULSIVO

Apuntes clase “Síndrome Convulsivo” realizada por Dr.Gianni Rivera, año 2010

MARIN M, ORDOÑEZ O, PALACIOS A. Manual de Urgencias de Pediatría, Hospital 12 de octubre, capitulo “Convulsiones y estatus convulsivo”, 2011.

Dr. Gianni Rivera. Protocolos de Epilepsia en el niño, HHHA, Junio 2011

DEFINICIÓN

• Convulsión: cuadro clínico de comienzo brusco caracterizado por contracciones musculares anormales, generalizadas o localizadas, tónicas, clónicas o tónico-clónicas acompañadas, en ocasiones, de alteración del nivel de conciencia. Pueden ser de naturaleza epiléptica o no.

• Crisis epiléptica o comicial: Manifestación clínica que es resultado de una descarga anormal y excesiva de un grupo de neuronas. La manifestación clínica consiste en un fenómeno anormal, súbito y transitorio, el cual puede incluir alteraciones de conciencia, motrices, sensoriales, autonómicas o psíquicas, las que son percibidas por el paciente o un observador.

• Epilepsia: Afección crónica, de etiología diversa, caracterizada por crisis recurrentes debidas a una descarga excesiva de las neuronas cerebrales en forma paroxística.

• Estatus epiléptico: Cualquier tipo de convulsión que dure más de 20 minutos o que recurra con tal frecuencia que el paciente no logre recuperar la conciencia entre ellas. Es una situación de emergencia que debe ser tratada adecuada 20% de los niños con epilepsia sufren al menos un episodio de Status Convulsivo. Las dos situaciones más frecuentes se refieren a:

• Niño en tratamiento por epilepsia y que puede haber suspendido la medicación.

• Niño previamente sano que debuta con un status febril, en el que debe descartarse una infección del sistema nervioso central.

ETIOLOGÍA POR EDADES

NEONATALES LACTANTES – PRE-

ESCOLAR ESCOLAR Y

ADOLESCENTE

• Asfixia (EHI) • Hemorragia intracraneana • Alteraciones metabólicas:

Hipocalcemia , Hipomagnesemia , Hipoglicemia , Hiponatremia o hipernatremia

• Infecciones: Meningitis • Malformaciones congénitas • Errores del metabolismo • Abstinencia de drogas • Infecciones congénitas

(TORCH)

• Convulsiones febriles • Enfermedades

crónicas neonatales • Infecciones:

meningitis, encefalitis

• Traumatismos • Epilepsia (Sd. West) • Neoplasias • Enfermedades

degenerativas

• Idiopáticas Epilepsia

• Infecciones: meningitis, encefalitis

• Traumatismos • Intoxicaciones • Tumores • Enfermedades

degenerativas • Genetopatías

Las crisis tónico-clónicas son el tipo de crisis más frecuente en los trastornos metabólicos

Page 35: Temas de examen Pediatria

CLASIFICACIÓN

DIAGNÓSTICO

Convulsión Febril

• Definición (“Consensus development panel. Febrile seizures”. Pediatrics 1980) 1. Episodio convulsivo asociado a fiebre 2. Niño entre 6 meses y 5 años 3. Sin signos de infección SNC o enfermedad neurológica aguda 4. Sin antecedentes de convulsiones afebriles previas

• Se excluyen las convulsiones con fiebre en niños que hayan tenido una convulsión afebril previa.

• No existe un límite claro de la fiebre por debajo del cual se deba dudar de este diagnóstico, pero es poco frecuente con T°<38-38,4°C)

• Las infecciones virales son las causas predominantes de las CF, observándose un claro patrón estacional en su incidencia (Herpes simplex tipo 6, Influenza A, entero- virus, adenovirus y Shigella)

• También se ha descrito un ritmo circadiano, siendo más frecuentes entre las 6 y las 12 de la noche.

Simples: Complejas

1. Generalizadas, tónico-clónica, no focal.

2. Duración crisis: <15 minutos 3. No repite en las sgtes. 24 hrs. 4. Recuperación completa en el

lapso de una hora.

1. Edad <6 meses o > de 6 años. 2. Focales 3. Duración crisis: >10 - 15 minutos 4. Crisis repetidas en las sgtes. 24

hrs.

Segú

n S

emio

logí

a.

I. CRISIS PARCIALES: sólo se activa una

región concreta del cortex

Simples Dependiendo del área afectada

provoca síntomas motores, sensitivos, sensoriales o

psíquicos

Complejas

Producen alteración del nivel de conciencia y automatismo, con

periodo de confusión tras la crisis

II. CRISIS GENERALIZADAS:

se produce una actividad eléctrica

simultanea en ambos hemisferios,

activandose todo el cortex.

Ausencias (pequeño mal)

Ausencia atípica.

Mioclónicas (único o múltiple).

Clónicas.

Tónicas.

Tónico-clónicas (grand mal).

Atónicas (astática). III - NO CLASIFICABLES

Page 36: Temas de examen Pediatria

• Por ser una patología benigna y considerando los efectos secundarios de los Fármacos Anti Epilépticos (FAE), no está indicado el tratamiento permanente con antiepilépticos, aun cuando el niño haya presentado episodios en otras oportunidades.

• El riesgo de recurrencia frente a una crisis febril simple se estima en 30%, lo que aumenta a un 80% después de la segunda crisis, disminuyendo con la edad, siendo muy infrecuentes después de los 4 años de edad.

Epilepsia Ausencia de la niñez “Petit mal”

• Epilepsia generalizada idiopática. Representa aproximadamente el 8% de las epilepsias de presentación en la niñez. Excelente pronóstico.

• Edad de comienzo: 3-12 años (máx. 5-7 años). • Características de las crisis

• “desconexiones” breves de 5 a 25 seg., muy frecuentes en el día. • Pueden ser puras o ir acompañadas. • La Hiperventilación forzada es un fuerte inductor de crisis.

• Diagnóstico diferencial: ausencias atípicas del SLG, epilepsia del lóbulo temporal. • Tratamiento: Ac.. Valproico

Epilepsia Tipo Gran Mal

• Alcanza alrededor del 30% de las epilepsias en niños prepuberes y adolescentes. • Crisis Típicas TCG: pérdida de conciencia, fase tónica, contracciónes clónicas.

Sueño y confusión postictal. Ocurren frecuentemente al despertar o al final del atardecer durante el período de relajación. Puede tener relajación de esfínteres o mordedura de lengua

• Diagnóstico diferencial: otras epilepsias.

DIAGNÓSTICO DIFERENCIAL

Eventos paroxísticos pueden ser confundidos con crisis epilépticas, tales como:

Causales de compromiso de conciencia:

Causales de movimientos anormales:

• Síncope (no cardiogénico o cardiogénico)

• Accidentes isquémicos transitorios

• Hipoglicemia • Crisis de pánico

• Cataplexia • Temblor, Coreoatetosis, Distonía

paroxística o Tics • Vértigo paroxístico • Aura migrañosa • Patología del sueño REM • Episodios de descontrol de impulsos

MANEJO INICIAL

MANEJO DE URGENCIA DE LA CRISIS AGUDA:

Aunque el resultado de un episodio de Estado epiléptico convulsivo (EEC) está determinado principalmente por su causa, la duración es también importante. Mientras más larga la duración del episodio, es más difícil de terminar, por lo que para propósitos prácticos, el acercamiento al niño que presenta con una convulsión tónico clónica que dura más de 5 minutos debe ser igual al niño con un status epiléptico establecido.

Page 37: Temas de examen Pediatria

ABC

• Colocar al paciente en posición decúbito lateral, aspirar secreciones. • Monitoreo de signos vitales • Oxigeno vía nasal o mascarilla a 100%. Favorecer vía aérea permeable,

oxigenación, realizar eventual intubación • Vía venosa permeable • Determinación de glicemia (capilar o micrométodo) para establecer la

necesidad de un bolo de glucosa

Bajar la T°:

• Paracetamol: 20 mg/Kg oral, o Ibuprofeno 10 mg/kg oral. .

Medicamentos de 1° línea

• No debe diferirse el tratamiento si no se logra una vía intravenosa.

Diazepam Midazolam Lorazepam

Inicio acción 1-3 min. 1,5-5 min. 1-3 min.

EV (mg/kg/dosis) 0,3 - 0,5 sin diluir 0,1-0,2 0,1

IM O,1-0,3 0,1

Rectal 0,5-1,0 0,5

Intranasal Sí

Recomendación Sin diluir

1-2 dosis IV intervalo 10 min

Lento. Sin diluir. Dosis máxima

5mg/dosis

Presentaciones Ampolla

Gel (5-10mg/kg)

Vigilar hipotensión y depresión respiratoria

Si en 5 minutos la crisis no ha cedido:

• Se puede aplicar dosis adicional de Diazepam (sin diluir) de 0.5 mg/Kg.

Si no se logra controlar:

• Utilizar Fenobarbital o Fenitoína IV a 15-20mg/Kg/dosis LENTO (10-20 min.), con monitorización continua.

Fenobarbital Fenitoína

EV (mg/kg/dosis) 15-20 15-20

Vel. de infusión 30 mg/min 1 mg/kg/min

Máximo 50mg/min

RAM

Hipotensión si es infundido rápidamente. Depresión

respiratoria, especialmente si se ha utilizado diazepam.

Hipotensión, arritmia.

1° opción: en TEC

• Nota: Si el paciente está previamente con fenitoína o fenobarbital, dosis de carga será de 5 mg/kg. Ajustando dosis subsecuentes según niveles plasmáticos.

Si la crisis no cede repetir por una vez

Si no cede ingresar a UCI para inducción anestésica

Pesquisa y terapia inicial de posibles causas precipitantes de status (hipoglicemia, alteración hidroelectrolítica, infección, fiebre, lesión ocupante de espacio) Descartar foco infeccioso.

Page 38: Temas de examen Pediatria

• Frente a una crisis febril, el aspecto más relevante es descartar una meningitis, por lo que se indica la punción lumbar en todos los lactantes menores de 12 meses y en los mayores ante la mínima sospecha, especialmente en niños con gran compromiso de estado general, vómitos, compromiso de conciencia mantenido, uso previo de antibióticos y obviamente ante la presencia de signos meníngeos.

• Si se está frente a un status EPILÉPTICO debe considerarse realizar una punción lumbar, debido que es frecuente su asociación con infecciones

• Tomar muestras de sangre. Las muestras pueden ser tomadas en APS y derivadas con el niño.

Punción lumbar:

No está indicada de forma rutinaria. De hacerla debe tomarse en cuenta las siguientes indicaciones:

• Primera convulsión febril en < 12 meses • Considerar entre 12-18 meses • > 18 meses no de rutina • De rutina en presencia de signos meníngeos o tratamiento ATB

Estudio de Neuroimagen:

• No es recomendada tras la primera convulsión febril (Academia Americana de Pediatría 1996)

• Indicada con examen neurológico anormal

Tratamiento profiláctico de las convulsiones

• Tratamiento continuo con anticonvulsivantes: -Acido valproico - Fenobarbital • Tratamiento selectivo intermitente: -Diazepam intrarrectal 0.5 mg/Kg/dosis c/12

horas si T>38grados

Tratamiento de Status de Ausencia y Status Mioclónico

• Ácido Valproico: 20 mg/kg dosis, jarabe 250 mg en 5 ml, diluir volumen 1/1 y emplear por ENEMA. (se obtienen niveles “Peak” a las 2-4 hrs. después de administrado.)

CRITERIOS DE HOSPITALIZACIÓN:

• Primera convulsión febril en menor de 18 meses • Convulsiones febriles o epilépticas con cambio importante en sus características

(patrón de crisis) o aumento importante del número de crisis. • Sospecha de enfermedad subyacente: encefalitis, meningitis, lesiones

postraumáticas, (maltrato infantil) etc.

• Determinación de gases, • Glucosa, • Calcio, • Electrolitos, • Hemograma, • Niveles de drogas antiepilépticas, • Cultivos (bacterianos y virales), • Estudios toxicológicos (dependiendo de la historia y examen físico).

Page 39: Temas de examen Pediatria

• Sospecha de Síndrome de West. • Alteración nueva del examen neurológico • Toda Convulsiones Febril Complejas. • Convulsión no febril. Son poco frecuentes. Se debe realizar una adecuada

historia clínica y exploración física y neurológica detallada, insistiendo en la toma de la presión arterial y de la glicemia.

NIÑO POSTCRÍTICO

• No necesita tratamiento específico. Asegurar vía aérea (decúbito lateral, tubo orofaríngeo), controlar constantes y vía venosa con sueroterapia de mantenimiento. Descender fiebre si la hay. Vigilar recuperación adecuada del nivel de conciencia. Investigación etiológica.

DERIVACIÓN

• Sospecha diagnóstica de Epilepsia (2 o más crisis convulsivas de causa no identificable), emitir Interconsulta con el diagnóstico de OBSERVACIÓN DE EPILEPSIA, esta debe incluir: anamnesis completa, descripción y caracterización del evento convulsivo + examen físico general y examen neurológico. Deseable fondo de ojo.

• Crisis febriles complejas se recomienda la evaluación con EEG, considerar realizar neuroimágenes e indicar tratamiento preventivo con Fenobarbital o ácido Valproico.

Un niño sano, con antecedentes de una crisis febril simple no requiere mayor estudio. No está indicado el EEG ni la realización de neuroimágenes. Sólo debe manejarse la fiebre y el foco infeccioso si corresponde y dar la información a los padres en relación al cuadro y las medidas para prevenir otra crisis.

Page 40: Temas de examen Pediatria
Page 41: Temas de examen Pediatria

Consideraciones • Petequias: hemorragias

puntiformes que tienen < 2 mm de diámetro

• Púrpura: hemorragias que tienen 2 a 10 mm de diámetro

• Equimosis: hemorragia >10 mm de diámetro

9. SÍNDROME PURPÚRICO

Leslie Raffini, MD, “Evaluation of purpura in children”, UpToDate, 2013

Clase Síndrome Purpúrico, UFRO, 2012

Mª Dolores López Saldaña, Púrpura de Schönlein-Henoch, Protocolos diagnósticos y terapéuticos en pediatría. AEPED.

MARIN M, ORDOÑEZ O, PALACIOS A. Manual de Urgencias de Pediatría, Hospital 12 de octubre, capitulo “Púrpura”, 2011.

DEFINICIÓN

Síndrome purpúrica engloba a varias condiciones clínicas que se presenta con purpura. Se define como purpura a la extravasación de glóbulos rojos en piel o mucosas y según tamaño puede ser clasificada como petequia o equimosis.

SOSPECHA DIAGNÓSTICA

La lesión purpúrica no desaparece con la presión

CBC: complete blood count; PT: prothrombin time; aPTT: activated partial thromboplastin time; DIC: disseminated intravascular coagulation; ITP: idiopathic thrombocytopenic purpura; HSP: Henoch Schoelein purpura; HUS: hemolytic uremic syndrome; TTP: thrombotic thrombocytopenic purpura. Courtesy of Leslie Raffini, MD.

Page 42: Temas de examen Pediatria

DIAGNÓSTICO DIFERENCIAL

Disrupción de la integridad vascular

Trombocitopenia (Plaquetas<150.000)

Alt. de la cascada de coagulación

• Trauma • Infección • Púrpura Schönlein

Henoch • Inducida por

drogas • Púrpura neonatal

fulminante • Sd Enhler Danlos

• Púrpura tombocitopénico Inmune

• Sindrome Hemólitico Uremico • Infiltración de la médula ósea • Trombocitopenia hereditaria • Drogas que inducen

trombocitopenia • Aplasia medular

• Enf. Von Willebrand • Hemofilia • Deficiencia vitamina

K • Hepatopatía • Otras deficiencia

congénitas

Trauma

• Es la etiología más frecuente

Infecciones:

• Pueden dar un púrpura fulminante. El meningoco es la causa más frecuente, pero existen otras como varicela, SBHGA, neumococo.

Púrpura de Schönlein-Henoch

• Es la vasculitis más frecuente en niños. Se presenta preferentemente en escolares (6 años apróx) varones y en meses de invierno.

• Es una vasculitis donde se produce depósito de IgA en la pared de los vasos sanguíneos y en el mesangio renal.

• Es de etiología desconocida, con frecuencia se encuentra el antecedente de afección del tracto respiratorio superior por el SBHGA, Yersinia o Mycoplasma (más raramente) o por virus (EB, varicela, parvovirus B-19, etc.). Otros desencadenantes pueden ser fármacos (penicilina, ampicilina, eritromicina, quinina), alimentos, exposición al frío o picaduras de insectos.

• Se caracteriza por • Púrpura palpable (de 2 - 10mm) simétrico, en EEII y nalgas

preferentemente, puede afectar cara, tronco y EESS. Y a menudo confluyen. Regresa en 1-2 semanas. Suele reproducirse al iniciar la deambulación.

• Artritis o artralgias, transitorias, no migratorias. Se inflaman grandes articulaciones (tobillos, rodillas). La inflamación es preferentemente periarticular y no deja deformidad permanente. Puede preceder al rash y ser la primera manifestación

• Dolor cólico abdominal que se debe a la extravasación de sangre y líquidos dentro de la pared intestinal, que puede llegar a ulcerarse, inaginarse o perforarse. El dolor suele aparecer después del exantema, asociado a vómitos si es grave.

• Manifestaciones renales. Son las que marcan la gravedad o el peor pronóstico a largo plazo. Pueden ir desde una hematuria aislada microscópica hasta la presencia de una glomerulonefritis rápidamente progresiva.

• Diagnóstico es clínico.

Page 43: Temas de examen Pediatria

Púrpura inducido por drogas

• Medicamentos que producen vasculitis: sulfonamida, penicilinas y fenitoína. La vasculitis usualmente se desarrolla dentro los 7-21 días de haber empezado el uso del medicamento.

• Medicamentos que producen trombocitopenia: cotrimoxazol, ácido valproico y fenitoina.

• Anticoagulantes

Púrpura Trombocitopénico Inmune

• Ocurre en niños entre los 2 -10 años, sin diferencia por sexo. Es la principal causa de trombocitopenia en niños.

• Es un trastorno agudo y autolimitado en un 50%. Suele existir una respuesta inmune exagerada ante un gatillante como infección viral.

• Se caracteriza por aparición súbita de petequias diseminadas. Se puede acompañar de hemorragias mucosas, y en menos del 10% con hematuria y hemorragia gastrointestinal. Menos del 1% va a presentar hemorragia intracraneal.

• El diagnóstico se basa en 2 características clínicas: • Trombocitopenia sin alteración de otra línea sanguínea. • Que no exista alguna condición clínica que pueda ser la causa de la

trombocitopenia.

TRATAMIENTO INICIAL

Púrpura Trombocitopénico Inmune

Manejo depende del riesgo hemorrágico. Es Expectante, salvo que haya manifestación de hemorragia activa. Recomendaciones generales:

Pacientes con recuento >20.000plaquetas/mm

3,

sin compromiso de otras series, sin

factores de riesgo, asintomático.

DERIVAR A POLICLÍNICO DE HEMATOLOGÍA. • No administrar fármacos IM. • No usar Ac. Acetilsalicílico ni derivados. • No usar AINES. • Restricción de actividad física hasta lograr nivel

hemostático seguro. • Protección gástrica frente al uso de corticoides y

terapia hormonal. Régimen hiposódico en caso de uso de corticoides.

Pacientes que al diagnóstico tengan <20.000 plaquetas/mm3

o presencia de sangrado, hasta que sea evaluado por hematología

Hospitalizar

Pacientes con sangrado incoercible Hospitalizar

Pacientes que cursen una emergencia o evento traumático Hospitalizar

Page 44: Temas de examen Pediatria

Considerar tratamiento: • En caso de

Sangrado por trauma significativo durante trombocitopenia severa. Por riesgo de Hemorragia intracraneal ⃰ (1/1000 niños con PTI)

Terapia corticoesteroidal: 70-80% de los pacientes responden. Reducen sangramiento por acción directa sobre el vaso sanguíneo. • Prednisona (más usado): 0.5 a 2 mg/kg/día por 2 a 4

semanas o 4 mg/kg/día por 3-4 días (max 60mg/día). Debe ser disminuida y suspendida después de 4 semanas de tratamiento.

• Metilprednisolona IV a 30 mg/kg/día por 2-3 días consecutivos.

Inmunoglobulina EV: >80% de los pacientes responden. 50% de los pacientes alcanza recuentos plaquetarios normales. • Dosis: 0.8 a 1 g/kg/día EV repitiendo la dosis a las 24h si

no hay respuesta.

Otros: Inmunoglobulina AntiD: No disponible en Chile. Tratamiento opcional, en casos especiales: Ac monoclonales o inmunosupresores.

Tratamiento de

emergencia: hemorragias

con riesgo vital

Transfusión de plaquetas: 1U por cada 5 kg de peso.

Inmunoglobulina EV 1 gr/kg y/o

Metilprednisolona 30mg/kg ev (máximo 1 gr).

Repetir inmunoglobulina o metilprednisolona en 24 horas si el recuento plaquetario es <50.000 y si se confirma el diagnóstico.

Ácido tranexámico 10 mg/kg/dosis cada 6-8 horas

Otros: Esplenectomía de urgencia o cirugía en el sitio de sangrado si fuera necesaria y factible (SNC, abdomen).

Remisión del PTI: • Respuesta completa: Recuento plaquetario >100.000/mm3 y sin sangramiento. • Respuesta parcial: Recuento plaquetario entre 30.000-100.000/mm3 y ausencia

de sangrado. • No respondedor: Recuento plaquetario <30.000/mm3

Púrpura de Schönlein-Henoch

• Se aconseja reposo en cama los primeros días. • Los AINE se emplean para aliviar las molestias articulares. • Los corticoides están indicados a dosis de 1-2 mg/kg en casos de intenso dolor

abdominal o hemorragia intestinal y si aparece vasculitis en SNC, afectación testicular o hemorragia pulmonar.

DERIVACIÓN

• Todo paciente en quien se sospechen problemas de hemorragia o coagulación.

Page 45: Temas de examen Pediatria

10. LEUCEMIA EN EL NIÑO

Apuntes clase “Cáncer en el niño” realizada por Dra. Primo, año 2010

MINSAL. Guía clínica leucemia en personas menores de 15 años, 2010.

MINSAL. Manual para la detección del cáncer infantil en centros de salud primaria

Nelson, Tratado de pediatría, “capítulo 487:Leucemias”, 17°edición

DEFINICIÓN

Las leucemias pueden definirse como un grupo de enfermedades malignas en la que una determinada célula hematopoyética da lugar a una proliferación clonal de células. La progenie de estas células muestra una ventaja de crecimiento sobre los elementos celulares normales, por su mayor velocidad de proliferación, por la menor incidencia de apoptosis espontánea o por ambos mecanismos.

EPIDEMIOLOGÍA

La leucemia es el cáncer más frecuente en los niños menores de 15 años correspondiendo al 35-40% de ellos. La incidencia, que es ligeramente mayor en los varones, varía de acuerdo al tipo de leucemia, siendo la Leucemia Linfoblástica Aguda la más frecuente (80% del total), seguido por Leucemia Mieloide Aguda. Las leucemias crónicas son muy poco frecuentes, menos del 5% del total y en general son de estirpe mieloide.

CLASIFICACIÓN

Edad de presentación de los diferentes cánceres en el niño.

Menor de 5 años 5 a 10 años Mayor de 10 años

Leucemias Leucemias Leucemias

Neuroblastomas Linfoma no Hodgkin Linfoma no Hodgkin

Wilms Enfermedad de

Hodgkin Enfermedad de

Hodgkin

Tu testicular Tu SNC Tu SNC

Retinoblastoma Sarcoma partes

blandas Tu células

germinativas

Leucemia

Trastornos linfoproliferativos

Leucemia linfoblástica aguda (LLA)

Linfoma

Trastornos mielproliferativos

Leucemia mieloide aguda (LMA)

Neoplasias mieloproliferativas.

Page 46: Temas de examen Pediatria

CAUSAS

• Origen desconocido.

Factor predisponente:

• Edad, Género, Raza, Historia familiar.

Factores precipitante:

• Radiación ionizante • HTLV (retrovirus) causante de la leucemia T del adulto • Inestabilidad cromosómica: anemia de Fanconi, ataxia-telangectasia • Sindrome de Down: incrementa el riesgo de leucemias agudas (10-20 veces) • Factores químicos: benceno, cloranfenicol.

CUADRO CLÍNICO

Prácticamente en todo paciente con leucemia se producen síntomas y signos secundarios a insuficiencia medular y/o Infiltración tumoral.

Insuficiencia medular Infiltración leucémica (Sd

Tumoral)

• Anemia (palidez), • Trombocitopenia (púrpura), • Neutropenia (fiebre neoplásica

por infección intercurrente)

• Adenopatías, • Hépatoesplenomegalia, • Infiltración encías (en leucemia

monocíticas).

Los siguientes síntomas y signos combinados pueden sugerir un cáncer hematológico:

• Fiebre • Dolor óseo, no asociado a

traumatismos • Hematomas • Sangrado fácil (encías, nasal,) • Infecciones recurrentes • Linfadenopatías

• Esplenomegalia • Poliartritis (Fenómenos

paraneoplásicos) • Cefalea y cambios de

comportamiento: los blastos leucémicos tienen afición por invadir el SNC

ANEMIA + PURPURA + FIEBRE Es una tríada sintomática que está presente casi en los 2/3 de los casos de

LEUCEMIA y si se acompaña de hepatomegalia, esplenomegalia, adenopatías y de hiperleucocitosis el diagnóstico está hecho.

MASAS CERVICALES

Más del 50% de todas las masas cervicales malignas corresponden a linfomas.

• L. HODGKIN: El 90% presentan adenopatías cervicales (generalmente unilaterales), con varios ganglios íntimamente relacionados entre sí formando paquetes. Se debe buscar otras adenopatías y esplenomegalia.

• LINFOMAS NO HODGKIN: las adenopatías son generalmente múltiples y

pueden ser bilaterales. En leucemia aguda son múltiples y muchas veces generalizadas.

Page 47: Temas de examen Pediatria

MASAS MEDIASTÍNICAS • De las masas mediastínicas en un niño, 40% son de origen maligno. • En mediastino anterior: las causas malignas son leucemia T o linfoma no

hodgkin tipo T, que afecte el timo, el diagnóstico diferencial debe hacerse con timo grande especialmente en lactantes y teratomas.

• En mediastino medio: las masas malignas corresponden principalmente a adenopatías hiliares por leucemias o linfomas de tipo T, linfoma de hodgkin. Las causas benignas más frecuentes son adenopatías secundarias a BRN o TBC (complejo primario) o quistes broncogénicos.

• En mediastino posterior: la principal causa maligna es el neuroblastoma.

RESEÑA DE CADA CUADRO CLÍNICO

LEUCEMIAS AGUDAS

• La Leucemia Linfoblástica Aguda corresponde al 75% de las leucemias en la edad pediátrica, con un peak de incidencia: 2 y 5 años de edad. Predominio varones.

• La historia generalmente es corta de unas 2 a 3 semanas o menos: palidez, cansancio, dolores ósteoarticulares cambiantes, y después o desde el inicio se presentan síntomas más alarmantes que hacen consultar: hemorragias (nariz, boca, piel petequias y equimosis), fiebre alta sin causa precisada y que se prolonga más de 2 días o se pasa y vuelve, dolores óseos que se hacen más intensos y producen impotencia funcional.

• Clínicamente es difícil diferenciar leucemia linfoblástica de la leucemia mieloide.

LINFOMA DE HODGKIN

• Se presenta clásicamente con un aumento de volumen cervical generalmente unilateral (menos frecuente en región axilar, inguinocrural).

• Adenopatías de carácter neoplásico (adenopatías) en cuello tercio medio e inferior y posterior a esternocleidomastoideo, fosas supraclavicular, axilas, mediastino, región inguinal, triángulo de Scarpa.

• La palpación permite definir conglomerado de adenopatías muy duras difíciles de delimitar entre sí que simulan un paquete o bolsa con nueces.

• Son adenopatías indoloras. • Puede haber síntomas generales: pérdida de peso (comparar ropa), sudoración

importante (especialmente nocturna), prurito, fiebre recurrente o que se prolongue más de una semana sin causa aparente.

LINFOMA NO HODGKIN

• Son los tumores de crecimiento más rápido en niños. • Se presentan más frecuentemente en la edad preescolar y escolar, pudiendo

presentarse a cualquier edad. • La forma más frecuente de presentación es como una masa abdominal que

afecta generalmente la zona ileocecal, de ahí que el síntoma más importante es el dolor abdominal que puede acompañarse de vómitos, distensión abdominal, diarrea, hemorragia intestinal en los últimos 15 días. A veces el dolor es muy intenso y puede ser signo de oclusión.

Page 48: Temas de examen Pediatria

DIAGNÓSTICO

La confirmación diagnóstica se realiza con la presencia de blastos >20% en médula ósea (según definición de OMS).

MANEJO INICIAL

• Pacientes con hematomas, sangrado, púrpura, o síntomas

• Sugerentes de anemia • Hemograma completo y VHS

• Niños > 4 años, con síntomas sugerentes de anemia, que no han sangrado

• Hemograma completo y VHS

• Todo paciente que presente dolor óseo, no relacionado a traumatismos u otra causa

• Hemograma completo y VHS, debe repetirse al menos una vez en caso de que las condiciones del paciente permanezcan en estudio y sin mejoría.

• Pacientes que presentan un cuadro infeccioso asociado con citopenias en hemograma (anemia y/o neutropenia, con o sin trombopenia)

• Referir dentro de 24 hrs (inmediato) a especialista

• Pacientes que presenten recuento de glóbulos blancos o frotis sanguíneo sugerente de leucemia aguda

• Referir dentro de 24 hrs (inmediato) a especialista

• Esplenomegalia persistente, sin otros síntomas que la explique.

• Derivar al especialista dentro de 24 hrs (inmediato)

• Pacientes con linfadenopatías de origen desconocido

• Linfadenopatías persistentes por más de 4 semanas

• presencia de ganglios >3 cm • presencia de ganglios palpables

en forma generalizada

• Solicitud de exámenes y envío a especialista

DERIVACIÓN

El paciente debe ser referido a un centro especializado autorizado para el tratamiento de leucemias agudas, que disponga de especialistas, con unidades de apoyo, unidades de aislamiento, etc. El PINDA (Programa de Cáncer Infantil) es un programa integral de detección, tratamiento y rehabilitación del Cáncer infantil que tiene una cobertura nacional En un Centro del PINDA se harán los exámenes necesarios: hemograma, mielograma, estudio de citometría de flujo, citogenética, etc., para el diagnóstico y para la tipificación de la leucemia.

Page 49: Temas de examen Pediatria

CONDUCTAS A SEGUIR MÉDICO GENERAL Y DE SERVICIOS DE URGENCIA

Ante la consulta de un niño por: • Sangramiento no explicado, en uno o

más sitios. • Palidez intensa sin causa explicable. • Fiebre sobre 38,5° C, asociado a uno

o ambos síntomas previos

Solicitar Hemograma, si presenta: • Anemia (con o sin sangrado) y/o • Neutropenia y/o • Trombocitopenia. • Leucopenia o leucocitosis con

presencia de células inmaduras.

Conducta:

1. Hospitalizar de urgencia.

2. Régimen cocido.

3. Evitar procedimientos invasivos.

4. Hidratación EV con suero alcalinizante sin K, 3000 cc/m 2 24 hrs,

asegurando diuresis de 100 cc/m2/hora.

5. Alopurinol 10 mg/kg cada 8 horas oral.

6. Solicitar exámenes de laboratorio de ejecución urgente (hemograma,

VHS, función renal, hepática, electrolitos plasmáticos, calcemia, fosfemia

y LDH, hemocultivos (2) y urocultivo); además imágenes (Rx. Tórax).

7. Iniciar tratamiento con antibióticos de amplio espectro ej :

• Amikacina 15 miligramos por k. de peso por día EV asociado con • Cefalosporina de tercera generación 150 mg/kg/día ev asociado con • Cloxacilina 200 mg/kg/dia ev • Si hay un foco infeccioso determinado, agregar antibióticos según

probables gérmenes involucrados. 8. Informar a padres y paciente (según su edad) de hipótesis diagnóstica.

9. Coordinar con centro especializado para el estudio y tratamiento de

leucemia aguda.

Considerar si corresponde:

Apoyo Transfusional: • Mantener recuento plaquetario sobre 20.000 x mm 3 . • Mantener Hb sobre 7,5 gr, siempre que no exista leucocitosis >100.000/ mm 3

Page 50: Temas de examen Pediatria

11. SÍNDROME FEBRIL SIN FOCO DEL LACTANTE

** Normas de atención de Servicio de Urgencia Infantil y Servicio Pediatría, Hospital Dr. Hernán Henríquez Aravena.

DEFINICIÓN

Enfermedad aguda febril, (T° rectal >38ºC) en la que después de una acuciosa historia clínica y un adecuado examen físico no es evidente la causa probable de la fiebre. Uno de los mayores y polémicos problemas a los que se enfrenta a diario el pediatra es la evaluación y el manejo de los niños menores de 3 años con fiebre sin foco aparente, con la posibilidad de que desarrollen una Bacteriemia oculta. Uno de los criterios más significativos en la búsqueda de estos pacientes que están cursando con bacteremia es el aspecto tóxico del paciente.

Apariencia tóxica: Se considera que un lactante tiene aspecto tóxico cuando está "pálido o cianótico, letárgico o inconsolablemente irritable, puede tener taquipnea y taquicardia con un mal llene capilar. La mayoría de las definiciones hablan de un lactante con un "aumento de la irritabilidad, anorexia, falta de contacto visual, mala perfusión y letargia. Esta es el principal factor de riesgo como predictor de bacteremia oculta, presentando un Odds Ratio de 5.26 (1.89-14.63).

La causa más frecuente de fiebre en los niños son las infecciones víricas. La etiología de infecciones bacterianas es diferente según la edad:

Neonatos: Bacilos gram negativos (Escherichia coli), estreptococos grupo B y con menor frecuencia Listeria monocitógenes, Salmonella y Neisseria meningitidis etc.

Lactantes de 1-3 meses de edad: Disminuyen las causas por gérmenes gran negativos y estreptococo grupo B y aumentan las producidas por S.pneumoniae, N. Meningitidis, Salmonella, H. influenzae (no vacunados)

Niños 3 a 36 meses: constituyen el mayor grupo de bacteriemia oculta, siendo actualmente, el Streptococcus pneumoniae la causa más frecuente de bacteriemia en lactantes de esta edad.

ENFRENTAMIENTO INICIAL

Es fundamental realizar una historia clínica detallada y examen físico meticuloso, así como pruebas de laboratorio seleccionadas. El riesgo de infección se basa en la edad, en el aspecto clínico y en el resultado de las pruebas analíticas. Las directrices se dividen en opciones conforme a la edad: • los < 1 mes, • de 1 - 3 meses y • de 3- 36 meses.

Page 51: Temas de examen Pediatria

CRITERIOS DE BAJO RIESGO (ROCHESTER) Niños 0 a 90 días

1. Buen estado general. 2. Previamente sano

• Recién nacido a término • Ningún tratamiento antibiótico en el periodo perinatal, ni • posteriormente • Ningún tratamiento por hiperbilirrubinemia idiopática • Sin hospitalización previa. • Ausencia de enfermedad crónica • No permaneció en la maternidad más tiempo que la madre.

3. Sin evidencia de infección en la exploración clínica 4. Pruebas de laboratorio

• Leucocitos: 5000-15.000/mm3. • Neutrófilos <10.000/mm3. • Sedimento urinario con 10 leucocitos/campo • Heces (sí diarrea) con <5 leucocitos/campo

Escala YIOS Escala YALE

Lactantes <3meses Lactantes >3meses

1. ACTITUD • Sonríe o no está irritable

(1) • Irritable pero consolable

(3) • Irritable no consolable (5)

2. ESTADO/ESFUERZO RESPIRATORIO

• No hay deficiencia y está vigoroso (1)

• Deterioro leve-moderado (taquipnea, retracciones, ronquidos) (3)

• Dificultad respiratoria o esfuerzo inadecuado (apnea, insuficiencia respiratoria) (5)

3. PERFUSION PERIFÉRICA • Extremidades frías,

moteadas (3) • Extremidades de color

rosa y con temperatura normal (1)

• Sensibilidad y

especificidad: 75%

• Valor predictivo negativo: 96%

1. Características del llanto • Grito potente de tono normal, o contento y sin

llanto (1) • Gime o llora (3) • Grito débil o gemido o de tono alto (5)

2. Reacción a la estimulación de un progenitor • Llora brevemente y deja de hacerlo o está contento

y no llora (1) • Llora por lapsos (3) • Llanto continuo o apenas si reacciona (5)

3. Variación de su estado • Si está despierto, así permanece, o si está dormido

y se le estimula, despierta (1) • Cierra brevemente los ojos cuando está despierto,

o despierta con la estimulación duradera (3) • Se duerme o no puede ser despertado(5)

4. Color • Rosado (1) • Extremidades pálidas o acrocianosis (3) • Pálido cianótico o grisáceo

5. Hidratación • Piel y ojos normales y mucosas húmedas (1) • Piel y ojo normales y boca levemente seca (3) • Piel pastosa con pliegue, mucosas secas y/o ojos

hundidos (5) 6. Reacción (habla, sonrisa) a estímulos sociales

• Sonríe o está alerta (1) • Sonríe o está alerta brevemente (3) • No sonríe, cara ansiosa o inexpresiva o no está

alerta(5) • Puntuación≤7 descarta

infecciones bacterianas graves.

Puntuación: ≥ 10 –Poco riesgo de enfermedad grave Puntuación : ≥ 16—Alto riesgo de enfermedad grave

Page 52: Temas de examen Pediatria

DIAGNÓSTICO DIFERENCIAL

Meningitis, sepsis, neumonías, infecciones del tracto urinario, osteomielitis, artritis séptica, enteritis bacteriana.

MANEJO INICIAL Y DERIVACIÓN

RECIÉN NACIDOS:

1. Hospitalizar. 2. Buscar foco de infección e iniciar terapia antibiótica. 3. Solicitar: hemocultivos, urocultivo, bioquímico y cultivo de LCR, Rx tórax. 4. ATB: Ampicilina más cefalosporina de tercera generación.

LACTANTE FEBRIL (28 A 90 DÍAS)

SIN APARIENCIA TÓXICA:

1. Buscar foco: hemocultivos, urocultivo, y estudio de LCR y dar una dosis de Ceftriaxona 50 mg/kg Intravenosa o intramuscular.

2. Reevaluar en 24 horas. • Si hemocultivos o cultivo de LCR son positivos, hospitalizar con

tratamiento antibiótico intravenoso. • Si urocultivo es (+) seguir norma de infección urinaria. • Si cultivos son negativos de alta, con medidas generales y con

control en forma ambulatoria.

CON APARIENCIA TÓXICA:

1. Hospitalizar. Hemocultivos, urocultivo, bioquímico y cultivo de LCR, e inicie tratamiento antibiótico IV.

LACTANTE FEBRIL (91 DÍAS A 36 MESES)

SIN APARIENCIA TÓXICA:

1. Si T° ≥ 39°C, solicite exámenes para determinar infección: hemograma, PCR, Rx Tórax, examen de orina y urocultivo. • Si los exámenes descubren el foco de infección, trate de acuerdo a

las normas. • Si el cribado de infección es positivo: PCR elevada, leucocitosis

>15.000, recuento absoluto de neutrófilos > 10.000, trate con ceftriaxona 50 mg/kg IV o IM. Controle en 24 horas. Si los cultivos son positivos hospitalizar y continuar terapia de acuerdo al foco infeccioso.

• Si los exámenes son normales dar de alta tratando la hipertermia, con Paracetamol 15 mg/kg o Ibuprofeno 10 mg/kg.

2. Si T° ≤ 39°C, trate la fiebre. Si el examen clínico es normal dar de alta con medidas generales, antitérmicos si son necesarios y cítelo a un control en consultorio.

CON APARIENCIA TÓXICA:

Hospitalizar e iniciar antibióticos por vía parenteral una vez realizados los cultivos.

Page 53: Temas de examen Pediatria

12. SÍNDROME FEBRIL PROLONGADO

Apunte de clase “Sindrome febril” realizada por Dr. Soza, UFRO.

Síndrome febril prolongado,, Dra. Tamara Viviani, SOCHIPE 2011.

PEREDO MS., VIVIANI T., PEÑA AN., Etiología del síndrome febril prolongado en niños. Rev Chil Pediatr 2007; 78 (5): 472-476

DEFINICIÓN

• T° > 38° axilar sobre 7 sin causa conocida, realizados exámenes básicos rutinarios.

• Según altores, a cualquier edad >7días, para otros <2 años: >5-7 días de fiebre, preescolar y escolar: >10 días, adolescente y adulto, entre 14-21 días.

• Síndrome febril sin foco: agudo, semiología y exploración básica infructuosas

• Síndrome febril prolongado: factor tiempo • Fiebre de origen desconocido (FOD): una vez efectuada una

exploración extendida

• Es más frecuente una presentación inusual de una infección frecuente que una

presentación usual de una infección infrecuente

ETIOLOGÍA

• Infección: 50-80% (más frecuentes respiratorias, digestivas, urinarias). • Mesenquimopatías: 10-15% (enfermedad reumática, AR, lupus eritematoso). • Neoplasias: 6-8% (leucemias, linfomas). • Misceláneas: 5-10% (diabetes). • Sin diagnóstico: 10%

ETIOLOGÍA POR EDAD

• <5años: predomina etiología infecciosa • 5-15 años: ↓ causa infecciosa, ↑ las oncológicas y reumato-inmunológicas • >15 años: sigue en ↑ causas reumato-inmunológicas y oncológicas.

TAMIZAJE INICIAL: ¡UNA REVISIÓN SISTEMÁTICA!

• Síntomas/signos: grales. y por sistema, exantemas, artralgias/artritis, secuencia de aparición y evolución

• Antecedentes: mórbidos (DVP, CC), contactos humanos y c/mascotas, fármacos previos, vacunas (BCG), estructura y dinámica familiar, viajes

• Examen físico: estado gral., ganglios, piel y mucosas, tejidos blandos, ap. locomotor, ex. Neurológico

• Infección Viral: PCR cuantitativa: inferior a 40-60 mg/L, Hemograma: leucopenia absoluta o relativa, linfocitosis, monocitosis. • VHS: < 40 mm/h

• Infección Bacteriana o inflamación: PCR cuantitativa: superior a 90 mg/L, Hemograma: neutrofilia, leucocitosis, anemia, trombopenia. • VHS: > 90 mm/h

Page 54: Temas de examen Pediatria

DIAGNÓSTICO DIFERENCIAL

Tabla: Diagnóstico etiológico específico de SFP

Casuística H. Sotero Del Río en 16 meses (n: 35).

M. S. Peredo y cols. Rev Chilena Pediatr 2007; 78 (5): 472-6

Virus de Epstein Barr

• Fiebre sostenida, poca cuantía, sin criterios de S. mononucleósico, cualquier edad.

• Hemograma anodino o linfocitosis. • Transaminasas: N o ↑ • IgM anti cápside: (+). Preferir IFI sobre ELISA

Citomegalovirus

• Fiebre de poca cuantía, BEG o astenia • Hospedero inmunocompetente • Adenopatías: variable • Antecedente de transfusión(es) sanguínea(s): a veces • Hemograma anodino o linfocitosis • Transaminasas: variable • IgM (+): ELISA

Bartonella henselae

• Clásico: enfermedad por rasguño de gato • Puede causar abscesos ocultos, focos osteomielíticos, FOD • Con frecuencia el antecedente de contacto con gato es desconocido u olvidado • Dg.: IgG > 1:256, ideal es obtener mayor dilución (+). Seguridad en descartar

otras causas • Utilidad terapéutica: incierta

Artritis inmunológica juvenil

• Fiebre héctica varias semanas, cede temporalmente con AINES • Exantema fugaz con la fiebre • Artralgia/artritis tardías

Page 55: Temas de examen Pediatria

• Escolares/ adolescentes • Se conserva relativo buen estado general

• Alteración significativa de reactantes de fase aguda: VHS-PCR-recuento de leucocitos

• Dos o más cursos de antimicrobianos sin efecto • Poliserositis en exploración por imágenes • Serología reumatológica (-). • Mielograma con aumento de serie blanca, neutrofilia

Fiebre asociada a fármacos

• Drug reaction with eosinophilia and systemic symptoms (DRESS) • Fiebre, CEG, exantema, adenopatías, compromiso multi-órganos (hígado, riñón,

pulmón, SNC) linfopenia o linfocitosis, eosinofilia • Asociado a: fenitoína, carbamazepina, fenobarbital, lamotrigina, sulfas, dapsona

MANEJO INICIAL

Paciente crítico vs no crítico

• No crítico: avanzar estudio (imágenes, biopsias) • Crítico (“séptico”): iniciar terapia antimicrobiana. Obtención de muestras previas

para serología, estudio inmunológico, microbiológico

RECOMENDACIONES

• Óptima relación con la familia • Mantener una actitud veraz, segura y prudente • Entregar una guía anticipatoria de la FOD • Sistematizar la observación clínica • Sistematizar el estudio • No “cazarse” con diagnósticos sin fundamentos • Solicitar oportunamente el concurso de terceros

• Pruebas terapéuticas juiciosas

S.F.P.

Hemograma - VHS -PCR elevados en

forma significativa

Infecciosos: imágenes (SNC-CV-Abdomen) -cintigrama-

biopsiascultivos- serologías. Prueba terapéutica

InmunológicoTumoral: mielograma biopsias serología.

Prueba terapútica

Hemograma - VHS -PCR normal o

cerca de normalidad

CMV- VEB

Page 56: Temas de examen Pediatria

DERIVACIÓN

Si a la 3° semana no mejora, fiebre no cede, exámenes se mantienen alterados o empeoran, continuar estudio: • Cintigrama ósea • TAC tórax, abdomen y cavidades paranasales • Exámenes reumatológicos • PPD • Biopsias (ganglio, piel)

Page 57: Temas de examen Pediatria

13. INFECCIÓN TRACTO URINARIO

Actualización en el diagnóstico y manejo de la infección Urinaria en pediatría, SOCHIPE 2012. rev chil Pediatr 2012; 83 (3): 269-278

DEFINICIÓN

• La infección del tracto urinario (ITU) se define clásicamente como la invasión, colonización y proliferación bacteriana del tracto urinario, que puede comprometer desde la vejiga hasta el parénquima renal.

• Afecta con mayor frecuencia a pacientes de sexo femenino en todas las edades, a excepción de los primeros 3 meses de vida.

SOSPECHA DIAGNÓSTICA

Si sale Leucocitos (-) y Nitritos (-) en paciente sintomático esperar resultado URC y sospechar otra causa.

FORMAS DE PRESENTACIÓN

Cistitis o ITU baja

• Infección limitada a la vejiga y a la uretra. Es más frecuente en mujeres > 2 años. • Clínica: disuria, poliaquiuria, urgencia, orina turbia, y molestias abdominales

bajas.

Pielonefritis aguda o ITU alta

• Compromete el parénquima renal. Es la forma más grave de ITU en niños. • Clínica: síntomas sistémicos como fiebre alta, compromiso del estado general,

decaimiento, dolor abdominal, dolor lumbar y frecuentemente vómitos y mala tolerancia oral. Los 2 elementos más sugerentes son fiebre y dolor lumbar.

ITU atípica

• Se comporta como una ITU alta que evoluciona en forma tórpida. Además pueden presentarse elementos que sugieren alteraciones anatómicas o funcionales de la vía urinaria tales como:

• Chorro urinario débil. • Masa abdominal o vesical. • Aumento de creatinina. • Septicemia. • Falla de respuesta al tratamiento antibiótico a las 48 horas. • Infección por germen no E. coli.

Sospecha Daignóstica (debe

plantearse frente a ...

Historia sugerente

Examen físico sugerente

MÁS examen de orina compatible

Leucocitos (+) y Nitritos (+)

Leucocitos (-) y Nitritos (+)

Clínica puede sugerir cualquier forma de presentación de ITU

Page 58: Temas de examen Pediatria

DIAGNÓSTICO

• El diagnóstico se confirma con un UROCULTIVO (+). El recuento de colonias significativo dependerá del método por el cual haya sido tomada la muestra.

Método de recolección N° de colonias por ml

Punción suprapúbica >1

Sondeo transuretral ≥ 10.000

Segundo chorro ≥ 100.000

Recolector ≥ 100.000

En niños sin control de esfínteres se recomienda el sondeo transuretral de no ser posible realizar punción vesical, idealmente bajo visión ecográfica directa.

En niños > 2 años o continentes una muestra de orina obtenida por segundo chorro es adecuada.

Debido al alto índice de contaminación un urocultivo positivo obtenido por bolsa recolectora no se considera ITU (bajo VPP, pero alto VPN)

• En aquellos casos donde exista duda el cintigrama renal con ácido dimercaptosuccínico (DMSA) o ecografía doppler renal pueden ser exámenes que ayudan a confirmar el diagnóstico.

TRATAMIENTO

Medidas generales

• Hidratación adecuada. • Educar sobre hábitos miccionales. Micción c/3hrs, no posponer deseo miccional. • Educar sobre hábitos defecatorios: evitar constipación, aseo adecuado.

TRATAMIENTO ANTIBIÓTICO ITU BAJA

• Lactantes entre 1-4 meses (cefalosporinas) • Cefadroxilo 50mg/kg/día c/8 -12 hrs VO

• Lactantes >4 meses • Nitrofurantoína 5-7 mg/kg/día cada 8-12 hrs VO • Cotrimoxazol 40/7 mg/kg/día cada 12 hrs. VO • Cefalosporinas

• Como 2°línea se pueden utilizar Quinolonas. • Ciprofloxacino 20-30 mg/kg/día c/24hrs VO, EV.

Duración del tratamiento: En ITU baja sin antecedentes de patología conocida de la vía urinaria, la evidencia actual se inclina hacia el tratamiento acortado de 3-4 días versus el tratamiento estándar de 7 días.

Si el paciente recibía profilaxis, debe iniciar tratamiento con otro antibiótico.

TRATAMIENTO ANTIBIÓTICO ITU ALTA

Lactantes <3 meses:

Page 59: Temas de examen Pediatria

Debe tratarse con ATB que cubran los gérmenes causantes de sepsis neonatal. Una vez confirmado el diagnóstico de ITU se continúa sólo con cefalosporina. Las terapias empíricas propuestas son:

• Ampicilina 150mg/kg/dosis c/12hrsEV + Gentamicina 4mg/kg/dosis c/24hrs. EV ó

• Ampicilina 150mg/kg/dosis c/12hrsEV + Cefotaxima 50mg/kg/dosis c/8hrs EV

Lactantes >3 meses

• Si no tiene aspecto tóxico, y es posible evaluarlo a las 48 hrs siguientes, se puede indicar ATB oral desde el inicio, Cefalosporinas.

• Cefadroxilo 50mg/kg/día c/8 -12 hrs VO • Cefuroximo 30mg/kg/día c/12hrs VO

• Si requiere tratamiento EV, se sugiere: • Ceftriaxona 100mg/kg/día c/24hrs EV

• Tratamiento de 2° línea: • Aminoglucósidos o quinolonas.

Duración del tratamiento: 7- 10 días. en recién nacido 10-14 días.

INDICACIONES DE LA HOSPITALIZACIÓN:

• Edad menor a 3 meses. • Sepsis clínica o potencial bacteremia. • Inmunosupresión. • Vómitos o incapacidad de tolerar medicamento oral. • Falta de adecuado control ambulatorio. • Falta de respuesta a terapia ambulatoria.

CONTROL

Control clínico a las 48 hrs • Sugiere buena respuesta a tratamiento la mejoría clínica a las 48 hrs y

el urocultivo positivo a E. coli. • Se define como mala respuesta al tratamiento la mantención de los

síntomas o el mayor compromiso del estado general a las 48 hrs. En este caso sospechar ITU atípica y realizar OC+URC de control y estudio de imágenes en forma precoz

Recomendaciones de estudio de imágenes

El estudio con imágenes en niños con un primer episodio de ITU debe ser selectivo, ya que el estudio rutinario a todos los niños no ha demostrado efectividad clínica y tiene un costo elevado. El estudio completo ya sea precoz o diferido se debe focalizar en los grupos de riesgo de daño renal:

• Menores de 6 meses (mayor riesgo de malformaciones y obstrucción de la vía urinaria que niños mayores).

• ITU atípica (alto riesgo de defectos parenquimatosos renales significativos y anomalías estructurales significativas).

Page 60: Temas de examen Pediatria

• ITU recurrente (la recurrencia de ITU se asocia a aumento progresivo del riesgo de daño renal).

Niños 0-12 meses Niños 1-3 años

Bu

ena

resp

ues

ta

ITU

gra

ve

o a

típ

ica

ITU

re

curr

ente

Bu

ena

resp

ues

ta

ITU

gra

ve

o a

típ

ica

ITU

re

curr

ente

Ecografía precoz NO SI SI NO SI NO

Ecografía diferida SI NO NO SI NO SI

DMSA 6-12 meses post-ITU febril

SI SI SI SI SI SI

UCG SI SI SI SELECTIVA SI SI

Estudio Utilidad

Cintigrama renal con DMSA

• Es el gold standard para el diagnóstico de pielonefritis, • Examen de referencia para el diagnóstico de defectos

parenquimatosos renales.

Ecografía renal

• Examen de elección para identificar alteraciones anatómicas (anomalías renales de número o de posición, doble sistema excretor, quistes, litiasis y alteraciones vesicales)

• Diagnostica abscesos renales

Eco renal con power doppler

• Permite localizar el nivel de la ITU

Uretrocistografía • Examen de referencia para la detección del RVU

(S:55%).

Cistografía isotópica directa

• Tiene menor dosis de radiación y es más sensible que la UCG para detectar todos los grados de RVU (S:97%).

SEGUIMIENTO

• Todo paciente tratado por ITU debe realizarse un OC+URO una vez terminado el tratamiento con el objetivo de confirmar la desaparición de microorganismos en la vía urinaria.

• Pacientes con ITU recurrente deben ser evaluados por especialista para decidir estudio y tratamiento a seguir.

ITU recurrente: se define como 3 o más ITU bajas, 2 o más pielonefritis o 1 pielonefritis más 1 ITU baja en un año

Profilaxis

Se aconsejan utilizar profilaxis sólo en las siguientes situaciones: • Diagnóstico antenatal de anomalía vía urinaria mientras completa estudio. • Menor de 2 años con ITU febril, hasta completar estudio de imágenes. • RVU GIII o mayor, ya que pacientes con grados menores de reflujo tienen baja

posibilidad de presentar nueva ITU febril. • ITU recurrente. • Disfunción vesical, mientras mejora patrón miccional.

Page 61: Temas de examen Pediatria

14. MENINGITIS BACTERIANA

** Normas de atención de Servicio de Urgencia Infantil y Servicio Pediatría, Hospital Dr. Hernán Henríquez Aravena.

DEFINICIÓN

• Enfermedad inflamatoria de las leptomeninges (aracnoides y piamadre).

Clasificación de meningitis

• Bacteriana o Piógena. • Aséptica. No evidencia infección bacteriana, usualmente con pleocitosis.

• Infección no bacteriana viral o micótica. • Inflamación no infecciosa: sarcoidosis, neoplasias, drogas.

CAUSAS

Recién Nacido: • Escherichia coli (K1). • Streptococcus agalactie (Grupo B). • Listeria monocytogenes. • Otras bacterias Gram (-). • Staphilococcus aureus.

Lactantes y Pre-escolares: • Streptococcus pneumoniae. • Neisseria meningitidis. • Haemophilus influenzae b* Escolares y Adultos: • Streptococcus pneumoniae. • Neisseria meningitidis.

* Notable disminución por efecto de vacunas.

CLÍNICA

• RN: Síntomas inespecíficos como CEG, rechazo alimentario, hipotermia o fiebre, vómitos, irritabilidad, convulsiones, hipoactividad.

• Lactantes: Inicio con fiebre y tos, llanto intenso y de tono agudo. Convulsiones, abombamiento de la fontanela anterior. Rigidez de nuca y signos meníngeos infrecuentes.

• Pre-escolares y Escolares: Síntomas generales: fiebre, calofríos, vómitos y cefalea. Compromiso de conciencia. Signos de focalización (peor pronóstico). Signos meníngeos Kernig y Brudzinsky.

• Los síntomas cutáneos como petequias y equimosis son sugerentes de meningococemia y de peor pronóstico, exigiendo conducta de manejo agresiva.

Indicaciones PL en sospecha del diagnóstico de meningitis

• Clínica de Meningitis, Apariencia Tóxica • Sospecha de Sepsis Neonatal • Lactante Febril < de 6 semanas • Sospecha de Sepsis en inmunodeprimidos. • Fiebre y Petequias • Enfermedad Febril en paciente que tuvo

cercano contacto con paciente con meningitis

Contradicciones para realizar una punción lumbar

• Compromiso cardiorrespiratorio grave

• Infección de la Piel • Coagulopatía (CID Aumenta

el riesgo de hematomas subaracnoídeos espinales)

• Hipertensión Endocraneana

Page 62: Temas de examen Pediatria

Situaciones en que es recomendable realizar TAC cerebral previo PL

• Compromiso importante de conciencia Glasgow < 10 • Pacientes inmunodeprimidos • Focalidad al examen neurológico • Compromiso de algún par craneano • Edema de papila • Sospecha de Hipertensión intracraneana

Características del LCR en niños sanos y en con diagnóstico de meningitis

Niño normal

RN Meningitis Bacteriana

Meningitis Viral

Leucocitos 0-6 0-30 > 1000 10 - 10.000

Neutrófilos 0 2-3 > 50 < 40

Glucosa (mg/dl) 40-80 32-121 < 30 > 30

Proteínas (mg/dl) 20-30 19-149 > 100 50-100

Eritrocitos 0-2 0-2 0-10 0-2

DIAGNÓSTICO

1. Cuadro clínico: Como presentación esporádica o asociado a un ambiente epidemiológico (enfermedad meningocócica).

2. Exámenes de Laboratorio: Punción Lumbar • Procedimiento básico del diagnóstico para el estudio del LCR. • Deben tomarse 3 muestras: 2 cc para estudio citoquímico, 0.5 cc para

estudio bacteriológico y 1 cc para examen de Látex. • Aspecto: Turbio a purulento, inicialmente puede ser claro. Presión

aumentada. • Glucosa: Típicamente con baja con valor inferior al 50% de la glicemia. • Proteínas: Aumentadas. • Leucocitos: >1000/mm

3 con predominio PMN. Inicialmente puede ser

normal o con predominio mononuclear (MN). • Estudio bacteriológico: Tinción de Gram: Orienta, sugiere pero no

informa etiología. No requiere de bacteria viva. • Cultivo: Siembra inmediata en medios de cultivo (Agar sangre, Agar

chocolate y caldo enriquecido). Incubación 37°C, 24 a 48 horas. Confirma diagnóstico etiológico, sensibilidad de agentes, detección de marcadores epidemiológicos. Rendimiento ≥ 80% si es precoz.

• Pruebas de diagnóstico rápido (Látex): Detectan antígenos bacterianos, específicos y sensibles aún con tratamiento antibiótico previo.

Hemocultivos: 2-3 muestras c/10-15 min., previo inicio de tto ATB. Otros cultivos: Lesiones piel, mucosas, derrames articulares, pleural, pericardio. Hemograma: Puede ser normal inicialmente. Clásicamente leucocitosis de predominio PMN. Proteína C Reactiva (PCR): Elevada. Tiene valor para discriminar infección bacteriana o viral y evolutiva para detectar precozmente complicaciones.

Page 63: Temas de examen Pediatria

MANEJO INICIAL

• Todo paciente con diagnóstico de MBA debe ser internado en UCI Pediátrica por lo menos 24-48 horas y dependiendo de la gravedad inicial y evolución.

El tratamiento inicial debe considerar la estabilización hemodinámica.

1. Aporte de volumen: 20-30 ml de suero fisiológico (SF) en forma rápida en los primeros 20-30 minutos.

• Si hay shock séptico el aporte se hará con mayor frecuencia, con evaluación posterior a cada bolo, monitorización estricta, y eventual conexión a ventilación mecánica (VM).

2. Uso de drogas inotrópicas: Evaluar condición del paciente especialmente

en primeras 24-48 horas. Evitar depresión cardiovacular. • Dopamina: 8-10 ug/kg/min • Dobutamina: 8-10 ug/kg/min

3. Corticoides: Deben distinguirse dos situaciones: • Shock séptico con signos de insuficiencia suprarrenal aguda, que es más

frecuente en meningococemia. Metilprednisolona: 4 mg/kg/iv cada 4 horas. Reducción progresiva 24-48 horas.

• Prevención de sordera. Uso de dexametasona 0.15 mg/kg/dosis cada 6 horas durante los primeros 4 días. Consensuado sólo para Meningitis por Haemophilus influenzae b, discutible en otras etiologías. Uso precoz idealmente 15 minutos antes de primera dosis de antibióticos.

4. Tratamiento anticonvulsivante. • Fenitoína 15-20 mg/kg/iv dosis de carga, que puede usarse en forma preventiva.

En presencia de convulsiones completar 30 mg/kg/día. Si persiste convulsión o paciente se encuentra en coma, usar Fenobarbital 20 mg/kg/ dosis inicial y luego según evolución.

5. Prevención y Tratamiento del Edema Cerebral. • Prevención: Evitar hipoventilación, adecuado aporte de oxígeno especialmente

en presencia de neumonía. Evitar hipotensión, tratar shock en forma adecuada. Evitar aporte exagerado de líquidos hipotónicos. Control adecuado de convulsiones especialmente en lactantes. Tener presente la complicación por secreción inapropiada de hormona antidiurética (ADH).

• Tratamiento: Cabeza en 30°, si no hay hipotensión. • VM con hiperventilación controlada. • PCO2: 25-30 mm Hg

Otras medidas: • Monitoreo: Según gravedad uso de catéter central para presión venosa central

(PVC) y catéter para presión arterial (PA). • Sedación: Diazepam 0.3 mg/kg o Midazolam 0.1 mg/kg, con VM adecuada. • No se usa Manitol en forma rutinaria, sólo en caso de fracaso con otras medidas

e idealmente con medición de presión intracraneana (PIC).

Page 64: Temas de examen Pediatria

• Antitérmicos: (no dipirona), con precaución evitando hipotermia.

6. Uso de Antibióticos • Debe ser precoz, inmediatamente establecido el diagnóstico y tomadas

muestras para estudio bacteriológico (LCR, sangre, posibles colecciones). Por excepción antes de tomar muestra (posible meningococemia grave, paciente en shock).

• El tratamiento inicial siempre es empírico, ajustando esquemas según agente etiológico identificado.

ESQUEMA ANTIBIÓTICO (RECIÉN NACIDO A 2 MESES)

EMPÍRICO BIASOCIADO

Cefotaxima • <7 días ó <2Kg, 150 mg/kg/día. C/12 hrs.

• >7 días 200 mg/kg/día. C/6 hrs. Ampicilina • Menor de 7 días ó menor de 2000 gramos, 100 mg/kg/día. Cada 12 horas. • Mayor de días 200 mg/kg/día. Cada 6 horas. • Mayor de 14 días 400 mg/kg/día. Cada 6 horas.

ETIOLÓGICO

Estreptococo A ó B

Penicilina G o Ampicilina (dosis similares) • Menor de 7 días 150 mil U/kg/día. Cada 8 horas. • Mayor de 7 días 200 mil U/kg/día. Cada 6 horas. • Mayor de 14 días 500 mil U/kg/día. Cada 4 horas. • Asociación de Aminoglicósido opcional primeros 7 ds.

Listeria • Continuar con Ampicilina

Bact. Gram (-) • Continuar con Ampicilina + Cefotaxima

ESQUEMA ANTIBIÓTICO (MAYOR A 2 MESES)

EMPÍRICO

Ceftriaxona 200mg/kg/día. C/12 hrs. Primeras 48-72 horas. Luego c/24 hrs.

ETIOLÓGICO

Neumococo* Penicilina Sódica ó continuar con Ceftriaxona • 500.000 U/kg/día. Cada 6 horas.

Meningococo • Cambio a Penicilina Sódica ó continuar con Ceftriaxona

Haemophilus influenzae b

• Continuar con Ceftriaxona

* Es importante sensibilidad regional del agente. En lugares de elevada resistencia el tratamiento empírico inicial es asociación de Vancomicina con Ceftriaxona, suspendiendo el antibiótico que corresponda según patrón de resistencia. En nuestro medio no se justifica aún esta medida, como tampoco PL de control a las 24 horas para detectar erradicación del agente.

Duración del tratamiento

• Etiológico: • Listeria: 14 días • Estreptococo: 14 días

• Gram (-) RN: 21 días

• Desconocido: • RN a 2 meses: 21 días • Mayor de 2 meses: 10-14

días

Page 65: Temas de examen Pediatria

• Meningococo: 7-10 días • Neumococo: 14 días • Hib: 10 días

7. Medidas Epidemiológicas

Aislamiento Respiratorio: Mínimo 24 horas de tratamiento antibiótico. Notificación Obligatoria: • Denuncia Inmediata por vía telefónica a Oficina de Epidemiología,

Secretaría Regional Ministerial de Salud, • Boletín de Denuncia Epidemiológica del Ministerio de Salud.

COMPLICACIONES

Inmediatas Mediatas

• Shock endotóxico con CID • Edema cerebral • Status convulsivo • Otras: secreción inapropiada, ADH

• Colección subdural • Empiema • Hidrocefalia

DERIVACIÓN

• En la IX Región, por su ruralidad, los pacientes deben trasladarse estabilizados,

con vía venosa y en lo posible con punción lumbar realizada. • En las primeras horas se pueden presentar complicaciones graves que pueden

poner en riesgo la vida y decidir el pronóstico, como shock séptico, estado convulsivo o edema cerebral.

Page 66: Temas de examen Pediatria

15. EXANTEMAS VIRALES

Apuntes de clase “Exantemas en pediatría”, realizada por Dr. Soza.

Manual CTO de Medicina y Cirugía, Capítulo 7.1. Enfermedades Exantemáticas. 8a edición

DIAGNÓSTICO DIFERENCIAL

1° enfermedad: Sarampión (measles, rubeola)

2° enfermedad: Escarlatina

3° enfermedad: Rubeola (rubella)

4° enfermedad: Kawasaki

5° enfermedad: Eritema infeccioso causada por el Parvovirus b-19

6° enfermedad: Exantema súbito (o roséola infantil), causada HVS 6-7

Varicela

Enterovirus

Eritema Multiforme

Page 67: Temas de examen Pediatria

1° Enfermedad 2° Enfermedad

Sarampión Escarlatina

Sinónimos

Edad Cualquier edad Escolares (5-15 años)

Etiología Morbilivirus - ARN SBHGA (pyogenes)

Mecanismo de infección

Respiratoria Respiratoria,

Secundaria herida

Contagiosidad 5 días antes de la erupción, hasta

5 días despúes. En fase aguda hasta 24hrs

después de comenzar tto ATB

Incubación 1-2 sem. 1 sem.

Pródromo 3-4 días: fiebre ++/+++, tos seca,

rinitis y conjuntivitis con fotofobia

12-24 hrs: Faringitis con fiebre +++/+++, aspecto de enfermedad

grave.

Exan

tem

a

Tipo

Maculo-papulares (rojo-intenso), que no se blanquea con la

presión. Al aparecer el exantema aparece fiebre +++/+++

Maculo-papulares puntiforme difusas (escarlata), “en lija”, se

palpa mejor que se ve, desaparece a la presión

Distribución Cefalo-caudal, comienza en la

nuca (centrífugo). Parcialmente confluente

Se inicia en cuello, tórax, extremidades. Al final cara.

Duración 6-8 días 7 días

Prurito No

Palmas y plantas Sí A veces

Resolución Descamación furfurácea, en el mismo orden en que apareció

Descamación foliácea, comienza en cara.

Enantema Máculas Velo del paladar

Otras lesiones

Manchas de Koplik: maculo-pápula blanquecinas sobre halo

eritematoso en la mucosa opuesta a los molares inferiores.

Signo de Pastía: hiperpigmentación pliegues

Triangulo de Filatov (alrededor de la boca sin lesiones)

Lengua frambuesa blanca a roja

Adenopatías

Submaxilares

Otros signos

Esplenomegalia ocasional, Linfopenia absoluta y hallazgos

Rx de adenopatías parahiliares e incluso infiltrados pulmonares.

Diagnóstico Clínico Clínico

Cultivo faríngeo o Test Packs

Tratamiento

Sintomático y aislamiento hasta 5 días después de desaparecer la

erupción. Gamaglobulina IM 0,02 ml/kg.

Indicada en lactantes expuestos no vacunados, pacientes con

enfermedad crónica e ID

Sintomático PNC IM: - <30Kg: 600.000 U/Kg ->30Kg: 1.200.000 U/Kg Tto contactos cultivo (+)

Complicaciones OMA, Neumonía, encefalitis Fiebre reumática, Glomerulonefritis

Page 68: Temas de examen Pediatria

3° Enfermedad 4° Enfermedad

Rubeola Kawasaki o Sd mucocutáneo ganglionar

Edad 3-10 años 3 meses – 5 años

Etiología Togavirus - ARN ¿Superantigenos? ¿Coronavirus?

Mecanismo de infección

Respiratoria

Es una vasculitis sistémica. Se plantea que puede ser 2° a daño inmunitario del endotelio mediado por superantígenos,

inducido por agentes infecciosos como el Coronavirus.

Contagiosidad 7 días antes del exantema

hasta 7 días después NO

Incubación 2-3 sem.

Asintomático 50%

Pródromo

1-3 días: cuadro catarral leve

Fiebre ++/+++, CEG, adenopatías dolorosas

retro-auriculares, cervicales posteriores y

postoccipitales.

Inespecíficos Fiebre +++/+++ de al menos 5 días de

evolución con mal estado general.

Exan

tem

a

Tipo Maculo-papulares,

morbiliforme

Exantema polimorfo y cambiante (maculopapular, morbiliforme,

urticariforme)

Distribución En cara primero y se extiende (centrifugo) Confluente en cara

Extremidades, tronco, cara

Duración 3 días

Prurito No

Palmas/plantas Sí (eritema)

Resolución Descamación leve Descamación leve

Enantema Forchheimer: rosado

blanquecino Estomatitis

Otros signos

Esplenomegalia Leucopenia,

Trombocitopenia, Linfocitos atípicos.

Fiebre elevada > o igual 5 días

Conjuntivitis bilateral no purulenta

Inyección faríngea;

Labios secos con fisuras y/o inyectados

Lengua “en fresa”

Edema, eritema en manos o pies,

Descamación de inicio periungueal

Linfadenopatía aguda cervical unilateral

Leucocitosis con desviación izquierda, anemia, VHS↑, PCR↑, trombocitosis. Pruebas hepáticas ligeramente ↑.

Diagnóstico Clínico - Laboratorio Clínico (cumplimiento de criterios)

Tratamiento Sintomático AAS + Ig

Complicaciones Artritis de pequeñas

articulaciones, encefalitis, púrpura trombocitopénica.

Vasculitis coronaria en las 2 primeras semanas, con posterior formación de aneurismas en «cuentas de rosario»

Page 69: Temas de examen Pediatria

5° Enfermedad 6° Enfermedad

Eritema Infeccioso o megaloeritema

Exantema súbito o Fiebre de los 3 días o Roséola infantil

Edad Escolar (5-15 años) Lactantes (<2años)

Etiología Parvovirus B19 - ADN HV tipo 6 – ADN A veces HV tipo7

Mecanismo de infección

Secreciones respiratoria y vía transplacentaria

Secreciones respiratoria

Incubación 1-2 sem. 1-2 sem.

Pródromo Ocasionales 3 días de fiebre alta sin foco, que

cesa con el exantema. Se presenta con buen estado general

Exan

tem

a

Tipo

3 etapas: 1º. Etapa: “del bofetón”:

eritema de ambas mejillas de inicio brusco.

2º. Etapa: maculo-papular, eritematosa.

3º. Etapa: aclaramiento central de las lesiones, dando aspecto de encaje o reticulado.

Se exacerba por cambios de T°, ejercicio, fricción, luz, stress

Maculo-papular, rosado pálido, poco confluente, granular

(roseoliforme)

Distribución

1º. Etapa: cara, ambas mejillas. 2º. Etapa: tronco, extremidades

en superficies de extensión 3º. Etapa: tronco, extremidades

Distribución simétrica

Tronco, cuello, y región retro-auricular, rara vez se generaliza

Duración 5-10 días 2-3 días

Prurito No No

Palmas y plantas

No

Resolución

Enantema No

Otras lesiones

Otros signos Artralgias

Adenopatías occipitales

Hemograma con Leucocitosis con neutrofilia (pese a ser un cuadro

viral). Después de 48 horas se torna con patrón viral, con

linfocitosis

Diagnóstico Clínico Clínico

Tratamiento Sintomático Sintomático

Complicaciones

Artritis y artralgias de grandes y pequeñas articulaciones.

Si presenta anemia hemolítica constitucional, pueda causar

crisis aplásica grave.

Encefalitis

Page 70: Temas de examen Pediatria

VARICELA Exantema por Enterovirus

Edad 5-10 años Cualquier edad

Etiología Varicela Zoster Enterovirus

Mecanismo de infección

Secreciones respiratoria Fecal-oral y por secreciones

respiratorias

Incubación 2-3 sem. 1 sem.

Asintomáticos

Pródromo 2-4 días: fiebre, cefalea, tos,

CEG. Ausentes o breves

Exan

tem

a

Tipo

Polimorfo: coexisten lesiones en distinto estado

evolutivo “en cielo estrellado” (macula, eritematosa, pápula,

vesícula, pústula y costras).

Exantema variable: Rubeoliforme, escalatiniforme, vesicular, urticarial petequial,

pustular

Sd pie-mano-boca (Coxsackie): lesiones vesiculo-papulosas en

mano, pie y boca.

Distribución Centrípeta (más en tronco

que extremidades) Mano, pie y otras localizaciones

Duración 5-7 días

Prurito +++ /+++

Palmas y plantas

Resolución Generalmente no deja

cicatrices.

Enantema Sí

Otras lesiones

Otros signos

Diagnóstico Clínico Clínico

Tratamiento

Sintomático, No usar ibuprofeno,

Aciclovir: 80 mg/kg/días c/6hrs por 5 días (en ID, si presenta complicaciones)

Sintomático

Complicaciones Impetiginización, Sd Reye,

Ataxia, Meningitis, Sd. Guillain-Barré, Neumonía

Profilaxis

Inmunoglobulina anti- varicela zóster: en las primeras 72 hrs post-

exposición.

Page 71: Temas de examen Pediatria

16. INFECCIÓN RESPIRATORIA ALTA

HERRERA O., QUEZADA A. Enfermedades respiratorias en pediatría, edición 2012

SÁNCHEZ I., PRADO F. Enfoque clínico de las Enfermedades Respiratorias del niño, PUC, 2° edición 2010

MINSAL. Guía Clínica AUGE: Infección Respiratoria Aguda Baja, de manejo ambulatorio, en menores de 5 años. 1° y 2° edición, 2005 - 2013.

OMA

ETIOLOGÍA

• Bacteriana principalmente : Streptococcus pneumoniae, Haemophilus sp, y Moraxella catarrhalis, y

• Se asocia a etiología viral sólo 1/3 de los casos.

DIAGNÓSTICO

Clínico: síntomas de inicio agudo + otoscopia con signos de derrame + signos y síntomas de inflamación del oído medio. • Síntomas más frecuentes:

• Fiebre e irritabilidad (puede ser la única manifestación en el lactante) • Signos de derrame en el oído medio:

• Abombamiento en la membrana timpánica (MT) • Movilidad limitada o ausente de la MT a la otoscopia neumática • Niveles hidroaéreos detrás del tímpano • Otorrea

• Signos y síntomas de la inflamación del oído medio: • Eritema de la membrana timpánica • Otalgia

TRATAMIENTO

Medidas Generales:

Reposo mientras dure la fiebre, Calor local, no taponar el CAE.

Control de fiebre, si T°> 38,5ºC rectal o 38ºC axilar (Paracetamol)

Aliviar el dolor, se puede utilizar: • Paracetamol 10-15 mg/kg/dosis, máximo c/6 horas, en caso de dolor, • Ibuprofeno 10mg/kg/dosis

ATB

1° línea • Amoxicilina: 80-90 mg/kg/día, c/12 hrs (APP 2013) • Amoxicilina: 75-100 mg/kg/día, c/12 hrs (MINSAL)

2° línea fiebre> 39°C,

H. influenzae y

M. catarrhalis B-lactamasas positivos

• Amoxicilina + Acido Clavulanico c/12 hrs. (amoxi 90 mg/kg/día + ac.clav.6,4mg/kg/día)

• En caso de alergia: Ceftriaxona

Page 72: Temas de examen Pediatria

3° línea y en caso de hipersensibilad no tipo I a la Amoxicilina

• Cefuroximo 30 mg/kg/día c/12 hrs (alto costo)

Hipersensibilidad tipo I a la amoxicilina

• Azitromicina 10 mg/kg/día por 1 vez, luego 5 mg/kg/día c/24 hrs x 4 días

• Claritromcina 15 mg/kg/día c/12 hrs

Duración de tratamiento ATB

Menores de 2 años o enfermedad severa: 10 días

6 años o más con enfermedad leve a moderada: 5 a 7 días

La observación sin uso de ATB en niños con OMA no complicada, es una opción un grupo limitado de pacientes, basado en un diagnóstico certero, edad, severidad de la enfermedad y un seguimiento asegurado.

SEGUIMIENTO

• Control a las 48 horas de tratamiento ATB • Enviar a Otorrino frente a:

• 3 o más episodios en un año, • otorrea persistente (más de 15 días), • hipoacusia persistente por más de 2 semanas.

• Enviar a Servicio de Urgencia en caso de: sospecha de mastoiditis o meningitis.

Page 73: Temas de examen Pediatria

SINU S IT IS

ETIOLOGÍA

• Bacteriana principalmente: Streptococcus pneumoniae, Haempophilus influenzae, Moxarella catarralis.

DIAGNÓSTICO

• Es fundamentalmente clínico: • Resfrio persistente con tos >10 días, • Algia facial, por al menos 3 -4 días consecutivos • Edema palpebral, Halitosis • Fiebre alta (> 39ºc), cefalea • Incremento de la descarga nasal purulenta, en cuadro de IRA que iba

inicialmente mejorando luego de 5-6 días • Al examen se puede observar:

• Descarga posterior • Cefalea, que aumenta con valsalva, decúbito, frío. • Cefalea Interocular o retro-ocular, frontal. • Hiponasalidad (rinolalia cerrada)

• La radiografía de senos paranasales no es específica, pero en > 1año, lo

elementos sugerentes son: opacificación completa, engrosamiento de mucosa mayor a 3 mm y presencia de nivel hidroaéreo.

• Se considera Sinusitis Aguda la que dura <4 sem. y crónica >12 sem.

*2 criterios mayores o 1 criterio mayor y 2 menores hacen el dg.

Criterios mayores: Criterios menores:

Dolor o sensación de presión facial

Congestión facial

Obstrucción nasal

Rinorrea, que puede ser purulenta, o descarga posterior

Hiposmia/anosmia

Pus en la cavidad nasal en el examen

Fiebre, sólo en la rinosinusitis aguda

Cefalea

Fiebre, en todas las no agudas

Halitosis

Decaimiento

Dolor dental

Tos

Otalgia

Consideración: el seno frontal empieza a desarrollarse a los 5-6 años y el maxilar, sólo completa su desarrollo a los 7-12 años, por lo tanto las sinusitis no son frecuentes en lactantes.

TRATAMIENTO

• Amoxicilina 75-100 mg/K/día cada 8 hrs. Por 10 a 14 días. • Amoxicilina-ac. Clavulanico 50 mg/k/día cada 8 hrs.

SEGUIMIENTO

• Control a las 48 horas de tratamiento ATB. Derivar a otorrino si presenta más de 3 cuadros al año o se hace crónica.

Page 74: Temas de examen Pediatria

FARIN GOAM IG DA LIT I S

ETIOLOGÍA

>3 años: Streptococcus betahemolítico Grupo A (agente más frecuente)

<3 años: Virus son más frecuentes, se caracterizan por presentar cuadros clínicos más generalizados. Los más frecuentes son: VEB (da exudado blanquecino), Herpes (provoca gingivoestomatis, por lo tanto ulcera en toda la boca no solo en la garganta, muy dolorosas e inclusive mal olor, asociada a fiebre alta), Enterovirus (produce herpangina, que se caracteriza por fiebre muy alta y vesículas en el paladar, sin úlcera), ADV.

DIAGNÓSTICO

Es principalmente clínico. El Score de Wald fue diseñado para orientar a una etiología estreptocócica (los parámetros con * forman parte del éste score, si tiene los 6 items (+) se considera un VPP 75%)

Presentación Clínica

• Edad 5 a 15 años* • Mayo a Noviembre* • T° >38.3ºC axilar* • Adenitis submaxilar* • Faringitis (eritema)* • Ausencia de catarro*

• Saburra blanquecina • Hiperemia faucial • Petequias palatinas • Exudado purulento en amígdalas no adherente • Adenopatías submaxilares sensibles.

Frente a duda etiológica se puede realizar

ANF: Frotis faríngeo para cultivo bacteriológico (S: 90-95% )

TEST PACK (ó Test de diagnóstico rápido para Streptococcus) por técnica de ELISA (S:79-90% )

TRATAMIENTO

Medidas generales:

• Reposo mientras dure el período febril, ingesta de líquidos y alimentos según tolerancia.

Medicamentos:

Paracetamol, en caso de fiebre sobre 38,5ºC rectal o 38ºC axilar.

Penicilina Benzatina: CONTRAINDICADA EN EL MENOR DE 4 AÑOS. • Menos de 27 kilos: 600.000 U IM por 1 vez • Más de 27 kilos: 1.200.000 U IM por 1 vez

Alternativa: Amoxicilina 90 mg/kg/día, dividida cada 8 o 12 horas,por 7 días

En caso de alergia a Penicilina o en el menor de 4 años: • Eritromicina 50 mg/kg/día c/6 por 10 días. Se puede usar también

Claritromicina.

SEGUIMIENTO

• Control a las 48 horas de tratamiento ATB

Page 75: Temas de examen Pediatria

• Derivar a Otorrino si ha presentado: >5 amigdalitis/año durante 3 años seguidos, o hipertrofia de tal magnitud que genera apneas obstructivas.

Page 76: Temas de examen Pediatria

LARIN G ITI S A GUD A

ETIOLOGÍA

• Virus Parainfluenza (es el más frecuente), le sigue VRS y ADV • Otras causas menos frecuentes: alergias (edema angio-neurótico), agentes

físicos (gases o líquidos calientes), agentes químicos (cáusticos, irritantes).

DIAGNÓSTICO

Es clínico, pero no olvidar hacer el diagnóstico diferencial con otras causa de obstrucción de la vía aérea superior.

Presentación clínica sugerente

• Síntomas clásicos: Tos perruna + disfonía + estridor, de mayor intensidad en la noche.

• Edad del paciente entre 6 meses y 5 años • Cuadro catarral previo • No hay salivación aumentada, y el niño es capaz de tomar líquidos • Fiebre baja (<38,5°C)

Clasificación

Grado I Grado II Grado III Grado IV

Disfonía (tos y voz)

Sí Fase de Agotamiento

• Disfonía, estridor, tiraje intenso;

• Palidez, somnolencia, cianosis;

• Aparente disminución de la dificultad respiratoria.

Estridor

Inspiratorio leve e intermitente,

Se acentúa con el esfuerzo (llanto).

Inspiratorio continuo

Inspiratorio y espiratorio

Tiraje (Retracción)

(-) Leve

+/+++ Intenso ++/+++

Otros

Hipoxemia (palidez,

inquietud, sudoración, polipnea)

TRATAMIENTO

Enfrentamiento Inicial

• Lo principal es NO MOLESTAR, diferir procedimientos que puedan empeorar el cuadro respiratorio (no colocar bajo lengua, no tomar muestra de sangre, no tomar Rx de tórax, etc)

• Evaluar severidad del cuadro respiratorio (criterios ABC)

TRATAMIENTO

GRADO I

• Observación. Manejo ambulatorio. Ambiente húmedo y frio

Page 77: Temas de examen Pediatria

• Se recomienda uso de Dexametasona 0,4 mg/kg IM, EV, VO si existe el antecedente de laringitis graves previas, o consultas previas en el mismo episodio. Si no hay Dexametasona, utilizar Prednisona 2 mg/kg/día por 1–3 días.

• Antipiréticos en caso necesario. • Indicación de volver a consultar en caso de progresión de síntomas hacia grado

2 o 3.

GRADO II

• NBZ c/adrenalina racémica al 2,25%: • Dosis 0.05 ml/kg/dosis, con SF para completa 4 cc dosis (mín 0.25 ml y

máx 1 ml). • Durante 10 min. con flujo de 8 lt/min. • Se puede repetir c/20min. Por un máx. de 3 veces. • Observar durante 2 hrs después de la última NBZ por probable efecto

rebote. • Alternativa: NBZ con adrenalina corriente (1/1000), 0.5-0.9 mg/kg/dosis. En

lactantes, se recomienda usar 2 ml de adrenalina en 2 ml SF. • Corticoides (preferentemente EV):

• Dexametasona 0,15 – 0,3 mg/kg/dosis IM, EV, VO (Amp: 10 mg/2 ml) • Betametasona 0,4 mg/kg/dosis IM, EV (Amp: 4mg/ml) • Prednisona 2 mg/kg/día c/12hrs VO por 2 – 3 días

• Hospitalizar si el paciente empeora o no mejora al cabo de 2 hrs de observación post tratamiento

Si en 24-48 horas no hay mejoría clínica se debe realizar endoscopia de la vía aérea para descartar lesiones concomitantes (cuerpo extraño, hemangioma, traqueítis, etc.)

GRADO III

• Hospitalización. • Aplicar medidas de grado 2 si hay demora en el traslado. • No utilizar por horario la adrenalina, sino que según respuesta clínica y

monitorizar efectos adversos. • O2 para SatO2>93%

GRADO IV

• Hospitalización inmediata • Traslado con oxígeno e idealmente intubado. En lugares apartados, y ante la imposibilidad de intubación, puede intentarse cricotirotomía.

SEGUIMIENTO

Page 78: Temas de examen Pediatria

17. ESTRIDOR EN EL LACTANTE

HERRERA O., QUEZADA A. Enfermedades respiratorias en pediatría, edición 2012

SÁNCHEZ I., PRADO F. Enfoque clínico de las Enfermedades Respiratorias del niño, PUC, 2° edición 2010

MINSAL. Guía Clínica AUGE: Infección Respiratoria Aguda Baja, de manejo ambulatorio, en menores de 5 años. 1° y 2° edición, 2005 - 2013.

DEFINICIÓN

Estridor: signo de obstrucción de la vía aérea extratorácica producido cuando el flujo a través de la vía aérea es alterado por una obstrucción parcial o completa.

DIAGNÓSTICO DIFERENCIAL

Supraglóticas

Estridor al inicio de la inspiración y es baja tonalidad

Infe

ccio

sas

Epiglotitis:

Es una emergencia médica, que ha ↓ por el uso de la vacuna anti- Haemophilus influenzae.

Clínicamente hay un compromiso importante del estado general, fiebre alta, odinofagia, ↑salivación (sialorrea}, disfagia, cambios en la voz, más retracción supraesternal.

Absceso retrofaríngeo

Absceso periamigdaliano (en etapas avanzadas da estridor)

Sd mononucleosico

Hiperplasia adenotonsilar

Difteria: Sialorrea

No

infe

ccio

sa

Cuerpo extraño: inicio brusco, asociado a episodio asfíctico

Laringomalasia: anomalía laríngea congénita más común, es la principal causa de estridor en el RN y lactante menor. ↑ de intensidad desde las 2 sem de vida hasta los 6 meses, desde cuando empieza a ↓ . A los 18 meses un gran porcentaje no presenta estridor. Se atenúa con la posición prono y extensión cervical.

Agiedema: inicio brusco, asociado a edema de cara, bucal, secundario a picada de insecto, ingesta de fármaco, alimento, etc.

Ingestión de cáusticos

Neoplasias, laringocele (congénito o secundario a intubación)

Atresia de coanas,

Glótica o Subaglóticas

Estridor que ocupa toda la inspiración o es bifásico. Su tonalidad es alta

Infe

ccio

sas

Traqueítis (o laringo-traqueo-bronquitis bacteriana)

Se presenta entre los 3 meses de vida a los 12 años. Se caracteriza por compromiso difuso de vía aérea, con producción de exudado pseudomembranoso, adherente, que provoca obstrucción progresiva.

Page 79: Temas de examen Pediatria

Existe antecedente de IRA alta de varios días de evolución, que abruptamente muestra un deterioro clínico con fiebre alta, estridor severo y dificultad respiratoria. Puede dar un cuadro muy grave o manifestarse como un Croup de evolución tórpida.

Etiología: Moxarella catarralis, Staphylococcus aureus, St. Pneumoniae.

Laringo-traqueo-bronquitis aguda (Croup):

Se presenta en niños de 6 meses a 5 años.

Se asocia a disfonía, tos perruna, cuadro de resfrío 2-4 días previos. Síntomas se agravan en la noche.

Etiología más frecuente: Parainfluenza 1 y 3, VRS, ADV y otros.

No

infe

ccio

sa

Cuerpo extraño

Laringoespasmo: se asocia a instrumentalización de la vía aérea, se produce espasmo en aducción de las cuerdas vocales. Otra causa es por hipocalcemia (pero es raro)

Compresión extrínseca de la tráquea, por anillo vascular, adenopatías de origen tumoral, masas mediastínicas, higroma quístico, etc.

Hemangiomas, Papilomas

Croup espasmódico (alérgico): se diferencia del Croup por no estar asociado a cuadro viral previo. El estridor es de corta duración y recurrente. Existirían factores alérgico y psicológicos asociados. Se maneja como un Croup.

Parálisis bilateral de las cuerdas vocales: se asocia a hidrocefalia y TEC

Banda laríngea

La presencia fiebre orienta a causas infecciosas

CLASIFICACIÓN

Clasificación de Westley

Ptje Estridor Retracción Entrada de

aire Cianosis (SatO2<92%,

FiO2 0,21)

0 Ausente Ausente Normal Ausente

1 En reposo, audible con

estetoscopio Leve

↓, pero audible

2 En reposo, audible sin

estetoscopio Moderada

Muy ↓, poco

audible

3 Severa

4 Con la agitación

5 En reposo

Puntaje 0-1: croup leve; 2-7: croup moderada; 8o+: croup severo

Clasificación

Grado I Grado II Grado III Grado IV

Disfonía (tos y voz)

Sí Fase de Agotamiento

• Disfonía, estridor, tiraje intenso;

• Palidez, somnolencia,

Estridor

Inspiratorio leve e intermitente,

Se acentúa con el esfuerzo (llanto).

Inspiratorio continuo

Inspiratorio y espiratorio

Tiraje (Retracción)

(-) Leve

+/+++ Intenso ++/+++

Page 80: Temas de examen Pediatria

Otros

Hipoxemia (palidez,

inquietud, sudoración, polipnea)

cianosis; • Aparente

disminución de la dificultad respiratoria.

TRATAMIENTO

Enfrentamiento Inicial

1. Lo principal es NO MOLESTAR, diferir procedimientos que puedan empeorar el cuadro respiratorio (no colocar bajo lengua, no tomar muestra de sangre, no tomar Rx de tórax, etc)

2. Evaluar severidad del cuadro respiratorio (criterios ABC)

3. La sospecha diagnóstica debe orientarse a diferenciar si la causa del estridor es por Croup o por otra causa de obstrucción de la vía aérea superior.

Cuadro que sugiere Croup: • Edad del paciente entre 6 meses y 5 años • Cuadro catarral previo • No hay salivación aumentada • Capaz de tomar líquidos • Fiebre baja (<38,5°C) • Estridor agudo, disfonía.

GRADO I

• Observación. • Manejo ambulatorio. • Se recomienda uso de Dexametasona 0,4 mg/kg IM, EV, VO si existe el

antecedente de laringitis graves previas, o consultas previas en el mismo episodio. Si no hay Dexametasona, utilizar Prednisona 2 mg/kg/día por 1–3 días.

• Antipiréticos en caso necesario. • Indicación de volver a consultar en caso de progresión de síntomas hacia grado

2 o 3.

GRADO II

• NBZ c/adrenalina racémica al 2,25%: • Dosis 0.05 ml/kg/dosis, con SF para completa 4 cc dosis (mín 0.25 ml y

máx 1 ml). • Durante 10 min. con flujo de 8 lt/min. • Se puede repetir c/20min. Por un máx. de 3 veces. • Observar durante 2 hrs después de la última NBZ por probable efecto

rebote. • Alternativa: NBZ con adrenalina corriente (1/1000), 0.5-0.9 mg/kg/dosis. En

lactantes, se recomienda usar 2 ml de adrenalina en 2 ml SF. • Corticoides (preferentemente EV):

• Dexametasona 0,15 – 0,3 mg/kg/dosis IM, EV, VO (Amp: 10 mg/2 ml) • Betametasona 0,4 mg/kg/dosis IM, EV (Amp: 4mg/ml) • Prednisona 2 mg/kg/día c/12hrs VO por 2 – 3 días

Page 81: Temas de examen Pediatria

• Hospitalizar si el paciente empeora o no mejora al cabo de 2 hrs de observación post tratamiento

Si en 24-48 horas no hay mejoría clínica se debe realizar endoscopia de la vía aérea para descartar lesiones concomitantes (cuerpo extraño, hemangioma, traqueítis, etc.)

GRADO III

• Hospitalización. • Aplicar medidas de grado 2 si hay demora en el traslado. • No utilizar por horario la adrenalina, sino que según respuesta clínica y

monitorizar efectos adversos. • O2 para SatO2>93%

GRADO IV

• Hospitalización inmediata • Traslado con oxígeno e idealmente intubado. • En lugares apartados, y ante la imposibilidad de intubación, puede intentarse

cricotirotomía.

DERIVACIÓN

• Si el paciente empeora o no mejora a las 2 horas post-tratamiento (persistencia

de estridor en reposo o estridor) • Si existen dificultades de acceso, visitas repetidas en 24 horas, o presentación

atípica (edad fuera del rango habitual o fuera de temporada)

Page 82: Temas de examen Pediatria

18. SIBILANCIAS RECURRENTES EN EL NIÑO

HERRERA, QUEZADA. Enfermedades respiratorias en pediatría, edición 2012

Apunte de clase de SBO, realizada por Dra. Betancourt

MINSAL. Guía Clínica AUGE: Infección Respiratoria Aguda Baja, de manejo ambulatorio, en menores de 5 años. 2° edición, 2013.

DEFINICIÓN

Sibilancias: se definen como sonidos musicales, de tonalidad alta, generados por el flujo de aire a través de una vía aérea intratorácica estrecha. Sibilancias recurrentes: 3 o más episodios de sibilancias antes del 3° año de vida. Esta definición permite un enfrentamiento inicial diagnóstico y terapéutico.

Existen 3 fenotipos de niños sibilantes

Sibilancias tempranas transitorias

60% (o sibilantes

transitorios precoces)

• Sibilancias empiezan durante el 1° año de vida, luego desaparecen. No se asocian a atopia.

• Se asocia a prematuros, padres fumadores y a un menor calibre de la vía aérea.

• Existe una función pulmonar disminuida al nacer que mejora con el tiempo.

• Predomina en el sexo masculino • IgE normal

Sibilancias persistentes de inicio temprano

20% (o sibilantes no

atópico)

• Sibilancias empiezan durante el 1° año de vida asociadas a infecciones respiratorias virales.

• Pueden persistir hasta la edad escolar, en relación a infecciones virales.

• Existe una función pulmonar normal • Tienden a desaparecer en la pre-adolescencia. • No tiene predilección por sexo. • IgE normal

Sibilancias tardías 20%

(o sibilante atópico asmático):

• Episodios frecuentes de sibilancia (>1 al mes), tos y dificultad respiratoria en relación a dificultad física, tos nocturna sin infección viral,

• Persisten en adolescentes y adultos. • Se asocian a antecedentes personales o familiares

de atopia. • Existe una función pulmonar normal al nacer que

disminuye con el tiempo. • Predomina en el sexo masculino

Page 83: Temas de examen Pediatria

DIAGNÓSTICO DIFERENCIAL

Signos de sospecha

Displasia broncopulmonar Prematuros<32 semanas, SDR, ventilación mecánica neonatal, requerimiento de O2 por más de 20 días.

Disquinesia ciliar Patología sinusal, OMA recurrente, bronquiectasia, situs inversus.

Fibrosis Quística

Mal incremento ponderal, sd de malabsorción, neumopatías, sinusitis recurrentes.

Secuelas crónicas por la agresión viral (bronquiectasias, bronquiolitis obliterante)

Obstrucción bronquial permanente y severa sin respuesta a broncodilatador o corticoide, después de una infección grave por ADV.

Cardiopatías congénitas (producen sibilancias por compresión extrínseca de los vasos del mediastino sobre los bronquios principales, o como manifestación de edema alveolo-intersticial crónico)

Presencia de soplo, hepatomegalia o cardiomegalia

Cuerpo extraño de la vía aérea Episodio agudo de asfixia y signos pulmonares asimetricos

Malformaciones pulmonares Hallazgo radiológico

Malformaciones vasculares Estridor

Trastornos de la deglución por daño neurológico

Neumopatía recurrente o persistente

Reflujo Gastroesofágico Vómitos recurrentes

Compresiones extrínsecas (adenopatías, quistes, tumores, anillos vasculares)

Sibilancias localizadas, unilaterales y persistente.

Anillo vascular Se inician antes del mes de vida

TRATAMIENTO EN EL NIVEL PRIMARIO

Manejo de crisis. Episodio Agudo de SBO

• El tratamiento inicial debe realizarse en APS y dependerá del puntaje de severidad asignado.

• El grado de severidad se calcula según el Score de Tal.

Manejo del niño sibilante recurrente

• Según las manifestaciones clínicas durante las crisis (exacerbaciones) y los periodos intercrisis, se clasifican en distintos grados de severidad, para los cuales se sugiere un tratamiento determinado.

Page 84: Temas de examen Pediatria

MANEJO DE CRISIS. EPISODIO AGUDO

Puntaje Clínico Basado en Score de TAL Modificado

Ptje FR* Sibilancias Cianosis FiO2 Retracciones

<6m >6m

0 <40 <30 No No 21% No

1 41-55

31-45 Fin espiración con fonendo

Perioral al llanto 21-28%

(+)

2 56-70

46-60 Inspir. y espir. con fonendo

Perioral al reposo (FiO2 30%) +

28-35%

(++)

3 >70 >60 Audibles a

distancia ++

Generalizada en reposo (FiO2 35%)

+

>35%

(+++)

* FR/min. RN: normal 50 resp por minuto/ Lactante: 30-40 resp por minuto ++ Si no hay sibilancias por insuficiente entrada de aire debido a obstrucción severa, debe anotarse puntaje 3 + Modificación hecha en Hospital Clínico de la P.U.C.CH.

Leve: 0-5 Moderado: 6-8 Severo: 9-12

Crisis Leve (Ptje ≤ 5/ SatO2≥ 93%)

Manejo • Se indican medidas generales y B2-adrenergico de acción corta, para manejo en Domicilio.

• Y se cita para control al día siguiente.

Medidas generales:

• Posición semisentada, • Alimentación fraccionada • Aseo nasalfrecuente • Control de T° (Paracetamol 15mg/kg/dosis máx. cada 6 horas)

Medidas Específicas

• Salbutamol 2puff c/4-6hrs por 5-7 días. Aplicar aerocámara sobre boca y nariz, con el niño sentado. Administrar 1 puff; luego, esperar 10-15seg. Retirar la aerocámara, esperar 1 minuto y repetir la operación. No importa si el niño llora.

• KNT Respiratoria

Page 85: Temas de examen Pediatria

Crisis Moderada (Ptje 6-8 / SatO2 91-93%)

Manejo • Hospitalización abreviada

Medidas generales:

• Posición semisentada, • Alimentación fraccionada • Aseo nasal frecuente • Control de T° (Paracetamol)

• Oxigenoterapia: obtener una SaO2 93- 95%.

Medidas Específicas

• Salbutamol 2puff c/10 minutos por 5 veces y re-evaluar • Su puntaje es ≤5: se maneja como crisis leve • Si puntaje es 6-8: agregar corticoides sistémicos Hidrocortisona 10mg/kg EV Prednisona 2mg/kg VO • Si responde bien a corticoide (ptaje≤ 5), se indica

tratamiento ambulatorio con Salbutamol 2puff c/4-6hrs + Prednisona 1-2 mg/kg/día, dividida c/12 hrs por 5 días (máx:40mg/día). Y se cita para control al día siguiente.

Crisis Severa (Ptje 9-12 / SatO2 ≤ 90%)

Manejo • Hospitalizar • Otras indicaciones de hospitalización son:

• Apneas, • Compromiso de conciencia, • Condición social adversa y • <3meses

Medidas generales:

• Posición semisentada, • Alimentación fraccionada • Aseo nasal frecuente • Control de T° (Paracetamol)

• Oxigenoterapia: obtener una SaO2 93- 95%.

Medidas Específicas

• NBZ con Salbutamol al 0,5% (lactante menor 0,5cc, lactante mayor y escolar 1cc; más SF para completar 4cc) cada 20 minutos por 3 veces y continuando cada 4-6 horas si responde favorablemente.

• Corticoides sistémicos por 5- 7 días (preferir VO) Prednisona 2mg/kg VO c/12 hrs (máx:40mg/día) Hidrocortisona 10mg/kg EV Dexametasona 0,3 mg/kg/dosis EV

• Se recomienda Bromuro de Ipatropio asociado al SBT en crisis moderadas a severas.

Criterios de Alta

• < 6 meses: que pase más de 24 horas sin O2 suplementario. • > 6 meses: 12 horas sin necesidad de O2 suplementario • Padres confiables

Page 86: Temas de examen Pediatria

En lactantes menores de 12 meses que cursan con primer cuadro de sibilancias, de moderado a severa gravedad, se sugiere utilizar Adrenalina Racémica en lugar de SBT. Dosis 0.05 ml/kg/dosis (mín 0.25 ml y máx 1 ml).

MANEJO DEL SIBILANTE RECURRENTE

Se realiza según el grado de severidad.

Leve Moderado Grave

Exacerbación Leves < 1 por mes

Moderadas > 1 por mes o síntomas persisten por 1 mes

Graves

Periodo intercrisis

Sin tos nocturna o muy ocasional

Tos nocturna

Despertar nocturno ocasional

Tos o sibilancias diurnas frecuentes

Tos nocturna casi todas las noches

Despertar nocturno frecuente

Tos o sibilancias diurnas casi todos los días.

Dificultad para alimentarse

Consultas de urgencias

No Ocasional Frecuente

Uso de corticoides sistémicos

No Ocasional Frecuente

Hospitalizaciones No Ocasional Sí

Leve:

• Requiere sólo uso de broncodilatadores durante la crisis

Moderada:

• Debe derivarse a especialista para estudio. Descartada causa secundaria se prueba CI en dosis bajas.

• Budesonida 100-200ug/día en 2 dosis • Fluticasona 100-200ug/día en 2 dosis

Severa:

• Manejo exclusivo del especialista, con CI en dosis medianas. • Budesonida 300-400ug/día en 2 dosis • Fluticasona 300-400ug/día en 2 dosis

DERIVACIÓN

A Programa IRA

Niños con 3 o más episodios en 1 año.

A especialista

SBO moderado (para descartar causa secundaria)

Page 87: Temas de examen Pediatria

SBO moderado con mala evolución clínica y /o radiológica, tras haber sido derivado para descartar causa secundaria.

SBO severo

Page 88: Temas de examen Pediatria

19. ASMA EN MENORES DE 15 AÑOS

MINSAL. Guía Clínica AUGE: Asma Bronquial moderada y grave en menores de 15 años. 2° edición, 2011.

Global Strategy for Asthma management and Prevention (GINA) 2010 update

HERRERA, QUEZADA. Enfermedades respiratorias en pediatría, edición 2012

Apunte de clase de Asma, realizada por Dra. Betancourt

Sociedad Argentina de Pediatría. Consenso de Asma Bronquial. 2007. 2ª parte

DEFINICIÓN

Enfermedad inflamatoria crónica de las vías aéreas, caracterizada por obstrucción bronquial a distintos estímulos, total o parcialmente reversible, cuya evolución puede conducir a una modificación estructural de dichas vías (remodelación), provocando una obstrucción bronquial no reversible. Características claves del Asma:

1. Inflamación crónica de las vías aéreas (con edema endoluminal). 2. Obstrucción bronquial parcial o totalmente reversible (de forma

espontánea o con tratamiento) 3. Hiperreactividad bronquial

BASES PARA EL DIAGNÓSTICO

El diagnóstico es fundamentalmente clínico en el menor de 5 años, se basa en un adecuado interrogatorio de los síntomas, antecedentes personales y familiares y un examen físico exhaustivo. En los mayores de 5 años el diagnostico se enriquece al poder incluir pruebas de laboratorio que corroboran la sospecha clínica. El diagnóstico es sugerente cuando en la historia clínica existen:

1. Episodios recurrentes de dificultad respiratoria, tos, sibilancias, que mejoran con el uso de broncodilatadores, pero la falta de respuesta no lo excluye.

2. Tos crónica o recurrente, especialmente en otoño y primavera 3. Síntomas como: sibilancias, tos y disnea, que empeoran en la noche o con la

hiperventilación (llanto, risa, ejercicio) y frecuentemente están asociados a infecciones virales, irritantes, alérgenos y frío.

4. Antecedentes personales de cuadros de atopia (como rinitis alérgica, conjuntivitis alérgica, dermatitis alérgica, etc), pólipos y sinusitis.

5. Antecedentes familiares directo de asma, rinitis alérgica u otra manifestación de alergia.

6. El examen físico en los periodos estables habitualmente es normal. Pero en durante las exacerbaciones son múltiples.

Exacerbaciones Leve -Moderada

Tos,

Sibilancias espiratorias,

Espiración prolongada,

Disminución del MP,

Signos de hiperinsuflación pulmonar (↑diámetro AP del tórax e hipersonoridad a la percusión)

Exacerbaciones graves

Dificultad respiratoria, Quejido,

Aleteo nasal,

Retracciones,

Polipnea,

Dificultad para hablar y alimentarse

Compromiso de conciencia variable y

↓ o ausencia del MP

Page 89: Temas de examen Pediatria

Además pueden encontrarse signos de dermatitis atópica y rinitis alérgica.

EXÁMENES DE APOYO

Espirometría, para evaluar obstrucción al flujo aéreo. Se puede realizar desde los 5 -6 años. Se considera alteración ventilatoria obstructiva a una relación VEF1/CVF disminuida, VEF1 disminuido, FEF 25-75% disminuido y CVF normal. Resultados post-broncodilatador: el ↑12% del VEF1 y el 30% del FEF25-75 se consideran significativo. En un año, un cambio ≥ 15% respecto a su valor teórico previo, en VEF1 y CVF puede considerarse significativo

Un gran porcentaje de los niños, la espirometría basal es normal en su período estable, inclusive es normal en niños con asma persistente. La espirometría post-broncodilatador, cuando evidencia una respuesta positiva, apoya el diagnóstico de asma. En caso de ser negativa, se sugiere realizar exámenes de atopia (test cutáneo) e hiperreactividad bronquial (test de ejercicio, metacolina) para aumentar la probabilidad diagnóstica

Pruebas de provocación bronquial con metacolina: evalúa hiperreactividad bronquial. Tiene ↑S, ↓E, por lo que es más útil para descartar asma (si el resultado es negativo) que para confirmarla (si el resultado es positivo). Una ↓20% del VEF en relación a dosis crecientes de metacolina es altamente sugerente de asma.

Pruebas de provocación bronquial con ejercicio: ↑E, ↓S, por lo que es útil para confirmar asma en niños con examen positivo. La ↓PEF (pick expiratory flow) 15% apoya la sospecha diagnóstica

Radiografía de Tórax: en un paciente asmático es normal, debido a que no existe alteración en el parénquima pulmonar. Sin embargo puede haber signos de Hiperinsuflación o de complicaciones, como neumotórax, neumomediastino o enfisema subcutáneo. Signos de Hiperinsuflación: Aplanamiento de los diafragmas, Horizontalizacón de las costillas, Hipertransparencia pulmonar, aumento de espacios intercostales y Herniación parénquima pulmonar entre los espacios intercostales.

Otras estudios imagenológico: Rx de Cavum y senos paranasales, según el cuadro clínico

Pruebas cutáneas (prick test): se recomienda en >4 años, para que tengan madurez inmunológica. Los alérgenos importantes son los que se inhalan. Los alérgenos alimentarios no son un factor precipitante común de los síntomas del asma. La limitación principal de este examen es que un test positivo no necesariamente indica que su asma es atribuible a la alergia.

IgE: Evaluar sensibilización alérgica

Resultados normales de la espirometría y de las pruebas de provocación, especialmente si se realizan en niños asintomáticos, no excluyen este diagnóstico

Page 90: Temas de examen Pediatria

CONSIDERACIONES DIAGNÓSTICAS

En relación a los menores de 5 años el diagnóstico de asma es difícil de confirmar por lo que suele utilizarse el concepto de Síndrome Bronquial Obstructivo o Sibilancias Recurrentes.

En los menores de 3 años existe un índice clínico predictor de asma (API)

API POSITIVO

Niño <3años con sibilancias frecuentes (>3 episodios/año) +

1 factor de riesgo mayor o 2 menores predicen asma.

Factor de Riesgo Mayor Factor de Riesgo Menores

• Asma en los padres. • Dermatitis atópica

diagnosticada

• Eosinofilia >4% • Sibilancias frecuentes sin

desencadenante viral • Rinitis alérgica

Con un API (+), se puede asegurar con un 77% de certeza que el niño padecerá asma a edad escolar.

*existen estudios que reportan una asociación entre infección grave por VRS en edades tempranas y el desarrollo de asma.

CLASIFICACIÓN

La clasificación habitual más usada del asma se basa en los niveles de severidad del asma, sin embargo la severidad puede cambiar con el tiempo y el tratamiento influye en su presentación y desarrollo. Es por esto que la nueva clasificación se basa en el nivel de control del asma y es útil y práctica para el manejo de ésta enfermedad.

Page 91: Temas de examen Pediatria

EXACERBACIÓN (EA)

Se define como un episodio agudo y progresivo de obstrucción de la vía aérea que se manifiesta por ↑ de la tos, silbido al pecho, dificultad respiratoria o una combinación de los anteriores que presenta distintos grados de severidad. Los desencadenantes frecuentes de EA son las infecciones virales y la exposición a alérgenos.

DIAGNÓSTICO DIFERENCIAL

Obstrucción de la vía aérea inferior central

• Cuerpo extraño de la vía aérea • Disfunción de las cuerdas vocales • Anillo vascular • Laringomalacia, membrana laríngea, estenosis traqueal • Malformaciones pulmonares • Tumores mediatínicos, adenopatías y otras • Adenoma bronquial y granulomas endobronquiales

Obstrucción de la vía aérea inferior periférica

• Bronquiolitis, Bronquiolitis obliterante • Fibrosis Quística • Displasia broncopulmonar • Diskinesia ciliar

Otras causas

• Reflujo Gastroesofágico • Sd de hiperventilación • Coqueluche • Disfunción de cuerdas vocales

Page 92: Temas de examen Pediatria

TRATAMIENTO

Se basa en 4 pilares fundamentales que deben ser considerados en el manejo de cada paciente:

Educación y autocuidado

Control de factores agravantes

Farmacoterapia

Manejo de las exacerbaciones

EDUCACIÓN Y AUTOCUIDADO

Los principales objetivos de la educación son: desarrollar habilidades de autocuidado y mejorar el cumplimiento del tratamiento. Estrategias que han demostrado beneficio en el control del asma: • Programas de salud personalizados, considerando la cultura, la capacidad

intelectual y la etapa del desarrollo de los que reciben la educación. • Educación relacionada con la técnica inhalatoria, • Proporcionar un plan de alta escrito con instrucciones claras sobre la terapia, su

dosificación y tiempo de uso y la fecha de la próxima consulta. • Asma no es sinónimo de certificado médico para evitar que el niño realice

Educación Física. Los pacientes bien controlados y que cumplen las medidas preventivas pueden realizar actividad física de forma prácticamente normal. Se debe restringir la carrera libre y Test de Cooper.

• Las causas de Asma inducida por ejercicio son: cambios de T° en la vía aérea por respiración bucal, Aire seco y congestión de las arterias bronquiales

• En niño con asma por ejercicio indicar un beta2adrenergico unos 15 minutos antes del ejercicio

CONTROL DE FACTORES AGRAVANTES

• Control ambiental: evitar humo de tabaco, alérgenos, irritantes en la casa y escuela, mascotas con pelos o plumas, etc.

• Evitar contacto con personas con infección respiratoria aguda. • En períodos epidémicos no ir a lugares con alta concentración de personas • Cumplir las recomendaciones de los programas de Inmunizaciones

FARMACOTERAPIA

Objetivo: Lograr y mantener el control clínico: síntomas mínimos o ausentes con buena calidad de vida. Los medicamentos para tratar el asma se pueden clasificar en: • Controladores son aquellos que se utilizan a diario y los • Aliviadores según necesidad del paciente para revertir la broncoconstricción y

aliviar síntomas. Para el tratamiento se debe clasificar al paciente según grado de control y otorgar tratamiento escalonado con diferentes opciones terapéuticas en 5 pasos.

Page 93: Temas de examen Pediatria

• Y se debe realizar control mensual para la monitorización y el cambio del tratamiento (si fuera necesario), y una vez controlado visitas cada 3 meses.

PASO 1 – Medicación de rescate Paciente con síntomas ocasionales

• Broncodilatador β 2 -agonista inhalado de acción corta.

• SBT 2 puffs según necesidad • Alternativa bromuro Ipratropio

cuando el SBT está contraindicado.

PASO 2 –

Pcte que usa SBT >3veces/semana y/o exacerbación en los 2 últimos años que haya requerido corticoides sistémicos

• Corticoide inhalado (CI)a bajas dosis • Alternativa: antileucotrienos

Si persiste sin control después de 2-3 meses, antes de pasar al paso 3 revisar:

• ¿Cumple tratamiento según indicaciones (con técnica y aerocámara adecuada)?

• ¿Se han cumplido las indicaciones de control ambiental?

• ¿Es posible derivarlo a un especialista?

PASO 3

• Niños >4 años: combinar el uso de CI a dosis baja/moderada + 2-agonista de acción prolongada (LABA) en un aerosol combinado.

• Ó ↑ CI a dosis a moderada. • Alternativa: Antileucotrienos asociado a CI.

• Niños <4 años: ↑ dosis CI • Alternativa: asociar a CI en dosis baja un

antileucotrieno (beneficio clínico menor)

• Si cualquiera de estas alternativas no demuestra ser efectiva pase al paso sgte.

Page 94: Temas de examen Pediatria

PASO 4 -

• ↑ CI a dosis mediana asociándolo a un LABA ó • en <4años CI a dosis mediana asociándolo a un

antileucotrieno

PASO 5 -

Excepcionalmente, • Si a pesar de los pasos anteriores persisten los

síntomas con exacerbaciones frecuentes y limitación de la actividad física: adicionar esteroide oral

• Niños >6 años con niveles ↑ IgE, reversibilidad funcional demostrada y frecuentes hospitalizaciones: adicionar Omalizumab (Ac monoclonal humanizado anti IgE)

• Si al cabo de 3 meses el paciente tiene bien controlada su asma con corticoides inhalados (CI) a dosis mediana o alta, reducir en 50%.

• Si al cabo de 3 meses logra el control con corticoides inhalados a dosis mediana o alta + LABA, alternativamente puede reducir en 50% el CI o suspender el LABA.

• Si el paciente ha estado asintomático (controlado) por 1 año, suspender y continuar con control médico.

Page 95: Temas de examen Pediatria

• La administración en inhalador de dosis medida (IDM) con aerocámara valvulada es equivalente a la NBZ, es más barato y más rápido su uso.

• Los corticoides no poseen efecto broncodilatador, pero mejoran la función pulmonar y disminuyen la hiperreactividad bronquial a largo plazo. Son fármacos preventivos que no producen un cambio en la historia natural del asma.

MANEJO DE LAS EXACERBACIONES

Leve Moderada Severa

Broncodilatador B2- acción corta (SBT) 2puff c/10min x3 veces

Corticoide oral (Prednisona 1-2 mg/K vo. Max 40/día)

O2 para SaO2 > 93%

Broncodilatador B2 acción corta igual EA leve.

Broncodilatador anticolinérgico (BI) en (20 ug/puff) 2-6 puff

Corticoide oral

O2 con mascára no recirculación

Vía venosa, bolo de SF

NBZ con SBT

NBZ con Bromuro de Ipatropio

Metilprednisolona o Hidrocortisona

Reevaluar en 1 hora

Hospitalizar Si SatO2<93%: Hospitalizar Si es >93% ALTA

Indicaciones de alta:

• Plan de acción escrito con signos de alarma • Broncodilatadores cada 4-8 hrs. por 7días • Corticoesteroide oral completar 5-7 días total. • Control médico en 3 a 4 días

DERIVACIÓN

• Pacientes que estando en el paso 4 (CI a dosis mediana + LABA ó CI a dosis mediana + antileucotrieno) no logran controlar su asma siempre que el cumplimiento del tratamiento sea óptimo.

Page 96: Temas de examen Pediatria

Fármacos de uso en el Asma A

nti

-in

flam

ato

rio

s

No

est

ero

idal

Antileucotrienos:

Montelukast (comp:4 –5–10mg)

Zafirlukast (comp:20mg)

Este

roid

al

Inhalados (ug/puff)

Budesonida (200)

Fluticasona (50-125-250)

Beclometasona (50-100)

Mometasona (200-400-800)

RAM:

Candidiasis orofaríngea, la disfonía y, ocasionalmente, la tos por irritación de la vía aérea superior.

Pueden ser prevenidos mediante el uso de los espaciadores y el hábito de enjuagar la boca luego dela inhalación.

Sistémicos

Prednisona (comp: 5-20mg; jarabe:1mg/ml – 4mg/ml)

Hidrocortisona (amp:100-500mg; comp:20mg)

Betametasona (comp:0,5-0,6 mg; amp:4mg; jarabe 0,5mg/ml; gotas:0,5mg/ml)

Metilprednisolona (amp: 40-80mg)

↓ riesgo de hospitalización

Efecto entre 4 - 6horas

Corticoides VO son tan efectiva como EV

Prednisona 2mg/kg (máx.40 mg

Hidrocortisona 4 mg/kg. c/4-6 hrs (máx.400 mg/ dia)

Metil-prednisolona 0.5-1 mg/kg c/4-6 hrs (máx.120 mg diarios).

Bro

nco

-dila

tad

ore

s

Bet

a2-

adre

nér

gico

s

Acción corta

Salbutamol (100ug).efecto a los 5 minutos, y dura 4 horas

Fenoterol (en mezcla) RAM: taquicardia, ↑intervalo QT, arritmias, hipertensión o a veces hipotensión, hipokalemia, temblor y alteración de la relación V/Q

Bet

a2-

adre

nér

gico

s

Acción larga

Salmeterol (en mezcla)

Formoterol (en mezcla)

An

ti-

colin

érgi

cos

Bromuro de Ipatropio (IDM 20ug/puff; NBZ 0,25mg/ml)

Son broncodilatadores menos potentes que los agonistas β2 y el inicio de acción es más tardío.

MEZ

CLA

S

Bro

nco

-d

ilata

do

res Salbutamol + bromuro de

ipratropio (100/20) µg/puff

Fenoterol + bromuro de ipratropio (BERODUAL®) (50 /20) µg/puff

Co

rtic

oid

es

+ B

2 a

cció

n la

rga Fluticasona/salmeterol

(BREXOTIDE®, SERETIDE®)en IDM 50/25; 125/25; 250/25 µg/puff

Budesonida/formoterol en IDM 80/4.5; 160/4.5 µg/puff

Page 97: Temas de examen Pediatria

20. INFLUENZA

MINSAL, “Guía de práctica clínica Prevención, diagnóstico y manejo Clínico de casos de Influenza”, 2013.

Vacuna contra influenza estacional y seguridad, CDC 2013.

DIAGNÓSTICO

Enfermedad Respiratoria tipo influenza (ETI)

Fiebre ≥ a 38,5ºC axilar y

Tos

Más alguno de los siguientes síntomas: Mialgias, Odinofagia y Cefalea

Más contacto con caso ETI en período de circulación viral alta

• Síntomas inespecíficos que comienzan de manera brusca • Fiebre de 38-40ºC con un pick a las 24 horas, • Duración entre 1 y 5 días, • Tos mucosa a purulenta

Confirmación etiológica

• Se efectúa mediante la realización de Inmunofluorescencia (IF). Debe tomarse en cuenta que una IF negativa NO descarta influenza en especial en períodos de alta circulación del virus.

LABORATORIO

TRATAMIENTO AMBULATORIO

Se tratará de forma ambulatoria a paciente que presenten: CLÍNICA compatible para ETI con o sin condición de riesgo.

Condiciones riesgo

1. Edad menor de 2 años, 2. Inmunodepresión, 3. Diabetes, 4. Daño pulmonar crónico, 5. SBOR,

6. Asma, 7. Cardiopatías congénitas, 8. Insuficiencia renal crónica, 9. Enfermedad neuromuscular, 10. Epilepsia.

Manejo de paciente con CLÍNICA compatible con ETI, sin criterios de gravedad y sin condición de riesgo

• Medidas generales: Reposo domiciliario según criterio médico. Hidratación adecuada.

• Tratamiento sintomático: Antipiréticos para el manejo de la fiebre. *el ácido acetilsalicílico está contraindicado en menores de 15 años.

Page 98: Temas de examen Pediatria

• Precauciones estándares: Lavado frecuente de manos, medidas de higiene general, cubrir la boca y nariz con pañuelo desechable al toser.

• Educar para consultar precozmente ante agravamiento de síntomas (criterios de gravedad)

Manejo de paciente con CLÍNICA compatible con ETI, sin criterios de gravedad y CON CONDICIÓN DE RIESGO

• Medidas generales • Tratamiento con antiviral: se debe dar antes de 48 hrs de la aparición de los

síntomas, ya que el efecto se ha demostrado mínimo si es más tardío. • OSELTAMIVIR o ZANAMIVIR

• Precauciones estándares: Lavado frecuente de manos, medidas de higiene general, cubrir la boca y nariz con pañuelo desechable al toser.

• Educar para consultar precozmente ante agravamiento de síntomas (criterios de gravedad)

CRITERIOS DE HOSPITALIZACIÓN

• Todo paciente que presente criterio de gravedad

Criterios de gravedad

1. Hipoxemia: SatO2 <93% con FiO2 ambiental 2. Deshidratación o rechazo alimentario (en lactantes) 3. Dificulta respiratoria o aumento del trabajo respiratorio 4. Compromiso hemodinámico 5. Consulta repetida por deterioro clínico

TRATAMIENTO HOSPITALIZADOS

Manejo de paciente con CLÍNICA compatible con ETI, CON CRITERIOS DE GRAVEDAD

• Tomar IF o test pack si se dispone • Iniciar inmediatamente tratamiento con antiviral • Precauciones estándares: higiene de manos, protección facial y uso de delantal,

mascarilla de tipo quirúrgico en caso de traslado el paciente, preferir el aislamiento de pacientes en pieza individual o mantener la separación de al menos un metro entre pacientes en las salas de atención.

• Notificar IRA Grave, en el “Formulario de Notificación Inmediata y Envío de Muestras a Confirmación de IRA Grave y Fallecidos por Influenza”

ANTIVIRALES

OSELTAMIVIR

• Se debe indicar por 5 días, a dosis adecuada por peso (*a los pacientes con insuficiencia renal y en hemodiálisis se les debe ajustar la dosis).

• RAM más comunes: Nausea , Vómitos, Diarrea y Dolor Abdominal

Page 99: Temas de examen Pediatria

A los 3 años y medio la mediana es de 15 kilos.

*Dosificación de oseltamivir para lactantes prematuros: 1 mg/kg dosis c/12 hrs por 5 días.

Zanamivir (inhalador)

• Dosificación: 2 inhalaciones c/12 hrs por 5 días (5 mg c/12hrs x 5 días) • Puede utilizarse como alternativa cuando el Oseltamivir está contraindicado, en

mayores de 7 años. • Contraindicaciones:

• El efecto adverso más reportado es el broncoespasmo, por lo tanto NO debe ser usado en personas con: Hiperreactividad bronquial, Asma y Enf. bronquial obstructiva crónica

• Niños <7 años para tratamiento y en <5años para quimioprofilaxis • RAM más comunes: Sinusitis, Mareos, Fiebre y/o Calofríos, Artralgia,

Reumatismo articular.

QUIMIOPROFILAXIS

• Debe ser administrada en contactos cercanos que presenten condiciones de riesgo, debiendo ser iniciada antes de 5 días desde el contacto con el caso índice, y consiste en Oseltamivir o Zanamivir (la mitad de la dosis por el doble de tiempo, es decir, cada 24 horas por 10 días)

Contacto cercano se refiere al contacto a menos de 1 mt. por un tiempo de exposición mayor a 15 minutos, con un paciente con sospecha o confirmación de Influenza.

Condiciones de riesgo:

• Embarazada: Vacunada o no, que habita bajo el mismo techo o es contacto cercano de un paciente sospechoso o confirmado con influenza.

• Paciente inmunodeprimido o con terapia inmunosupresora: Vacunado o no que habita bajo el mismo techo o es contacto cercano de un paciente sospechoso o confirmado con influenza.

Page 100: Temas de examen Pediatria

21. NEUMONÍA

MINSAL. Guía Clínica AUGE: Infección Respiratoria Aguda Baja, de manejo ambulatorio, en menores de 5 años. 1° y 2° edición, 2005 - 2013.

HERRERA O., QUEZADA A. Enfermedades respiratorias en pediatría, edición 2012

SÁNCHEZ I., PRADO F. Enfoque clínico de las Enfermedades Respiratorias del niño, PUC, 2° edición 2010

DEFINICIÓN

• Inflamación aguda del parénquima pulmonar que puede comprometer alvéolos, intersticio o ambos, cuya etiología puede ser viral, bacteriana o mixta. Los términos «bronconeumonia» y «neumonitis» no deben utilizarse para denominar entidades clínicas

• Neumonia Adquirida en la comunidad: Ocurre en paciente que no ha estado hospitalizado en los últimos 7 días o que lleva menos de 48hrs hospitalizado

• Neumonía Nosocomial: se presenta después de 48 horas hospitalizado o dentro de los 7 días posteriores al alta.

ETIOLOGÍAS

• RN: SHGB, Gram (-) E.Coli, Listeria monocitogena Staphylococcus aureus (se asocia a la desnutrición primaria), St. Pneumoniae, VRS.

• Lactantes: VRS (es la principal causa), ADV (los serotipos 7h y 2b, pueden ocasionar neumonía grave, con secuelas pulmonares), S. pneumoniae, Parainfluenza, Metaneumovirus (se asocia a neumonías con obstrucción bronquial)

• Preescolar- Escolar: S. pneumoniae (causa más frecuente bacteriana), Mycoplasma pneumoniae (causa más frecuente de neumonía atípica), Chamydia pneumoniae (junto al mycoplasma se relacionan con la recurrencia de broncoespasmo en niños susceptible), Influenza.

Virus emergentes: • Bocavirus, coronavirus y mimivirus. Infecciones mixtas: • Las combinaciones más frecuentes son VRS o influenza con neumococo • La varicela predispone a la infección por S.pyogenes y S. aureus, dando lugar a

neumonías graves.

DIAGNÓSTICO

• Es principalmente clínico y no requiere evaluación radiológica de rutina.

• Los síntomas más frecuente son: tos, fiebre y dificultad respiratoria.

Taquipnea, aleteo y quejido son la triada predictora de neumonía

Page 101: Temas de examen Pediatria

Menores de 3 meses:

• Son inespecíficos, aislados o poco manifiestos • Decaimiento, Rechazo alimentario • Tos, Polipnea • Apnea • Fiebre o hipotermia, Diarrea

Lactante

• Predomina el CEG, Irritabilidad y palidez, Decaimiento, Rechazo alimentario. • Tos, taquipnea, Retracción torácica, Quejido • Disminución del MP, Espiración prolongada, Sibilancias , Crepitaciones • Fiebre y cianosis

• No presenta signos clásicos de condensación pulmonar

Preescolar y escolar

• Decaimiento, Cefalea • Tos con expectoración, Puntada de costado • Presenta signos clásicos de condensación: ↓ MP, matidez, broncofonía, soplo

tubario y crepitaciones. • Fiebre alta, calofríos

Taquipnea (definición OMS) • < 2 meses: > 60 rpm • 2 – 12 meses:> 50 rpm (Normal: 25-40) • 1 – 5 años: > 40 rpm(FR Normal: 20-30) • > 5 años: > 25 rpm(FR Normal: 15-25)

Nota: la taquipnea podría no presentarse en un niño con retracción pronunciada u otros signos de descompensación respiratoria

Bacteriana Viral Mycoplasma

Cualquier edad Menores de 2 años 5-15 años

Todo el año Invierno Todo el año

Inicio súbito Estado tóxico general

Inicio variable Concomitancias con otros casos en la familia.

Inicio insidioso Taquipnea poco frecuente

Fiebre alta Baja cuantía (excepto VRS) Variable

Dolor Torácico* Coriza, Conjuntivitis * Exantemas, artralgias *

Signos de condensación Siempre obstrucción, a veces condensación

Signología obstructiva

Condensación pulmonar Rx: relleno alveolar lobar sementario o multifocal

Sin signos de condensación clínica Rx con compromiso intersticial difuso o peribronquial, atelectasias subsegmentaria. Hiperinsuflación

Leucocitos > 10.000/mm3, con neutrofilia. PCR >30mg%

Leucocitos < 10.000/mm3, con escasa neutrofilia. PCR < 40mg%

*síntomas asociados

Page 102: Temas de examen Pediatria

Características Neumonías por ADV

• Fiebre alta mantenida (>39°C), aspecto séptico • Obstrucción bronquial severa, que conduce a insuficiencia respiratoria. • Compromiso multisistémico: SNC (compromiso conciencia), Renal (síndrome

nefrítico sub-clínico), Piel (exantema maculo – papular), Hepático (hepatitis), Ocular (conjuntivitis hemorrágica).

• Hemograma con leucopenia o leucocitosis, con desviación a la izquierda • PCR variable • Rx tórax con hiperinsuflación, compromiso intersticial, consolidación multi o

unilobar

La edad es el mejor predictor de la etiología

ESTUDIOS COMPLEMENTARIOS

• Rx de tórax, AP y lateral: Utilidad en confirmar diagnóstico, controlar evolución y descartar complicaciones (atelectasia). Consideración: la mejoría clínica es más rápida que la radiológica (hasta por 4 semanas)

• Hemocultivo: Baja positividad (no superior al 20%). • Serología: Útiles en la pesquisa de Mycoplasma pneumoniae y Chlamydia

pneumoniae, cuyo cultivo es dificultoso. Recordar que la IgM tarda 7-10 días en aparecer y hasta 10 meses es negativizarse.

• Detección rápida (<1 hora): Test pack para VRS, Influenza A-B y ADV. • Detección de antígenos por Aspirado nasofaríngeo IFD o IFI:

• S: 80-85% para VRS – VI – VPI- MPV , E: 80-90% • S: 50% para ADV, E: 95%

• Proteína C Reactiva Orientan o coadyuvan a discriminación entre viral o bacteriana. Valor de corte según método, en nuestro medio es 30 a 40 mg.

TRATAMIENTO

Manejo Ambulatorio

• Reposo en cama, semisentado. Evitar exceso de abrigo. • Adecuada ingesta de líquidos, alimentación fraccionada según tolerancia, • Aseo nasal frecuente • Control de T° con medidas físicas (baño o compresas tibias) y/o paracetamol (en

caso de fiebre>38,5ºC) • Broncodilatadores, en caso de obstrucción bronquial:

• SBT 2 puff cada 4-6 horas por 5-7 días • ATB empírico: por ser un cuadro grave, con potencial riesgo vital, en el cual es

difícil aislar o detectar el agente causal. El pronóstico mejora a mayor precocidad del tratamiento.

• Amoxicilina 80-100 mg/kg/día, c/12hrs x7 días (máx.2grs/día) *en >5 años puede prescribirse Amoxicilina 50-80 mg/kg/día, ya que la resistencia microbiana en este grupo etáreo es menor. • Frente a sospecha de neumonía atípica (mycoplasma o chlamydia) o

paciente con antecedente de alergia a betalactámicos, usar macrólidos.

Claritromicina 15mg/kg/día c/12hrs x 10 días (máximo 1 gr/día) o Eritromicina 50mg/kg/día c/6hrs, por 10 días (máximo 2 gr/día) o

Page 103: Temas de examen Pediatria

Azitromicina 10mg/kg/día c/24hrs, separada de los alimentos, x5 días (máximo 500 mg/día).

• Control médico a las 24 horas en el lactante <6 meses y a las 48 horas en el niño mayor.

Manejo Hospitalario

Indicaciones de Hospitalización

1. Menor de 3 meses (riesgo de apneas y paro cardiorespiratorio) 2. Necesidad de oxigenoterapia (SatO2<93% y signos de hipoxemia) 3. Complicaciones:

• Derrame o empiema pleural • Absceso pulmonar: raro, generalmente se piensa en Q. hidatídico. • Septicemia, shock, insuficiencia renal, falla multiorgánica • Otros focos: Artritis, osteomielitis, pericarditis, meningitis peritonitis • Resistencia antibiótica

4. FR > 70 rpm en lactantes, > 50 rpm en niños mayores 5. Neumonía grave o aspecto tóxico desde el ingreso 6. Rechazo alimentario, Deshidratación 7. Enfermedad crónica subyacente 8. Riesgo social 9. Mala respuesta al tratamiento inicial en 48 – 72 hrs

ATB en <3 meses

• Ampicilina 150 mg/kg/día c/6 hrs + Amikacina 15mg/kg/día c/24hrs, por 7 días • Ampicilina 150 mg/kg/día c/8hrs + o Cef. de 3° Gen. • En el RN que se sospeche:

• Infección estafilocócica se agrega cloxacilina • Infección por chlamydia trachomatis se agrega eritromicina 50mg por

kg/día cada 6 horas por 14 días.

ATB en >3meses -5 años

• Ampicilina EV 150-200 mg/kg/día c/6 horas por 10-14 ds. • PNC EV 200.000 kg/día c/6horas • Después de 4 días mínimo, dar de alta con Amoxicilina oral (switch theraphy)

para cumplir 7 días de tratamiento en total. • En neumonía atípica:

• Adicionar o cambiar a Claritromicina 15 mg/kg/día c/12hrs por 10-14 ds.

ATB Neumonía grave, supurada, fracaso terapéutico 1° línea- 48hrs.

• Cefotaxima 100mg/kg/día, c/6 hrs por 7 días • Ceftriaxona 100 mg/kg/día c/12hrs por 10-14 días, según evolución clínica. • En neumonía supurada:

• adicionar Cloxacilina 100-200 mg/kg/día, c/4-6 hrs, si hay ambiente epidemiológico de infección estafilocócica.

Resistencia del Streptococus Pneumoniae:

Esta dada por mutaciones en el sitio de accipon de la penicilina, con ↓de la afinidad de las portienas de unión del ATB. Se produce por adquisipon de ADn

Page 104: Temas de examen Pediatria

de espescies resistentes.La ↓ de la afinidad podría contrarestarse con↑ de la dosis de betalactamico o con el uso de cefalosporinas de 3° generación. Los macrólidos son ineficientes.

Recordar que la resistencia del neumococo es por PBP y no por B-lactamasas.

Criterios neumococo resistente (16% en Chile): • <2 años o 18 meses*, Sala cuna o jardín, ATB en últimos 3 meses • OMA recurrente, hospitalización previa, Inmunosupresión

COMPLICACIONES

Derrame pleural paraneumónico

Empiema pleural

Neumotórax

Derrame pericárdico

Miocarditis

Septicemia

Atelectasias

Absceso pulmonar

Fracaso terapéutico

Page 105: Temas de examen Pediatria

22. CONSTIPACIÓN

Apuntes de clase “Constipación en niños” realizada por Dr. Eduardo Hebel, UFRO, 2010.

Shaman Rajindrajith, Constipation in Children: Novel Insight Into Epidemiology, Pathophysiology and Management. J Neurogastroenterol Motil 2011;17:35-47

DEFINICIÓN

Es una retención anormal de materias fecales que se manifiesta clínicamente por deposiciones de menor contenido acuoso (consistencia aumentada) que producen disconfort al paciente.

Consideraciones:

• Un patrón normal de evacuación intestinal es cuando el niño, hace día por medio o cada dos días, pero sin síntomas.

• Condiciones para una normal evacuación: Volumen adecuado de materia fecal, normal actividad propulsiva de colon y recto, normalidad anatómica, reflejo recto-esfinteriano, e indemnidad del eje sistema nervioso central, columna vertebral y esfínter anal externo

• Los criterios de Roma III para niños a diferencia de la clasificación en adultos se basan en el síntoma principal y no están en función del órgano afectado

ETIOLOGÍA

No Orgánicas Orgánicas

• Entrenamiento esfinteriano coercitivo

• Predisposición genética

• Dietéticas, escasa ingesta de fibra

• Abuso sexual

• Malformaciones: Ano imperforado, anterior, estenosis • Metabólicos: Hipotirodismo, Hipercalcemia,

Hipokalemia, CF, DM, Enf. Celíaca • Neuropáticas: SNC, Medula espinal • Enf. Inervación/ musculares: Enfermedad de

Hirschsprung, Displasia neuronal, Miopatía o Neuropatía Visceral.

• Alt. Musculares: Prune Belly, Gastrosquisis y S. Down • Drogas:: Opiáceos, Fenobarbital, Sucralfato, Antiácidos,

Antihipertensivos, Anticolinégicos, Antidepresivos • Miscellaneous: Ingestión de metales pesados , Vit D

FACTORES DE RIESGO

• Dieta pobre en fibra • Estrés psicológico • Alergia a la proteína de la leche de vaca • Predisposición familiar • Prematurez • Vivir en áreas urbanas

Page 106: Temas de examen Pediatria

DIAGNÓSTICO

Existen 3 entidades descritas en la Clasificación ROMA III, teniendo cada una criterios diagnósticos diferentes

Disquezia infantil

Debe cumplir los 2 siguientes en menores de 6 meses: • Episodios de esfuerzos y llanto por 10 minutos antes de una evacuación

efectiva (deposiciones normales) • Ausencia de otros problemas de salud

No indicar nunca el termómetro, es muy peligroso, se puede romper y puede causar más daño. Sí las soluciones de glicerina.

Constipación Funcional en <4años

• Debe incluir 1 mes con 2 de los siguientes en niños < 4 años: • <2 deposiciones a la semana. • 1 episodio encopresis/semana. • Historia de retención fecal excesiva. • Historia movimientos intestinales dolorosos o intensos. • Presencia fecalomas en recto. • Historia de heces de gran tamaño. • Síntomas que alivian con deposiciones.

Constipación Funcional en > 4 años

• Debe cumplir 2 de los siguientes en niños > 4 años, por lo menos 1 vez por semana 2 meses antes del diagnóstico:

• < 2 de posiciones a la semana. • 1 episodio encopresis/semana. • Historia de posturas retentivas o retención fecal voluntaria. • Historia movimientos intestinales dolorosos o intensos. • Presencia de fecalomas en recto. • Historia de heces de gran tamaño

TRATAMIENTO

NASPGHAN Guías Clinicas-1999-naspghan.org 4 pilares fundamentales: 1. Educación 2. Desimpactación 3. Tratamiento de mantención 4. Modificación conductual

1. Educación

• En que consiste una normal evacuación • Hábitos intestinales y actividad escolar, Hábitos dietéticos, Factores

psicosociales, Actividad física.

Page 107: Temas de examen Pediatria

2. Desimpactación

Desimpactación del Fecaloma y Vaciamiento intestinal (1-2 sem) • Dieta hídrica (las primeras 2 sem no es una dieta rica en residuos o fibras) • Iniciar tratamiento de la fisura anal en lactantes, para romper el ciclo vicioso del

dolor que sigue llevando a la constipación • Opciones

• Enemas evacuantes diarios: Solución salina, Sol. Fosfato • Solución electrolítica polietilenglicol (PEG): Es un estimulante

osmótico, que se administra VO, si bien es un procedimiento no invasivo, es un procedimiento más largo. Además es mal tolerado por los niños, porque el sabor es muy malo

Proctoclísis, consiste en inyectar suero al fecaloma. Todavía se indica en algunos casos, sobretodo cuando existen fecalomas inamovibles

3. Tratamiento de mantención

Mantención y Prevención de reimpactación (6-12meses) • Dieta rica en fibra en niños constipados (edad en años + 5 = grs de fibra por día) • Farmacología en constipación crónica funcional

• Lactulosa (Solución 70% ): 1-3 ml/kg/día (muy costosa y tiene dos RAM: diarrea, dolor abdominal. Si tiene esto es mejor retirarlo de apoco.

• Leche de Magnesia: 0.5-3 ml/kg/día (es bueno pero mal tolerado, por mal sabor)

• Polyethylene glicol: 0.8 g/kg/día (Más económico) Es muy bueno pero en niños muy pequeños pero puede causar una diarrea que puede llegar a deshidratar al niño.

• Aceite Mineral: 1-3 ml/kg/día (es la Vaselina), es lo que se usa. Pero puede producir mal absorción intestinal. Usarla en >3 años por el riesgo de aspiración ( puede producir una neumonía química severa)

En niños no se encuentra justificado el uso de laxantes. Hay que usar fármacos osmóticos como es la lactulosa. Estudio en Japón: estableció que los niños responden mejor con aceite mineral que con lactulosa. El aceite mineral en 10 veces mas barato que la lactulosa, pero siempre en mayores de3 años.

4. Modificación Conductual

• Entrenamiento del esfinter • Reforzamiento positivo • Apoyo psicológico • Tratamiento con Biofeedback: Mediante un sistema para normalizar el reflejo

esfinteriano. No es 100% efectivo.

SEGUIMIENTO

Retiro de las medidas

• Tras 1 año de tratamiento retirar gradualmente fármacos, con mantención dieta rica en fibras (Estilo de vida) y Control periódico de la frecuencia y características de las deposiciones

Page 108: Temas de examen Pediatria

CONTROL

El tratamiento es de 1 – 1,5 años, con controles periódicos en ese tiempo Objetivo del tratamiento: • A 1 año tratamiento (> 3 Evac./sem, <2 epis. escurr/mes, ausencia de dolor abd,

sin apoyo farmacológico por un mes. • Ausencia de encopresis y escurrimiento.

Page 109: Temas de examen Pediatria

23. DIARREA AGUDA

Tratamiento de la diarrea, Manual Clínico para los Servicios de Salud, Organización Panamericana de la Salud, 2008.

Manual MIR, Enfermedades del aparato digestivo I. Gastroenteroenterologia.

DIARREA AGUDA EN LA INFANCIA. Actualización sobre criterios de diagnóstico y tratamiento

MANUAL DE LABORATORIO, Dirigido a Servicios Clínicos y Toma de muestra del Hospital Dr. Hernán Henríquez Aravena de Temuco, Marzo 2011.

DEFINICIÓN

Diarrea: 3 o más episodios de deposiciones líquidas en 24 horas. Sin embargo, la disminución de la consistencia es incluso más importante que la frecuencia. Se considera aguda si dura menos de 15 días.

ETIOLOGÍA

• Viral: generalmente afecta lactantes y niños pequeños, sobre todo en época otoñal, de comienzo brusco, con vómitos y fiebre que preceden en varias horas al comienzo de las deposiciones diarreicas.

• Bacteriana: más frecuente en época estival en niños mayores y con condiciones deficitarias de higiene personal, ambiental o alimentaria.

• Parásitos • Secundaria a terapia antibiótica

Más frecuentes Menos frecuente

Bacterias • E. coli enteropatógeno • Shigella • E. coli

enterohemorrágico • Salmonella sp.

• E.coli toxigénico • E.coli invasor • Yersinia enterocolítica • Campylobacter jejuni • Vibrio cholerae • Clostridium difficile • Aeromona hidrófila

Virus • Rotavirus • Virus de 27nm (virus pequeños) • Adenovirus , Astrovirus , Calicivirus

Parásitos • Giardia lamblia • Cryptosporidium

parvum

• Entamoeba histolytica

ESTUDIO

Las características de las deposiciones son de gran valor diagnóstico, dado que permiten inferir el mecanismo fisiopatogénico involucrado y el sitio comprometido, estableciendo la siguiente clasificación: DIARREAS ACUOSAS involucran a intestino delgado, son líquidas, abundantes y llevan a la deshidratación con pérdidas variables de electrolitos, según el mecanismo de acción.

• Secretoras: mediadas por enterotoxinas que actúan a través del AMP cíclico, estimulando la secreción activa de agua y electrolitos hacia la

Page 110: Temas de examen Pediatria

luz intestinal (pérdidas importantes de sodio). Por ejemplo: Vibrio cholerae, Shigella, Salmonella, Staphylococcus y Clostridium perfringens.

• Malabsortivas: por disminución de la superficie de absorción de la mucosa originan pérdidas intermedias de electrolitos y ocasionalmente pueden producir deposiciones con moco y estrías de sangre. Por ejemplo: Giardia lamblia, Rotavirus, Escherichia coli enteropatogénica (ECEP) y E. coli enterohemorrágica (ECEH).

• Osmóticas: atraen agua y provocan deposiciones ácidas con bajo contenido de sodio (Na) y aumento en la producción de gas.

DIARREAS DISENTÉRICAS, con invasión y penetración de la mucosa del colon y a veces del ileon terminal, se caracterizan por fiebre alta, heces frecuentes, pequeñas con moco y sangre, acompañadas de cólicos, pujos y tenesmo. Por ejemplo: Shigella, Escherichia coli enteroinvasiva (ECEI) y raramente Salmonella, Campylobacter yeyuni, Yersinia enterocolítica y Entamoeba hystolitica. El examen clínico debe ser completo, para descartar otros focos infecciosos. Los datos de mayor relevancia son: el estado de hidratación, que depende del tiempo de evolución, de la magnitud de las pérdidas y de la calidad y cantidad de líquidos administrados.

PRESENTACIÓN CLÍNICA SEGÚN ETIOLOGÍA

Agente Características

Rotavirus • Mec. Transmisión: fecal oral • Más frecuentes en: menores de 2 años concurrentes a jardín

otoño-invierno. • Causa lesiones irregulares en el epitelio del intestino delgado,

que da lugar a una atrofia de las vellosidades. • Se caracteriza por deposiciones líquidas, vómitos y fiebre de

hasta 39°C. Los vómitos son un síntoma muy constante (90% de los pacientes).

• La deshidratación es frecuente.

E.coli enteropatógeno

• Se presente en niños <1año • Se adhiere estrechamente a la mucosa del intestino delgado • produciendo una lesión característica (disolución del ribete en

cepillo en el sitio de la adherencia de la bacteria) • Se caracteriza por un comienzo insidioso, deposiciones líquidas

de mal olor, puede acompañarse de vómitos y la fiebre puede estar presente pero es de poca cuantía y dura 1-2 días. En ocasiones y en algunos serotipos, las deposiciones pueden ser extremadamente abundantes asemejando perfectamente un cólera.

Staphylococcus aureus . Duración:<24 hrs P. Incubación: 1-7 hrs

• Mec. Transmisión: Alimentos contaminados por manipuleo de persona colonizada.

• Es el agente causal más frecuente de toxi-infección alimentaria. • Cuadro dado por la enterotoxina. Inicio del cuadro brusco con

náuseas y vómitos intensos. • Tratamiento sintomático.

Page 111: Temas de examen Pediatria

Clostridium perfringens .

• Mec. Transmisión: • Cuadro dado por la enterotoxina • Gastroenteritis leve. • Tratamiento sintomático, en casos graves penicilina.

Vibrio parahemoliticus

• P. Incubación: 15-24 hrs . • Mec. Transmisión: pescado mal cocido

Vibrio cholerae

• Mec. Transmisión: Agua y alimentos contaminados.

Clostridium botulinum P. Incubación: 18-36 hrs

• Mec. Transmisión: conservas caseras • bacilo anaerobio gram-positivo con 7 tipos de toxinas • Cuadro neuromuscular bilateral y simétrico (inicio en pares

craneales y posteriormente parálisis descendente) precedido de diarrea, vómitos y dolor abdominal.

Salmonella no typhi: Autolimitada. 1 a 7dias.

• P. Incubación: 6-72 hs • Mec. Transmisión: Alimentos contaminados, incluyendo huevo

crudo. En caso de grave afectación sistémica: ATB antibióticos (TMP-SMX, cloranfenicol).

AGENTES ASOCIADOS A DISENTERIA

Agente Características

Shigella (flexneri, sonnei, boydii)

• Mec. Transmisión: Agua y alimentos contaminados y fecal-oral. Se trasmite directamente por contacto de persona a persona ya que la dosis infectante es baja (de 10 a 100 microorganismos).

• Causa inflamación, microabcesos, úlceras y producción de mucus.

• Más frecuente entre 6 meses y 2 años. • Se caracteriza por comienzo brusco con deposiciones líquidas,

fiebre alta y toxemia y luego dentro de las 24 horas siguientes aparece la disentería y tenesmo.

• Por liberación de su neurotoxina pueden existir síntomas neurológicos y convulsiones.

• En un porcentaje importante de los casos no se produce disentería, pero el examen de leucocitos fecales es positivo en un casi 100%.

• Artritis reactivas a las 2-3 semanas. • Tratamiento de elección: ciprofloxacino.

E. coli entero-hemorrágico

• Producen 1 o 2 toxinas similares a la toxina de la Shigella dysenteriae I y se llaman toxina Shiga 1 y toxina Shiga 2. Estas toxinas inhiben la síntesis proteica produciendo daño celular directo.

• Se puede iniciar con síntomas generales como fiebre, dolor abdominal, irritabilidad, fatiga, cefalea, mialgias y confusión. Luego aparece la diarrea no sanguinolenta por 24-28 horas para luego iniciarse la diarrea sanguinolenta.

• El dolor abdominal es intenso y desproporcionado a los hallazgos del examen físico. Es espasmódico e intermitente.

Page 112: Temas de examen Pediatria

Campylobacter yeyuni:

P. Incubación: 1-7 días

• Mec. Transmisión: Aves de corral, aguas contaminadas. • La diarrea puede ser líquida, pero en un tercio de los casos

aparecen heces disentéricas al cabo de uno o dos días. No es frecuente que se produzcan vómitos y la fiebre es generalmente baja.

• El dolor abdominal puede ser tan intenso que semeje una apendicitis aguda.

Yersinia enterocolitica

• Invade la mucosa del ileon y colon. Puede producir lesiones semejantes a las de la enf. de Crohn. La presencia de úlceras aftosas (1-2mm diámetro) durante un episodio de gastroenteritis orienta a yersiniosis.

• En niños mayores y adultos produce con más frecuencia adenitis mesentérica cuya presentación semeja una apendicitis aguda.

• Tratamiento de elección: Ciprofloxacino

Amebiasis • Se debe plantear dicho tratamiento cuando en el examen microscópico de las heces frescas realizado en un laboratorio fiable se observen trofozoítos de E. histolytica que contienen eritrocitos fagocitados o cuando, después de administrar dos antimicrobianos diferentes habitualmente eficaces frente a las cepas de Shigella de la región, no se haya producido mejoría clínica.

Cryptosporidium parvun y Giardia lamblia si bien se asocian a diarrea prolongada, pueden dar episodios de diarrea aguda.

PRESENTACIONES CLÍNICAS

Intoxicación bacteriana alimentaria

• Ingestión de alimentos contaminados con bacterias o toxinas generadas por éstas. Para el diagnóstico es necesario que: dos o más personas sufran el cuadro de diarrea aguda tras la ingesta de una comida común y que un análisis epidemiológico implique a un alimento como responsable.

Diarrea aguda del viajero

• Cuadro diarreico agudo que afecta a las personas que se desplazan de un área con una baja tasa endémica de diarrea a otra de mayor.

• La causa más frecuente es el E. coli enterotóxico, aunque otro microorganismo causante es el virus Norwalk.

• El cuadro clínico se inicia a las 72-96 horas de la llegada con dolor abdominal y diarrea intensa. Autolimitada en 48 horas.

Colitis asociada a antibióticos

• Etiología: toxinas del Clostridium dificile que prolifera al alterarse la flora bacteriana tras utilización de ATB de amplio espectro. Más frecuentemente implicada: clindamicina. Pero la puede producir cualquier antibiótico.

• Clínica: inicio durante o hasta 10 días después de la administración del antibiótico (30%). Suele ser diarrea sanguinolenta con afectación general. Puede complicarse con megacolon tóxico o perforación colónica.

Page 113: Temas de examen Pediatria

DIAGNÓSTICO

Coprocultivos

• Detecta mediante cultivo: salmonella spp, shigella spp, yersinia enteroclítica y Vibrio.

• Detectas mediante gram (coprocultivo directo): Campylobacter spp. • Por detección de toxina, identifica: Clostridium difficile. • En 2 días se puede saber si es negativo o positivo para salmonella, shigella,

yersinia o Vibrio. • Si se pide buscar ECEH, tarda 15 días o más (derivado a ISP)

Es de mucha utilidad cuando resulta positivo, porque nos permite un tratamiento antibiótico adecuado, sin embargo la positividad del coprocultivo es baja y un resultado negativo no descarta infección bacteriana.

Leucocitos fecales:

Es un examen de screening. En general indican inflamación del colon por patógenos con capacidad invasora.

Rotaforesis:

Es de alto rendimiento y se solicita en todo cuadro diarreico severo o que persista más allá de 3 días.

Examen parasitológico de deposiciones:

Es un examen de baja positividad, por lo que un parasitológico negativo no descarta la presencia de parásitos.

pH y cuerpos reductores en deposiciones:

La intolerancia a la lactosa es la causa más frecuente de diarrea prolongada, por lo que el examen es de mucha utilidad cuando la diarrea se prolonga más allá del tiempo esperado (7 días). La presencia de un pH bajo 6 y cuerpos reductores positivos en 2 a 3 oportunidades en un paciente que está recibiendo lactosa, nos orienta fuertemente a una intolerancia a la lactosa.

Diarrea

Sin sangre BCG, inmuno-competente

Observación

Fiebre >5 días

Coprocultivo

Compromiso sistémico, inmunocomprometido,

RN.

Intrahospitalaria

Disentérica

Page 114: Temas de examen Pediatria

TRATAMIENTO

ALIMENTACIÓN

• El objetivo es dar tantos alimentos ricos en nutrientes como el niño acepte. • No debe interrumpirse la alimentación habitual del lactante durante la diarrea y,

una vez finalizada esta, debe aumentarse. Nunca debe retirarse la alimentación y no deben diluirse los alimentos que el niño toma normalmente.

• Siempre se debe continuar la lactancia materna. • Los alimentos ricos en potasio, como los plátanos y el jugo de fruta recién hecho

resultan beneficiosos. • Ofrezca comida al niño cada tres o cuatro horas (seis veces al día). Las tomas

pequeñas y frecuentes se toleran mejor que las grandes y más espaciadas.

HIDRATACIÓN

Hallazgos a la exploración física según la gravedad de la deshidratación:

LEVE O

INAPARENTE MODERADA O

CLÍNICA GRAVE

Pérdida de agua corporal

< 50 ml/kg peso o < 5% del peso

50 -100 ml/kg peso ó

6-9% del peso

100 ml/kg peso o >10% del peso

Condición general

Bien, alerta *Irritable *Letárgico o inconsciente

Ojos Normales Algo hundidos Muy hundidos y

secos

Lágrimas Presentes Ausentes Ausentes

Mucosas orales

Húmedas Secas Muy secas

Sed Paciente bebe normalmente

*Paciente bebe con avidez,

sediento

*Paciente bebe mal o no es capaz de

hacerlo

Pulso Normal y lleno Rápido Rápido y débil

P° arterial Normal Normal o baja Choque

Diuresis Disminuida Muy disminuida Anuria

Fontanela anterior

Normal Hundida Muy hundida

DIAGNÓSTICO

No tiene signos de

deshidratación

≥ 2 signos de la columna,

incluyendo al menos un signo destacado con *

≥ 2 signos de la columna, incluyendo

al menos un signo destacado con *

TRATAMIENTO PLAN A Pesar al paciente

si es posible, y usar PLAN B

Pesar al paciente si es posible, y usar PLAN C, URGENTE

Plan A

• Administrar mayor cantidad de líquidos. • Mantener alimentación adecuada. • SRO después de cada deposición diarreica o vómito.

Page 115: Temas de examen Pediatria

• Líquidos inadecuados: bebidas endulzadas con azúcar, que puede causar diarrea osmótica e hipernatriemia. Algunos ejemplos son: bebidas gaseosas comerciales; jugos de fruta comerciales; té endulzado.

• Otros líquidos que también deben evitarse son los que tienen un efecto estimulante, diurético o purgante, por ejemplo: café, algunos tipos de té o infusiones medicinales.

Plan B

• SRO 75mg/kg en las primeras 4 horas. Después de 4 hrs evaluar nuevamente. • Las soluciones deben ser recién preparadas, preferiblemente con agua recién

hervida y enfriada. • Si el niño pide más SRO, dar más. • Si el niño vomita, esperar 10 minutos y luego continuar con más lentitud. • Consumir dentro de las 24 horas de preparada. • No sobrepasar los 250 ml/kg/día.

SALES DE REHIDRATACIÓN ORAL CASERA

En un litro de agua potable o hervida, agregue: • 2 cucharadas de azúcar • 1/4 cucharadita de sal • 1/4 cucharadita de bicarbonato de sodio. 1 cda rasa = 15 grs /1 cdta rasa = 5 grs.

Plan C

Fluidos EV inmediato según OMS ó “Sodio 70, 50 o 35”

Edad Inicialmente: 30ml/kg en

Luego: 70ml/kg en

Lactante < de 1 año 1 hora* 5 horas

Niños > de 1 año 30 min* 2 horas y media

MEDICAMENTOS

Suplementos de Zinc

Numerosos estudios han revelado que la administración de 10 a 20 mg diarios hasta la desaparición de la diarrea disminuye significativamente la gravedad y duración de la diarrea en niños menores de 5 años.

Antimicrobianos

• NO dar antibióticos, salvo que se demuestre Shigella • Disentería por Shigella: Ciprofloxacino 15 mg/kg c/12 hrs durante 3 días

(alternativa: Ceftriaxona 50 a 100 mg/kg c/24hrs IM durante 2-5 días. • Amebiasis: Metronidazol 10 mg/kg c/8hrs durante cinco días (10 días para la

enfermedad grave)

“Antidiarreicos” y antieméticos

No tienen ningún beneficio práctico para los niños con diarrea aguda o persistente. No previenen la deshidratación ni mejoran el estado nutritivo, que deben ser los objetivos principales del tratamiento. Algunos tienen efectos colaterales peligrosos e incluso pueden ser mortales. Estos medicamentos nunca deben administrarse a niños menores de 5 años

Page 116: Temas de examen Pediatria

• Adsorbentes (por ejemplo, esmectita, carbón activado, colestiramina). Estos medicamentos tienen la capacidad para unirse e inactivar las toxinas bacterianas u otras sustancias que causan la diarrea y por su pretendido efecto “protector” de la mucosa intestinal. Sin embargo, ninguno ha demostrado en la práctica su verdadero valor para el tratamiento sistemático de la diarrea aguda infantil.

• Inhibidores de la motilidad intestinal (por ejemplo, loperamida, atropina) pueden reducir la frecuencia de las deposiciones en los adultos. Sin embargo, no reducen apreciablemente el volumen de las heces en los niños pequeños. Es más, pueden causar íleo paralítico grave, el cual puede ser mortal, y prolongar la infección debido a que se retrasa la eliminación de los microorganismos que la originan. Las dosis terapéuticas normales pueden producir sedación y algunos medicamentos de este grupo farmacológico resultan tóxicos para el sistema nervioso central y causan la muerte según los informes. No se debe administrar ninguno de estos medicamentos a los lactantes ni a los niños con diarrea.

• Antieméticos (clorpromazina) la sedación que causan puede obstaculizar la terapia de rehidratación oral. Por este motivo, los antieméticos nunca deben administrarse a los niños con diarrea. Además, los vómitos desaparecen cuando el niño está rehidratado.

COMPLICACIONES

• Deshidratación (la más frecuente). • Acidosis metabólica. • Hipokaliemia (más intensa si existe abundante moco en las deposiciones). • Hipomagnesemia (posible tetania).

Page 117: Temas de examen Pediatria

24. DIARREA CRÓNICA

Clase “DIARREA CRÓNICA.”, realizada por Dra. María Eugenia Arancibia, Universidad de Chile

CANALES R., ALLIENDE G., Diarrea crónica en el niño, Rev Chil Pediatr 2012; 83 (2): 179-184

DEFINICIÓN

Según la Organización Mundial de la Salud la diarrea se define como 3 o más evacuaciones de deposiciones líquidas o acuosas en 24 horas. Se destaca que la consistencia y volumen de las deposiciones son más importantes que su frecuencia. Se considera aguda cuando la duración es menor a 7 días y crónica cuando se presenta por 30 o más días.

SOSPECHA DIAGNÓSTICA

• Diarrea que presenta por 30 o más días

EVALUACIÓN INICIAL

En primer lugar, es importante determinar si el paciente tiene efectivamente una diarrea crónica. En ocasiones el escurrimiento observado en un paciente con constipación crónica puede ser erróneamente interpretado como diarrea. El estudio debe ser sistemático y progresivo, orientado por la historia clínica. Se inicia con una evaluación del estado nutricional y con exámenes generales destinados a descartar infección y evaluación de las funciones digestivas y absortivas. Luego, en algunos casos, se requerirá de estudios más específi cos a cargo del especialista.

Page 118: Temas de examen Pediatria

Exámenes generales

• Hemograma y VHS: evaluar presencia de anemia, signos de carencia de hierro, leucocitosis, eosinofi lia, VHS elevada.

• Perfi l Bioquímico: albúmina, globulinas, glicemia, hiperbilirrubinemia. • Estudio de Coagulación: défi cit de Vitamina K. • Coprocultivo corriente y con tinciones especificas para Campylobacter yeyuni,

yersinia enterocolítica y cryptosporidium. • Parasitológico seriado de deposiciones. • Cuerpos reductores y pH en deposiciones. • Esteatocrito. • Hemorragias ocultas en deposiciones. • Orina completa y urocultivo.

PRESENTACIONES CLÍNICAS SEGÚN ETIOLOGÍA

Giardiasis

Es un protozoo que produce la mayoría de las veces una infección asintomática pero en otros casos puede producir diarrea ya sea autolimitada o diarrea crónica. En general el cuadro se caracteriza por dolor y distensión abdominal, deposiciones abundantes y en ocasiones esteatorrea. Diagnóstico se realiza solicitando un PSD pero hay que tener en cuenta que la sensibilidad no es muy alta, la solicitud de tres PSD la aumenta a sobre un 90%, pero esto es engorroso por esta razón cuando exista una alta sospecha de infección, a pesar de un estudio negativo se debe tratar al paciente. El medicamento de elección es el Metronidazol por 7 a 10 días.

Enfermedad Celiaca

Se caracteriza por una intolerancia permanente al gluten, proteína presente en el trigo, la avena, centeno y la cebada. Es una condición determinada genéticamente Aunque la clínica puede ser variable, la enfermedad se presenta luego de un periodo de meses de ingestión de gluten, con diarrea insidiosa, deposiciones esteatorreicas muy voluminosas, de mal olor, pastosas y grasosas. Compromiso progresivo del estado nutritivo, distensión abdominal, apatía, vómitos, disminución marcada de la masa muscular y atrofia glútea; este es el cuadro clásico, lo ideal es tener un alto índice de sospecha realizando un diagnostico precoz sin esperar un gran deterioro del paciente. Existe una forma grave de presentación, la crisis celiaca donde se produce diarrea con serios trastornos en el equilibrio hidroelectrolítico. Estudios de absorción intestinal: caroteno (basal y con sobrecarga) y d-xilosa, la que determina el estado de la pared. El estudio de absorción de grasas tiene como gold estándar al método de van de Kamer, de poca utilidad clínica por lo engorroso que es. Menos exacto son la realización de Sudan III y el esteatocrito.

Diarrea Crónica Inespecífica:

Se presenta en niños entre 6 meses y 5s años . Se caracteriza por presentar diarrea en general bien tolerada, sin compromiso del estado general. Las deposiciones pueden ser líquidas, semilíquidas con frecuencia con lienteria.

Page 119: Temas de examen Pediatria

El diagnóstico se realiza descartando malabsorción (d-xilosa, caroteno, hemograma normales) parásitos con PSD, alergia alimentaria, diarrea asociada a antibiótico. El tratamiento consiste en realizar una restricción del exceso de líquidos, jugos, golosinas manteniendo en los otros aspectos una dieta normal sin restricción de fibra.

Alergia Alimentaria:

Principalmente observamos alergia a la proteína de leche de vaca, a pesar de que existen muchas proteínas en la leche de vaca la principal implicada es la b-lactoglobulina. Puede presentarse clínicamente de variadas formas: cólicos infantiles, rectorragia producida por colitis, sintomatología respiratoria (sibilancias, rinitis), lesiones cutáneas como eczema y reacción anafiláctica, la enteropatía por leche de vaca se manifiesta por diarrea y deterioro del desarrollo pondoestatural. Los antecedentes familiares de alergia, el estudio histológico si hay diarrea y la mejoría al suprimir la proteína de leche hacen el diagnóstico. Este cuadro es transitorio y deben utilizarse para su tratamiento idealmente lactancia materna o hidrolizados de leche de vaca ya que existe alergia cruzada con las leches que contienen soya.

Fibrosis Quística:

Enfermedad autosómica recesiva del cromosoma 7 donde existe una alteración en una proteína conocida por CFTCR, que regula la conductancia transmembrana produciendo alteraciones en varios sistemas. A nivel gastrointestinal se produce una insuficiencia pancreática exocrina con la consecuente diarrea con malabsorción El diagnóstico se realiza con exámenes generales que demuestran mala absorción y el test de sudor alterado en al menos dos oportunidades con niveles de cloro o sodio mayores de 60 meq/lt. Una vez realizado el diagnóstico se debe realizar aporte de enzimas pancreáticas en cada alimentación además de un aporte nutricional adecuado para el gasto aumentado que presentan.

Intolerancia a Hidratos De Carbono:

Provocado por déficit enzimático ya sea primario como es el caso del déficit de sacarasa/isomaltasa manifestada por intolerancia primaria a la sacarosa que se manifiesta cuando se ingiere ese sustrato, o secundaria como es el caso de déficit de lactosa secundaria a una gastroenteritis como la provocada por rotavirus. La diarrea se caracteriza por ser liquida, ácida y explosiva y en general con distensión abdominal. El estudio demuestra un ph bajo 5,5.en las deposiciones.

Page 120: Temas de examen Pediatria

DIAGNÓSTICO DIFERENCIAL

Menores de 6 meses

Mayores de 6 meses y preescolares

Escolares

• Síndrome post enteritis • Fibrosis quística • Alergias alimentarias • Inmunodeficiencias • Enfermedad por inclusión

vellositaria • Defectos de

transportadores • Clorhidrorrea congénita • Malabsorción de glucosa-

galactosa • Enteropatía autoinmune

• Síndrome post-enteritis • Enfermedad celíaca • Alergias alimentarias • Giardiasis • Diarrea crónica

inespecífica • Intolerancia a disacáridos • Tumores • Enfermedad inflamatoria

intestinal

• Enfermedad celíaca

• Intolerancia a disacáridos

• Enfermedad inflamatoria intestinal

• Síndrome de intestino irritable

• Tumores

TRATAMIENTO INICIAL

Medidas generales

Estabilizar al enfermo, si lo requiere. Asegurar una adecuada hidratación y nutrición, normalizando la dieta cuando sea posible, evitando regímenes innecesarios o prolongados. Es recomendable además, no suspender la lactancia materna, no introducir alimentos nuevos, retirar jugos de fruta, dado su alto contenido de azúcar, evitar empleo de antibióticos y evitar uso de inhibidores del peristaltismo.

Medidas específi cas

Orientadas por las conclusiones obtenidas del proceso diagnóstico, están destinadas a resolver el problema que originó la diarrea.

DERIVACIÓN

Page 121: Temas de examen Pediatria

25. SÍNDROME HEMOLÍTICO URÉMICO

Apuntes de clase “Síndrome Hemolítico Urémico” realizada por Dra. Mella, UFRO-2010.

Seminario “Síndrome Hemolítico Urémico” realizado por Dra. Dinamarca.

Pedro Zambrano, Angela Delucchi, Felipe Cavagnaro. “Síndrome hemolítico urémico en Chile: presentación clínica, evolución y factores pronósticos”, Rev Méd Chile 2008;

Patrick Niaudet, MD. “Treatment of Shiga-like toxin associated (typical) hemolytic uremic syndrome in children” . Up to date 2010

Manual CTO Medicina y Cirugía, Tomo Pediatría “Síndrome Hemolítico Urémico”, 8va ed.

DEFINICIÓN

• El Síndrome Hemolítico Urémico (SHU) es una microangiopatía trombótica que afecta a diversos órganos, con predominio sobre el riñón, el tubo digestivo y el sistema nervioso central.

GENERALIDADES

• Es la causa más frecuente de insuficiencia renal aguda en <3 años en Chile. • La Escherichia coli productora de shigatoxina (Stx) causa el 90% de los SHU que

ocurren en los niños, de estás, la E.coli enterohemorrágica O157:H7 es la más frecuente.

• La Stx se une a receptor Gb3 (membrana de células blanco), esto facilita la internalización de la toxina, inhibiendo la síntesis proteica y la muerte celular.

• La Stx es transportada por GR, LB, monocito y granulocito • Historia natural del SHU Shigatoxina positiva(Stx +)

• ≈60% resolución. • ≈30% Proteinuria, HTA, otras secuelas menores. • ≈5% IRC, AVE, otras secuelas mayores. • ≈3-5% muerte.

CLASIFICACIÓN

Síndrome hemolítico-urémico típico (90-95%)

Síndrome hemolítico-urémico atípico (5-10% de los casos)

• Antecedente de gastroenteritis enteroinvasiva por E. coli serotipo 0 1 57:H7, aunque también hay casos precedidos por Shigella.

• La E coli 0157:H7 es comensal del intestino de animales como las vacas, y produce Shiga Toxina. Una mínima dosis de bacterias produce daño, porque es la toxina, no la bacteria en si, la que provoca el daño.

No está asociado a diarrea. Se han descrito múltiples causas: • Formas genéticas (50%) • Infecciones (neumococo, VEB, virus

cocksakie,etc.) • Fármacos (ciclosporina, tacrolimus,

anticonceptivos orales, etc.), • Formas autoinmunes, pos-trasplante

(renal, hepático y de progenitores hematopoyéticos), les, esclerodermia, etc.

• Relacionados con verotoxinas sin antecedente de diarrea.

Page 122: Temas de examen Pediatria

CLÍNICA

Entre 1y 15 días tras el episodio desencadenante (diarrea disentérica), se producen:

Alteraciones hematológicas (preceden al fallo renal): • Anemia Hemolítica que se manifiesta con astenia y palidez intensa o ictericia

leve con orina de color pardo-rojizo. • Trombopenia leve-moderada que se presenta como petequias, equimosis o

púrpura, siendo excepcionales los signos de sangrado masivo. No hay alteraciones de la coagulación ni fenómeno de CID.

Disfunción renal: • Oligoanuria (>50%) junto a alteraciones hidroelectrolíticas y metabólicas

características de la IRA. • Todos presentan hematuria (sólo 3 0% es macroscópica) • HTA ocurre en 1/3 de los pacientes por hipervolemia y daño vascular y suele ser

de difícil control.

Afectación del SNC (20%)* • Periodos de irritabilidad o de somnolencia, incluso las convulsiones.

Afectación gastrointestinal:* • Esofagitis, prolapso rectal, invaginación, perforación intestinal. • Puede haber hepatomegalia con hipertransaminasemia.

*no olvidar que receptores existen no solo a nivel renal El SHU atípico se caracteriza por la falta de pródromos gastrointestinales; el inicio es más insidioso con una fase prolongada antes del desarrollo de disfunción renal severa. La HTA es frecuente y, en ocasiones, refractaria al tratamiento.

SOSPECHA DIAGNÓSTICA

• Triada clásica: Anemia hemolítica microangiopatica, Trombocitopenia, IRA • Cuadro que comienza 5-19 días post inicio de cuadro de diarrea. • Los principales síntomas para sospechar son:

• Oliguria, • Edema • Marcada palidez. • Poco frecuente el purpura.

Anemia hemolítica microangiopatica de

grado variable Trombocitopenia IRA

• Esquistocitos>10% • Hiperbilirrubinemia

indirecta. • ↓ Haptoglobina sérica • Aumento de la LDH • Severidad de la anemia

no se correlaciona con severidad IRA

• Frecuentemente no causa púrpura ni sangrados

• Grado de trombocitopenia no se correlaciona con severidad de IR

• Pruebas de coagulación son normales o con mínimas alteraciones.

• Mayoría hematuria microscópica

• A mayor leucocitosis peor pronóstico (>13.000 al inicio)

Page 123: Temas de examen Pediatria

Diagnostico Microbiológico

• Screening: coprocultivo en agar mackonky en sorbitol • Cultivo de deposiciones 92% (+) dentro de los 6 días de inicio de la diarrea. Sin

embargo la aparición de SHU podría demorar hasta 20 días después del inicio de la diarrea. Por lo que podríamos no encontrar la bacteria, aunque si la toxina, ya que dura más tiempo.

• Examen confirmatorio: Anticuerpos contra antígeno O157 H7 • Biostar OIA SHIGATOX: Técnica rápida basada en inmunoensayo óptico que

permite detectar Stxs directamente de las deposiciones. Resultado se obtiene en 15 – 20 minutos. Sensibilidad 100% y especificidad de 98%.

• Premier EHEC, Meridian: Elisa en deposiciones (muestra o colonias).Sensibilidad 89% (100%) y especificidad de 100%.Resultados en 24 horas.

DIAGNÓSTICO DIFERENCIAL

• Invaginación intestinal • Sd. Tóxicos • Sepsis

• Encefalitis • Leucemias • Vasculitis

MANEJO INICIAL

• Explicar a familia (curso impredecible) • Hospitalizar, en UCI en caso de deterioro clínico, inestabilidad cardiovascular y

oligoanuria (50% requiere diálisis durante fase aguda) • El tratamiento es en general de soporte

• Balance hídrico estricto, peso diario • Evitar sobrecarga de volumen: furosemida ev. • Hidratación ev con cristaloides isotónicos para obtener óptima

protección del riñón. • Monitorización de ELP y ácido-base. • Se recomienda transfundir GR si hemoglobina < 6g/dl o Hcto <18%. • La transfusión de plaquetas está reservada para pacientes con

sangrado significativo, necesidad de realizar procedimiento invasivo y recuento de plaquetas < 10.000.

• No usar Antibióticos: ↑ liberación de toxinas (hasta 56%). Por lo tanto en un

cuadro de diarrea en niños, NO dar antibióticos, salvo que se demuestre Shigella • NO usar antiespasmódicos. Porque hacen que las bacterias se retengan por más

tiempo en el intestino y por lo tanto mayor producción de toxinas.

• Synorb-Pk: actúa como un absorbente de toxinas, PERO debe usarse dentro de los 3 primeros días, después no sirve, por lo que habría que darle a todas las diarreas para q sea útil. Su costo es elevado.

DERIVACIÓN

• Derivar a centro de mayor complejidad, como es un cuadro que afecta mucho órganos puede presentar pancreatitis , AVE, Coma, etc.

• Por lo anterior, tras pasar el cuadro agudo, debe tener control con especialista con exámenes de función Renal, registro de P/A y Ecografia renal.

Page 124: Temas de examen Pediatria

26. HEPATITIS AGUDA

Apuntes de clase “Hepatitis Aguda” realizada por Dr.Soza, UFRO-2010.

Manual de diagnóstico y tratamiento de enfermedades digestivas; sochigas; 2008

DEFINICIÓN

Se define como proceso inflamatorio agudo del tejido hepático ocasionado principalmente por virus hepatotropos, que tienen en común una afinidad selectiva por las células hepáticas. Puede presentarse desde cuadros leves y autolimitados, a formas graves y progresivas, que pueden conducir a insuficiencia hepática, cirrosis e incluso carcinoma hepatocelular.

CAUSAS

Infecciosas No infecciosas

• HEPATITIS A-B-C-D-E-G • VHS • CMV • VEB • ADV • Virus coxsackie

• Metabólicas/Geneticas: • Tirosinemia hepatorenal • Hepatopatía mitocondrial • Desorden de oxidación de ác. Grasos • Déficit de alfa 1 antitripsina

• Hepatitis AUTOINMUNE • Inducida por DROGAS • Isquemia

• Considerar que en RN, la presencia de ictericia de predominio directo esta asociada a causas bacterianas y por virus no hepatotropos, como: listeria, E.Coli, VEB, CMV.

FORMAS DE PRESENTACIÓN

Asintomática o inaparente

Anictérica o subclínica Ictérica o colestásica

• Presencia de antígenos en suero sin sintomatología acompañante

• Sintomatología presente pero sin ictericia, con elevación de transaminasas

Presenta 3 fases: • Prodrómica • Ictérica • Convalecencia

HEPATITIS A

Agente Virus RNA, pequeño, familia Picornavirus.

Transmisión Fecal-oral

Contagiosidad Es 7 días antes y durante los primeros 7 días de la fase ictérica, aunque se puede excretar hasta los 30 días.

Incubación 15-50 días. Promedio 30 días.

Cronicidad NO

Prevalencia Chile endemia intermedia

Presentación • Asintomática Anicterica (14 – 80%). Difícil de diferenciar de otras formas de gastroenteritis vírica

• Icterica: <6 años <10% , 6-14 años: 40-50%, >14 años: 70-80%

Page 125: Temas de examen Pediatria

Cuadro clínico

• Periodo Prodrómico (<7 días): frecuente náuseas, anorexia, vómitos, dolor abdominal difuso o hipocondrio derecho, ocasionalmente diarrea, a veces constipación, pérdida de olfato,. Se pueden presentar síntomas generales como adinamia, artralgias, fatiga. Es rara la fiebre.

• Fase Aguda o ictérica (7-10 días): Coincide con el compromiso enzimático del hígado. Comienza con hipocolia o acolia, coluria que desaparecen cuando aparece la ictericia, pero persisten los síntomas generales y digestivos. Se detecta hepatomegalia de magnitud variable, prurito.

• Fase de convalecencia (1 mes). Junto con disminuir la ictericia, se normalizan exámenes de función hepática y el hígado regresa anatómicamente.

• El laboratorio revela aumento de bilirrubina de predominio directo, elevación de transaminasas (GOT/GPT) y en los casos graves, disminución de la protrombina.

HEPATITIS B

Agente Virus de ADN, familia Hepadnaviridae

Transmisión Parenteral, sexual, secreciones y vertical.

Contagiosidad Fuente principal de infección: portadores crónicos sanos y pacientes con hepatitis aguda

Incubación 60-180 días.

Presentación Ictericia: < 5 años < 10%, >5 años 30-50% 90% va a la resolución 1% hepatitis fulminante

Prevalencia Baja prevalencia.

Cronicidad 5 – 10% (Adultos). 80 – 90% (R.N.)

Cuadro clínico

Las manifestaciones clínicas son similares a la hepatitis A, aunque más insidiosa. No son infrecuentes los síntomas y signos extrahepáticos como exantemas o artralgias. En las formas crónicas se puede asociar a glomerulonefritis.. Asociada con el VHD es más grave porque aumenta el riesgo de insuficencia hepática aguda. Las formas de infección crónica por HVB están asociadas a mayor incidencia de cáncer hepatocelular Laboratorio • HBsAg (marcador de infección por VHB) • HBeAg (marcador de replicación viral e infecciosidad) • Anti HBc (Ac anti-core igM e igG) marcador de infección aguda y crónica. • Anti Hbe (indica seroconversión, marca mejoría) • Anti HBs (marcador de inmunidad natural o por vacuna).

HEPATITIS AGUDA POR TOXICOS

2 tipos de reacciones hepáticas ante un tóxico: • Toxicidad directa: predecible, dosis dependiente, período de latencia

corto, no tienen respuesta de hipersensibilidad (artralgias, fiebre, eosinofilia, rash cutáneo). Son ejemplos: tetracloruro de carbono, tetraciclina, Amanita phalloides, paracetamol.

Page 126: Temas de examen Pediatria

• Toxicidad idiosincrásica: impredecible, no dosis dependiente, período de latencia variable -a veces después de terminado el tratamiento-, un 25% tienen reacción de hipersensibilidad. Son ejemplos: isoniazida, halotano, metildopa, eritromicina, cotrimoxazol, difenilhidantoína.

HEPATITIS AGUDA ALCOHOLICA

El daño hepático viene determinado por duración, dosis diaria de consumo y cierta susceptibilidad individual (en general, >60gr/día en varones y >30-40 gr/día en mujeres durante 10 años dan evolución a cirrosis). Lesiones hepáticas por el alcohol: • Hígado graso: Puede haber ligero aumento de GPT. Totalmente reversible. • Hepatitis alcohólica aguda: Sucede tras ingesta aguda excesiva. Cursa igual que

una hepatitis aguda viral y puede ser desde oligosintomática hasta hepatitis fulminante (si aparecen signos de encefalopatía indica mal pronóstico). En más del 50% de los casos cursa con fiebre alta >39°C.

DIAGNÓSTICO

LABORATORIO HEPATITIS AGUDA VIRAL

• Aumento de GPT y GOT en 10-100 veces su valor normal (<35-40 UI). El grado de aumento no tiene valor pronóstico.

• Hiperbilirrubinemia (> 2.5 mg./dl. da ictericia visible) • Fosfatasas Alcalinas y GGT elevación discreta, excepto en forma colestásica. • Tiempo de protrombina generalmente alrededor de 60 a 70 % • Para diagnosticar la etiología de la hepatitis se realiza serología para los distintos

virus. • HEPATITIS A: CLINICA + SEROLOGÍA (Anticuerpos IgM anti-VHA) • HEPATITIS B: Detección de forma aguda: CLINICA + SEROLOGÍA

• HBsAg (marcador de infección por VHB) • HBeAg (marcador de replicación viral e infecciosidad)

HEPATITIS AGUDA POR TOXICOS

• Antecedente de ingesta de fármaco hepatotóxico

LABORATORIO HEPATITIS AGUDA ALCOHOLICA

• CLINICA + pruebas de laboratorio + antecedente de ingesta. • Laboratorio. Aumento típicamente no muy marcado de las transaminasas -en

general menor de 400 UI/l-, predominando la GOT (cociente GOT/GPT >2), leucocitosis, datos de daño de la función hepática -alargamiento t protrombina, trombopenia, albúmina baja, hipergammaglobulinemia con ascenso marcado de IgA.

MANEJO

TRATAMIENTO - HEPATITIS A

• Sintomático. • Reposo por 2 semanas. • Rara vez requiere hospitalización. Se sugiere si:

• Sospecha de Insuficiencia hepática aguda < 1%

Page 127: Temas de examen Pediatria

• Tiempo de protrombina < 50% • Presencia de encefalopatía • Cuadro inicial severo, deshidratación, dolor abdominal intenso.

• Inmunoglobulina G (policlonal sérica) < 1 año: Pre y post exposición, inmunidad pasiva, dura hasta 6 meses. Dosis 0.02 ml/kg IM

TRATAMIENTO - HEPATITIS B

• Sintomático: Tratamiento no ha demostrado beneficio clínico • Específico: lamivudina 3 mg/kg día (máximo 100 mg/dia), aprobada en niños.

También se puede utilizar Interferon alfa • Curso severo o prolongado

• Cuagulopatia INR >1,5 • Bilirrubina >10 mg/dL por más de 4 semanas • Falla hepática aguda

TRATAMIENTO - HEPATITIS AGUDA ALCOHOLICA

• En los casos severos están indicados los glucocorticoides; en el resto, medidas sintomáticas, nutrición y vitaminas.

COMPLICACIONES

HEPATITIS A

• Hepatitis Recurrente • Falla hepática aguda < 1% casos. Más común en pacientes con enfermedad

hepática subyacente

HEPATITIS B

• Insuficiencia hepática aguda, con encefalopatía, coagulopatias y edema cerebral. • Cirrosis infrecuente durante la infancia • Hepatocarcinoma • Mortalidad Aguda 0.5-1%

Page 128: Temas de examen Pediatria

27. TALLA BAJA

Apuntes de clase “Trastornos del crecimiento y desarrollo” realizada por Dra.Molina, UFRO-2010.

Seminario “Talla Baja”, Dra Karina Dinamarca, UFRO.

DEFINICIÓN

• Talla por debajo del percentil 3 o menos de - 2DS en las curvas (para T/E).

Ojo: Discordancia con talla target y Desaceleración en velocidad de crecimiento.

La gran mayoría de las tallas bajas serán de tipo familiar y la talla baja constitucional. Y el resto tendrá algo endocrino o sistémico, entre otras.

CONSIDERACIONES:

• Muchos niños cambian de percentil los primeros 6 meses de vida, pues se van acomodando a su talla genética.

• Dentro de los 2 a 3 años de vida puede haber un cambio de percentil normal, no >2 (o sea un cambio de más de 2 percentiles no sería normal)

• Todo cambio de percentil después de los 3 años en adelante es anormal. • No olvidar el crecimiento esperado normal según edad.

REGULADORES DEL CRECIMIENTO Y DESARROLLO

Intra-uterina

• Fx maternales: nutrición, hábitos, enfermedades. • Fx de promoción del crecimiento : IGF I y II y otros • Fx placentarios: adecuado funcionamiento del sinciciotrofoblasto • Insulina

2-3 años

• Depende principalmente de la nutrición, Pero también hay persistencia de efectos de factores que influyeron en el crecimiento fetal.

• Hormonas tiroideas regulan la secreción de GH.

>3 años • Controlado principalmente por GH.

Pubertad • Regulado por una combinación de GH y esteroides sexuales. • Los esteroides aumentan la secreción endógena de GH y la producción

de IGF-I. • Se produce el estirón puberalcon crecimiento fijo.

1ºaño:

•25 cm/año

2º año:

•11cm/año

3ª – pubertad:

•4-7cm/año

Pubertad: crecimiento fijo

•Mujer: 20-25 cm

•Hombre: 25-30 cm

Inicio pubertad: • Mujer: Tanner 2 • Hombres: testículo 2,5cm = 4ml

Page 129: Temas de examen Pediatria

SOSPECHA DIAGNÓSTICA

• Cambio de más de 2 percentiles dentro de los 2 a 3 años de vida. • Todo cambio de percentil después de los 3 años en adelante es anormal. • Bajo 2 DS o bajo p 3.

EVALUACIÓN INICIAL

1. Anamnesis: • EMBARAZO: enfermedades maternas previas, enfermedades propias

del embarazo, medicamentos, drogas, radiación, etc. • CRECIMIENTO INTRAUTERINO: antecedentes ecográficos. • PERINATALES: Peso y Talla de nacimiento, Apgar, Prematurez, PEG,

etc.. 10 y 20 % de los PEG no hacen un crecimiento compensador, permaneciendo bajo p3. Estos niños se benefician de terapia con hormona de crecimiento, por lo que es aconsejable que se mantengan en control pediátrico y si a los 4 años no se han logrado ubicar en su canal de crecimiento según la carga genética, se deriven a endocrinología para eventual tratamiento con GH.

• ANTECEDENTES MÓRBIDOS: Enfermedades respiratorias, Gastrointestinales, Renales, Cardíacas, etc.

• DESARROLLO PSICOLÓGICO: comportamiento social, rendimiento escolar, maltrato, enuresis, encopresis, insomnio, uso de psicofármacos o drogas, etc.

• ALIMENTACIÓN: calidad y cantidad de nutrientes que recibe, trastornos de la conducta alimentaria (anorexia, bulimia). La encuesta alimentaria debe incluir: tiempo de lactancia materna exclusiva, tipo y horarios de alimentación, interés por la comida, vómitos etc.

• ACTIVIDAD FÍSICA: cantidad y calidad. Es conocido el ejercicio excesivo en período de crecimiento rápido (pubertad) compromete la velocidad de crecimiento.

• Talla padres y patrón de crecimiento

2. Examen físico: • Completo acusioso y detallado • Medición

antropométrica incluye: Peso , Talla, Circunferencia cráneo, Segmentos corporales, Envergadura* Evaluación nutricional

Varones: (talla mamá + talla papá) + 13 2

Mujeres: (talla mamá + talla papá) – 13 2

A este valor se le aplica la desviación estándar que es de 4,5 cm en mujeres y 5 cm en varones.

Page 130: Temas de examen Pediatria

• Medir al niño, en lugar adecuado donde por lo menos tenga 3 puntos de apoyo. Lactantes en posición supina (longitud) en un podómetro. Niños mayores: medición de pie (estatura) en estadiómetro.

• La ENVERGADURA o brazada, es la medición del la distancia entre el

extremo distal de ambos dedos medios al extender los brazos de manera perpendicular al eje axial. Un aumento de esta relación se observa en Sd. De Marfan, Neoplasia Endocrina Múltiple tipo 2B, Homocistinuria, Anormalidades Vertebrales y Eunucoidismo. Se encuentra disminuido en pacientes con Displasias esqueléticas.

• Evaluar si se trata de una TALLA BAJA PROPORCIONADA O

DESPROPORCIONADA. Para esto debemos medir los segmentos corporales y que correspondan con su desarrollo adecuado para cada edad. El segmento inferior es la distancia desde el borde superior de la sínfisis púbica y el suelo (con separación de pies de 3 cm), y el segmento superior es la diferencia de la talla y el segmento inferior.

• Si es proporcionada evaluar relación peso talla. (IPT): • Si es normal o esta aumentado (sobrepeso y talla baja) es muy

sospecho de presentar una alteración endocrina (las más frecuentes son: Déficit de GH, el déficit de hormona tiroidea, el exceso de cortisol (Sd cushing) y Sd Turner.

• Evaluar DENTICIÓN. Los niños hipotiroideos se retrasan en la

aparición de los dientes.

• Buscar estigmas genéticos o fascies características: Sd Turner, Meckel, Cornelia de Lange, etc.

• Perímetro cefálico y dismorfias : Descartar también dismorfias que

puedan orientarme a Sd. Genéticos como sd down , prader wily, Turner, silver russel

3. Curva de crecimiento: • Evaluar canal de crecimiento, ver en que percentil está creciendo el

niño(a), requiere varias mediciones. 4. Velocidad de crecimiento • Evaluar velocidad de crecimiento y extrapolar los cm que crece al año,

para esto requerimos por lo mínimo 6 meses de diferencia una medición a otra

5. Determinación edad ósea: • Se efectúa mediante una radiografía de mano y muñeca no

dominante, las placas se observan en Atlas de Greulich y Pyle. En menores de 1 año usar rx de rodillas

• Lo normal es + - 2 DS o + - 2 años.

Page 131: Temas de examen Pediatria

DIAGNÓSTICO DIFERENCIAL

Talla baja familiar

“Grupo heterogéneo en los cuales el origen del compromiso de la talla no puede ser explicado completamente con los métodos disponibles a la fecha” Se define por: 1. Ser adecuado para su edad gestacional 2. Proporciones corporales normales 3. Sin evidencia de patologías sistémicas crónicas,

endocrinológicas, ni de orden psiquiátrico. 4. Adecuada ingesta de alimentos. 5. Antecedentes de familia materna y/o paterna con

tallas bajas. 6. Talla de nacimiento normal o baja, desaceleran su

crecimiento los primeros años de vida. 7. Velocidad de crecimiento adecuada, por un canal

bajo la mediana. 8. Peso adecuado a la talla. 9. Estudio paraclínicos creeningnormal. 10. Rx. edad ósea concordante con edad cronológica. 11. Pronóstico de Talla final similar a talla de padres.

Retraso constitucional

1. Ambos padres talla normal. 2. En un alto % antecedentes de desarrollo puberal tardío

en padres u otro familiar cercano. 3. Peso y talla de nacimiento normal 4. Desaceleran su VC después de los 6 meses, estabilizando

su curva alrededor de los 3 años, posteriormente VC

Proporcionadas

VC normal

Talla baja familiar

Retraso constitucional

Desaceleración VC, con PT disminuido

Causas sistémicas nutricionales. Enf. crónicas no endocrinas

Desaceleración VC, con PT aumentado

Enf. endocrinas

Sd Turner

Desproporcionadas

Compromiso de extremidades

Acondroplasia, Displasias epifisiaria múltiple

Compromiso tronco y

extremidades

Acondrogénesis, Displasisa condroectodérmica,

Raquitismo

Hipotiroidismo de larga evolución

Page 132: Temas de examen Pediatria

normal, por debajo del canal de -2DS. 5. Se descartó patología sistémica. 6. RX edad ósea con retraso de más de un año. 7. Inicio puberal tardío. 8. Pronóstico de talla igual o superior a padres.

Enfermedades crónicas no endocrinas

• Talla baja proporcionada. Velocidad de crecimiento subnormal • Relación P/T disminuida • EO retrasada en relación a EC Causas: • Gastrointestinales : Sd. Malabsorción -> EC • Renales: ATR, ERC. • Pulmonares: FQ • Cardiopatías: CC cianóticas y con shunt izquierda a derecha. • Anemias: ferroprivas, hemolíticas (talasemias) • Deprivación psicosocial • Desnutrición

Enfermedades endocrinas

• 5 a 10 % de todos los casos deTB • Talla baja proporcionada, EO retrasada • Relación P/T normal o aumentada Causas: • Hipotiroidismo

Forma congénita: talla normal al nacer, con rápido deterioro postnatal del crecimiento estatural y del desarrollo psicomotor.

Forma adquirida, el primer signo notorio es la frenación del crecimiento, pudiendo pasar inadvertidos los otros síntomas de hipofunción.

• Deficiencia de hormona de crecimiento

Se presenta más en varones (4:1)

Congénitos: asociados o no a alteraciones craneofaciales y/o malformaciones del SNC.

Adquiridos: TEC, radiación, neoplasias, idiopático. • Hipercortisolismo

Asociado a: obesidad, HTA, fascie de luna, distribución troncal de grasa.

SÍNDROME DE TURNER

Desaceleración VC + P/T normal o ↑

98% presenta talla baja

60% tiene la forma clásica de disgenesia gonadal 45 xo y todas las características somáticas típicas.

Talla de nacimiento normal o baja. EO habitualmente = EC.

P/T normal o aumentado

Se debe indicar GH apenas quiebre curva de crecimiento (+8,5cm).

El 40% restante, puede tener como única manifestación el retraso de talla

Estas pacientes no presentan el estirón puberal.

Page 133: Temas de examen Pediatria

MANEJO INICIAL

• Se recomienda estudio en pacientes con TB < 3DS o bien con VC baja.

DERIVACIÓN

• Una vez descartada presencia de enfermedades sistémicas, el paciente debe ser derivado a especialista.

Page 134: Temas de examen Pediatria

28. VACUNAS DE USO HABITUAL

MINSAL- Clase Dr. Soza, Tutorial Dra. González.- Laboratorio GlaxoSmithKline

CON SI DERA CI ON E S GE N ERAL E S

Los niños y niñas que fueron prematuros/as deben vacunarse según su edad cronológica, por ejemplo, si tiene 2 meses de nacido debe recibir las vacunas que reciben los niños/as de esa edad, sin importar cuánto se adelantó su nacimiento. Excepto la BCG que debe colocarse cuando el RN pese al menos 2000 grs. Las vacunas con microorganismo vivo atenuado están contraindicada en niños con terapia inmunosupresora: prednisona o su equivalente en dosis de 2 mg/K/día hasta 3 meses de terminada dicha terapia, y en niños que estén recibiendo dosis decrecientes de corticoides, hasta 3 meses de recibir dosis <0,5 mg/K/día y en pacientes con infección por VIH sintomáticos. Si se colocan Ig se pueden administrar Ag vivos atenuado sólo después de 6 semanas, excepto Ig para la enfermedad de Kawasaki, en la que se debe

Page 135: Temas de examen Pediatria

esperar al menos 11 meses entre la administración de la vacuna y la vacunación con microorganismos vivos atenuados.

REACCIONES ADVERSAS

Locales: casi el 50% de las vacunas inyectables provocan dolor en el sitio de vacunación, enrojecimiento y/o aumento de volumen transitorio con induración alrededor del sitio de inoculación. Generales: Fiebre (máx.48 horas), irritabilidad y pérdida del apetito.

CONTRAINDICACIONES ABSOLUTAS

• Reacción anafiláctica a dosis previa de una vacuna. • Antecedente de encefalopatía en los 7 días siguientes a vacunación con

pentavalente o dTpa. • En el caso de los virus vivos atenuados, es contraindicación absoluta:

inmunodeficiencia congénita, terapia inmunosupresora prolongada y embarazo o posibilidad de embarazo en los próximos 4 meses.

INCUMPLIMIENTO DEL CALENDARIO DE VACUNACIÓN

• Si se interrumpió: se debe completar el calendario, administrando las dosis que faltan hasta la edad actual. No es necesario reiniciarlo.

• Si le faltan dosis sucesivas: se debe recalendarizar, reproduciendo el intervalo entre las dosis.

• Si los padres no se acuerdan, mejor administrar, ya que no existen evidencias de riesgos al repetir dosis de vacunas, excepto la DTP de células enteras, la que no debe administrarse después de los 6 años.

DETALLE CALENDARIO DE VACUNACIÓN (PNI) Actualizado el año 2013

EDAD VACUNA

RN PN

I

BCG (desde 1949)

> 2 meses

Extr

a P

NI

Antimeningocócica

2 y 4 meses PN

I

Pentavalente: Hep B (desde 2005), Hib (desde 1996), DTP (desde 1954).

Polio Oral (VPO) (desde 1978).

Neumocócica conjugada Synflorix®(desde 1° enero 2011)

> 2 meses

Exrt

a

PN

I

Neumocócica conjugada PREVENAR ®

Rotavirus Oral

Infanrix hexa®, reemplaza la pentavalente+VPO

6 meses PN

I Pentavalente

Polio Oral

Page 136: Temas de examen Pediatria

>6 meses <23 meses P

NI

Influenza

1 año PN

I Tresvírica : Sarampión, Rubeola, Paperas desde 1990

Neumocócica conjugada (Synflorix®) desde 2010

>1 año Ex

tra

PN

I

Antihepatitis A (nacidos después 2005)

Combinada hepatitis A y B (nacidos antes 2005)

Anti Varicela (VARILRIX®)

18 meses PN

I Pentavalente

Polio Oral (refuerzo)

4 años 2012: se suspende DTP

>5 años

Extr

a P

NI

Fiebre Tifoidea

1º Básico PN

I Tresvírica

dTp (acelular) (desde 2012)

2° Básico 2012: se suspende DT

8º Básico PN

I

dTp (acelular)

Niñas pre-adolescentes Ex

tra

PN

I

Papiloma virus

TI PO S D E VACUN AS

Page 137: Temas de examen Pediatria

VACUNA S V IV A S ATENU ADAS

BACTERIAS VIVAS ATENUADAS

BCG (deriva de Bacilo de Calmette y Guerin)

Aplicación: RN (1 dosis, el 2005: se suprimió el refuerzo de 1° básico)

Chile fue un pionero regional, en el inicio de la vacunación nacional con B.C.G. en 1949

• Única vacuna que no posee el mismo microorganismo contra el que se quiere prevenir. Contiene Mycobacterium bovis

• No evita la enfermedad en personas expuestas al bacilo, justificándose su uso fundamentalmente porque evita las formas graves de la enfermedad. Eficacia protectora del 75% contra las formas extrapulmonares, principalmente la miliar y meníngea

• El tiempo de protección de la vacuna BCG es de aproximadamente 15 años. • Se debe dar con un intervalo de 4 semanas con la de los 2 meses. • En general la vacuna se considera segura, con una incidencia de efectos

adversos de 0.1 a 3.6% como adenitis supurada (1/100), osteítis (1/100.000), lupus vulgaris y BCG diseminada (1/1.000.000)

• Se espera que después de 2 o 3 semanas, en el sitio de la vacuna aparezca una inflamación enrojecida de 5-10 mm, no dolorosa. A veces aparece una secreción amarilla y puede demorar semanas en cicatrizar, dejando una marca. Se recomienda mantener esta zona seca, descubierta y no aplicar alcohol, cremas o desinfectantes. Si se moja durante el baño secar suavemente.

• La ausencia de la reacción local en el sitio de punción, no implica una menor respuesta inmune.

VIRUS VIVOS ATENUADOS

Tresvirica (MMR)

Sarampión (Measles); Paperas (Mumps); Rubeola (Rubella)

Dosis: 1 año y 1° básico. Inyección SC

• Con este esquema la seroconversión para sarampión y parotiditis es 95% y para rubéola 98%. La protección conferida por esta vacuna es duradera y probablemente por toda la vida.

• Se coloca a los 12 meses y no antes, porque el niño a los 12 meses pierde la inmunidad materna (mujeres vacunadas con esta vacuna inducen inmunidad transplacentaria a sus hijos)

• La inmunidad dura 15 años o más. • Por la Anti-Paperas: previene las localizaciones meningoencefálicas y las

secuelas como sordera. • Está contraindicada en inmunosuprimidos. • Reacciones: Entre 5-10 días después de administrada la vacuna puede haber

fiebre moderada, romadizo, inflamación de ganglios y pintas rosadas en la piel. Si aparece fiebre, desabrigar, dar líquidos y administre paracetamol

Page 138: Temas de examen Pediatria

Polio oral (VPO de Sabin))

Aplicacións: 2, 4, 6 y 18 meses

Es trivalente: virus polio 1, 2 y 3.

La vacunación anti poliomielitis se inició en Chile en 1961 con vacuna inyectable (Salk) y a fines de 1962 se continuó con vacuna oral (Sabin), hasta nuestros días.

• Si el paciente vomita durante los 10 primeros minutos posteriores a la ingestión de la dosis, es indicación para volver a administrarla.

• Contraindicada en pacientes inmunosuprimidos y sus hermanos, en ellos se recomienda la VPI

• Generalmente no produce reacciones. En 1:1.000.000 puede producir la misma enfermedad.

• Como en ocasiones este virus vacuna puede adquirir características epidémicas y producir en algunos casos poliomielitis paralítica asociada a vacuna (PPAV), los países industrializados, que llevan varios años sin casos de enfermedad adoptaron el cambio de vacuna oral a vacuna inyectable (VPI) que evita los casos de parálisis post vacuna.

Varicela VARILRIX®

>1año: 2 dosis

>12 años: 2 dosis (0,4 meses)

• Se recomienda a partir del año. • Protege entre 70-90% contra la infección y un 95% contra manifestaciones

graves de la enfermedad. • Se puede colocar el mismo día que la tresvírica, pero diferente sitio anatómico.

Si no se coloca el mismo día, deben estar separadas por al menos 4 semanas.

Rotavirus (Rotarix® / Rotateq ®)

Dosis Rotarix®: 2, 4 meses (se puede dar desde las 6 semanas a 6 meses)

Dosis Rotateq®: 2, 4 y 6 meses

• Se administra por via oral y se excreta por deposiciones. • La edad máxima para comenzar la vacunación es a los 3 meses y medio. Y la

edad máxima para aplicar la última dosis es a los 8 meses. • Existen 2 tipos: Rotarix® que es Monovalente y Rotateq®, que es pentavalente. • Está contraindicada en inmunosuprimidos.

VACUNAS ENTERAS INACTIVADAS

BACTERIAS ENTERAS INACTIVADAS

Antipertussis entera

Dosis: 2, 4 , 6, 18

Bacilos inactivados de Bordetella pertussis.

• Dura: 6-8 años. • Contiene Thimerosal

Page 139: Temas de examen Pediatria

• Permite que la enfermedad se mantenga en niveles endémicos, con brotes interepidémicos cada 2 a 3 años. El impacto sobre el número de muertes es notable.

• Es reactogénica: > 6 años • Su presencia en la vacuna DTP, hace que presenta mayor riesgo de presentar

reacciones adversas como fiebre, eritema, edema, dolor, irritabilidad, decaimiento, anorexia, vómitos.

Cólera

>2años

• La vacunación contra el cólera no previene la introducción de la enfermedad a un país. Por ello, la se modificó el Reglamento Sanitario Internacional en 1973 con el propósito de que no se exija a ningún viajero la vacuna contra el cólera.

• La vacuna posee una eficacia baja, por lo que dan un sentido falso de la seguridad.

VIRUS ENTEROS INACTIVADOS

Polio Inyectable (Salk o VPI)

• Evita los raros casos de parálisis fláccida aguda (PFA) por efecto de la VPO, especialmente las 1ª dosis.

• La VPI induce anticuerpos de tipo sérico, también provoca protección mucosa pero de menor intensidad que la oral.

• Existe en su versión inyectable sola y como parte de la Hexavalente (Infanrix®)

Hepatitis A (Havrix ® y Epaxal ®)

Se recomienda en >1año. Dosis: 0 y 6 meses / IM

• Dura por más de 20 años. • La hepatitis A es la primera causa (por magnitud) de hepatitis fulminante en

Chile. • Chile se encuentra en una condición denominada endemia intermedia (tasa 0,3-

88 casos por 100.000 hbtes). • Eficacia protectora de casi 100%.

Influenza Estacional

Dosis: monodosis, entre los 6-23 meses

• Los virus influenza A, son clasificados en sub-tipos de acuerdo a dos antígenos de superficie: Hemaglutina (H) y Neuramidasa (N).

• Cada año la vacuna contiene 3 cepas virales. Usualmente 2 de tipo A y 1 de B, que se estima corresponden a los que circularán en la próxima estación.

Antirrábica

Dosis: 0, 7, 14 y 28

• Se usa en individuos expuestos (mordidos por perro, murciélago o gato sospechoso) y aquellos con riesgo de exposición, como veterinarios, personal de

Page 140: Temas de examen Pediatria

laboratorio que trabaje con el virus rábico, taxidermistas y cuidadores de animales, y cazadores.

• Lo ideal es poder observar al animal durante 10 días, si no se puede, es mandatoria la vacunación inmediata.

VACUNAS FRACCIONADAS INACTIVADAS

PROTEICAS

Antihepatitis B

Dosis IM: 2, 4 , 6, 18 meses (como parte de la pentavalente)

Se incluyó a partir del año 2005 en el programa nacional

• Contiene Thimerosal • La aplicada en Chile corresponde a la de segunda generación, que es resultado

de la aplicación de la biología molecular y de la recombinación genética del ADN lo que ha permitido expresar el HBsAg del virus en diversas células entre las que se destaca la levadura común, Saccharomyces cerevisiae.

• Su objetivo principal es impedir la infección crónica, sus secuelas y el estado de portador. El objetivo secundario es imposibilitar la infección aguda.

• Las vacunas actualmente disponibles, son seguras y con una efectividad de más de 95% en la prevención de infección crónica.

• Dura 10 años • Se aplica por vía IM en región deltoídea, excepto RN y lactantes en región

antero-lateral del muslo. • Esquema de vacunación en adolescentes o adultos no vacunados: 0, 1 y 6

meses. Pero se recomienda aplicar la Vacuna combinada Twinrix®, ya que es suficiente con 2 dosis (a los 0 y 6 meses).

Pertussis acelular

Dosis: 1° y 8° básico

• Minimiza las reacciones adversas de la vacuna inactivada entera o completa.

Papiloma Virus

3 dosis a partir de los 9 años hasta los 26 años

Dosis Gardasil ®: 0, 2, y 6 meses

Dosis Cervarix ®: 0, 1 y 6 meses

Son vacunas recombinantes. Existen 2 tipos: • Gardasil ®:tetravalente- VPH 6,11,16y18 • Cervarix ®: bivalente – VPH 16 y 18

POLISACARIDAS PUROS

Haemophilus Influenzae tipo B (Hib)

Dosis IM: 2,4,6 y 18

Se introdujo desde julio de 1996 en Chile.

• Forma parte de la Pentavalente. Contiene Thimerosal

Page 141: Temas de examen Pediatria

• Los casos clínicos de enfermedad invasora por Hemophilus influenzae tipo b que se presentan en forma esporádica, en nuestro país se debe en la inmensa mayoría de ellos a programas incompletos o niños no respondedores o inmunosuprimidos.

• Reacciones locales como dolor, enrojecimiento, inflamación y generales como pérdida de apetito, fiebre, somnolencia, irritabilidad, diarrea y vómitos.

Meningocócica no conjugada

Dosis: >2 años Dura 3-4 años

• Contra serogrupos A, C, Y y W-135.

Neumocócica no conjugada (Pneumo 23 ®)

Dosis: >2 años

• Al ser no conjugada, sólo contiene la pared celular (no proteína), por lo tanto son poco inmunogénicas en niños <2 años.

• Serotipos: 1, 2, 3, 4, 5, 6B, 7F, 8, 9N, 9V, 10A, 11A, 12F, 14, 15B, 17F, 18C, 19A, 19F, 20, 22F, 23F, 33F.

• Se recomienda en pacientes con riesgo de adquirir infec. neumocócicas sistémicas:

• Asplenia funcional o anatómica, • Sd nefrótico o falla renal crónica, • Anemia a células falciformes, • Inmunosupresión: transplantes o terapia de citoreducción, • Pérdida de LCR, Infección por HIV

• Se considera como parte del PNI en personas de 65 años.

POLISACARIDAS CONJUGADAS

Meningocócica conjugada (Nimenrix®)

Tetravalentes: Neisseria meningitidis serogrupos A,C, W135 e Y

Se recomienda en >1 año, 1 dosis IM (deltoide o muslo)

Puede administrarse concomitantemente con cualquiera de las siguientes vacunas: las vacunas contra la varicela, hepatitis B y la hepatitis A, la tresvirica, la antineumocócica conjugada o la contra la influenza estacional.

Neumocócica 10 Valente (Synflorix®)

Se aplica 2,4 y 12 meses, IM (en el muslo contralateral de la pentavalente)

10 valente: 4, 6B, 9V, 18C, 19, 23F, 1, 5, 7F.

• Produce una inmunización activa de lactantes y niños a partir de 6 semanas hasta 2 años de edad contra la enfermedad causada por los serotipos 1, 4, 5, 6B, 7F, 9V, 14, 18C, 19F y 23F de Streptococcus pneumoniae (incluyendo sepsis, meningitis, neumonía, bacteremia y otitis media aguda) y contra la otitis media aguda causada por Haemophilus influenzae No Tipificable.

• Es la única que incluye protección contra Haemophilus influenzae no tipificable, lo que parece ser una ventaja real, pero difícil evaluar

• Reacciones: Dolor en la zona inyectada. En caso de fiebre dar paracetamol

Page 142: Temas de examen Pediatria

Neumocócica conjugada (PREVENAR13 ®)

A partir de las 6 semanas de vida hasta los 5 años

Sin vacuna neumocócica previa: 3 dosis en intervalo de 2 meses.

13 valente: 4, 6B, 9V, 14, 18C, 19F, 23F, 1, 5, 7F, 3, 6A, 19A.

• La incorporación del serotipo 19A le da leve ventaja frente a la 10 valente. • Es utilizada para prevención de infección invasora, neumonía y otitis media

aguda causada por neumococo en lactantes y niños desde las 6 semanas hasta los 5 años de edad. También está indicada para la inmunización activa para la prevención de la enfermedad invasiva causada por neumococo en adultos de 50 años de edad o mayores.

• Se puede ofrecer para que sea colocada asociada a la del PNI, no son excluyentes entre sí. Se considera que los niños hasta los 5 años que hayan recibido la pauta completa con Prevenar7® o Synflorix®, se beneficiarán de una dosis adicional de Prevenar 13® (dosis de rescate) administrada con un intervalo mínimo de 8 semanas tras la última dosis de vacuna antineumocócica, ya que permite ampliar la protección frente a los serotipos que añade la Prevenar 13®.

VACUNAS ASOCIADAS

DTP:

Se introdujo en Chile en 1954

Protege por 7- 10 años

• Es una inyección IM que incluye 3 vacunas: toxoide diftérico, toxoide tetánico, pertussis entero.

• Desde 1973 el coqueluche se presenta en brote epidémico cíclico. • En el caso del tétano y la difteria, estas se presentan casi exclusivamente en

pacientes no vacunados o con esquemas incompletos. • RAM: fiebre (incluso >40°C), eritema (incluso >20mm), edema (incluso >20mm),

dolor, irritabilidad, decaimiento, anorexia, vómitos.

dTpa:

• Es una inyección que incluye 3 vacunas: toxoide diftérico en dosis reducida, toxoide tetánico y dosis reducida de pertussis acelular. Si bien se han registrado reacciones adversas con esta vacuna, son mucho menores y de menor intensidad que las que se pueden presentar con la DTP.

• La inmunogenicidad de las vacunas acelulares y su eficacia es comparable a las vacunas de células enteras, su ventaja más significativa es su menor reactogenicidad y la posibilidad de usar en niños >6 años, adolescentes e incluso en adultos.

• En relación a las complicaciones graves en menores de siete años, cuando se presentan, la revacunación no debe aplicarse nuevamente y estas contraindicaciones son tan válidas para las vacunas de células enteras como las acelulares, y son las siguientes:

Reacción anafi láctica dentro de los 3 días post-vacuna

Encefalopatía no atribuible a otra causa dentro de los 7días postvacuna

Enfermedad neurológica progresiva (Ej. convulsiones, sd de West, etc.)

Page 143: Temas de examen Pediatria

DT:

• Se utiliza en pacientes que sufren heridas traumáticas, luego de 10 años de haber finalizado el esquema de vacunación primaria y los refuerzos de la niñez.

Pentavalente

• Es una inyección que incluye cinco vacunas: contra la difteria, tétanos, pertussis, hepatitis B y la bacteria Haemophilus b

• Reacciones y cuidados: Puede aparecer fiebre por 2 ó 3 días e irritabilidad. En la zona de la vacuna puede aparecer una hinchazón rosácea y dolor. Generalmente deja una dureza bajo la piel que no duele pero puede durar semanas o meses. Para el dolor se recomienda aplicar paños fríos. Si aparece fiebre administrar paracetamol en la dosis recomendada por su pediatra y ofrézcale pecho con mayor frecuencia.

• Contiene Thimerosal

Infanrix hexa®

• Es una inyección que incluye 6 vacunas contra la difteria, tétanos, pertussis acelular , hepatitis B, Haemophilus b y Polio inyectable.

• No contiene Thimerosal

Twinrix ®

• Es una vacuna que contiene el virus de la hepatitis A (HA) purificado inactivado y el antígeno de superficie purificado de la hepatitis B (HBAgs).

• Esta indicado en niños nacidos antes de enero del año 2005. • Dosis en <16 años: 2 dosis (0, 6 meses) • Dosis en >16 años: 3 dosis (0, 1 y 6 meses)

VACUNA ANTI-INFLUENZA

La vacuna contra la influenza estacional 2012-2013 protege contra el virus de influenza A H3N2 el virus de la influenza B y el virus H1N1 2009. Inmunidad dura 1 año. Los lactantes y niños menores de 8 años deben recibir 2 dosis separados por 2 semanas Hay dos tipos de vacunas contra la influenza estacional con licencia de FDA: • Vacuna inactivada trivalente contra la influenza (TIV): un virus de la influenza A

(H3N2v), un virus de la influenza A (H1N1) y uno de la influenza B. Cada año, antes de que comience la temporada de influenza, una o más cepas de virus incluidos en la vacuna, son cambiados basándose en la vigilancia mundial de los virus de la influenza y la aparición y la diseminación de nuevas cepas.

• Vacuna contra la influenza atenuada en virus vivos (LAIV): se suministra en atomizador nasal. Puede ser utilizada en personas saludables de entre 2 y 49 años, que no estén embarazadas.

Page 144: Temas de examen Pediatria

Se recomienda vacunar la población de mayor riesgo: • Embarazadas, a partir de la 13ª semana de gestación; • Lactantes entre los 6 y los 23 meses; • Personas de 65 años y más; • Trabajadores de avícolas y de criaderos de cerdos y • Personas entre 2 y 64 años portadores de alguna condición de riesgo

Diabetes

Enfermedades pulmonares crónicas: asma bronquial; EPOC; fibrosis quística; fibrosis pulmonar de cualquier causa.

Cardiopatías: congénitas; reumática; isquémica y miocardiopatías de cualquier causa

Enfermedades neuromusculares congénitas o adquiridas que determinan trastornos de la deglución o del manejo de secreciones respiratorias.

Obesidad Mórbida

Insuficiencia renal en etapa 4 o mayor

Insuficiencia renal en diálisis.

Insuficiencia hepática crónica.

Enfermedades autoinmunes como Lupus; escleroderma; artritis reumatoidea, enfermedad de Crohn, etc.

Cáncer en tratamiento con radioterapia, quimioterapia, terapias hormonales o medidas paliativas de cualquier tipo.

Infección por VIH.

Inmunodeficiencias congénitas o adquiridas En las cuatro últimas condiciones de riesgo mencionadas puede considerarse, con indicación de médico tratante, retrasar la administración de la vacuna hasta la remisión o estabilización de la enfermedad de base, de modo de lograr una mejor respuesta inmune. La postergación se basa en lograr mejor inmunogenicidad de la vacuna, no en problemas de seguridad.

Contraindicaciones para la vacunación anti-Influenza:

• Reacciones alérgicas severas a algún componente de la vacuna en dosis previas • Reacciones alérgicas severas al huevo • Historia de Guillain-Barré, a dosis previas de Influenza. • Personas con terapia inmunosupresora (prednisona o su equivalente en dosis de

2 mg/Kg/día, hasta tres meses de terminada dicha terapia y personas que estén recibiendo dosis decrecientes de corticoides, hasta 3 meses de recibir dosis inferiores a 0,5 mg/K/día).

• Contraindicaciones temporales: personas con enfermedad aguda severa (Ej.meningitis, sepsis, neumonía).

Reacciones Adversas

• Las reacciones más frecuentes registradas luego de la vacunación en niños y adultos son dolor en el lugar donde se les inyecta, lo que normalmente desaparece en <2 días sin necesidad de tratamiento.

• Fiebre, malestar, mialgia y otros síntomas sistémicos uelen afectar más frecuentemente a personas que no han sido expuestas anteriormente a

Page 145: Temas de examen Pediatria

antígenos del virus de la influenza en una vacuna (por ejemplo, niños pequeños).

• Ocasionalmente se ha informado de síntomas oculares o respiratorios (por ej.: ojos rojos, voz ronca, tos) durante las 24 horas posteriores a la administración de la TIV, pero suelen ser leves y desaparecen rápidamente sin un tratamiento específico.

• Los componentes de la vacuna rara vez causan reacciones alérgicas reales, llamadas también reacciones de hipersensibilidad, entre algunos receptores. Los síntomas de hipersensibilidad inmediata van desde urticaria leve y angioedema (hinchazón bajo la piel) hasta anafilaxis.

• El monitoreo de la seguridad de las vacunas contra la influenza estacional a lo largo de muchos años no ha detectado una vinculación clara con el síndrome de Guillain-Barré (SGB). Sin embargo, si existiera un riesgo de GBS asociado con las vacunas contra la influenza actuales, no superarían los 1 o 2 casos por millón de personas vacunadas. Esto es mucho menor que el riesgo de contraer influenza grave, la cual puede prevenirse mediante la vacunación.

ANEX O : CA M PAÑA D E V ACUNAC IÓ N CO NTRA W-135

Comenzó en octubre del 2012 para todos los niños > 9 meses y < 5 años del país, por ser el grupo más vulnerable a la enfermedad. Entre octubre y diciembre del 2012 el 100% de los niños de la población objetivo fueron inmunizados. En marzo 2013 se inició una nueva etapa de la campaña, que busca: 1. Completar la protección de los menores de 2 años, ya que el sistema

inmunológico de éstos niños no es suficientemente maduro como para generar el nivel de anticuerpos necesario contra esta cepa de la bacteria.

2. Incorporar a los niños que han cumplido o cumplirán sus 9 meses entre el 1 de enero y el 30 de junio de este año.